You are on page 1of 177

CÓ CHÍ THÌ NÊN

TỔNG HỢP ĐỀ THI HSG


LỚP 10
OLYMPIC 30/4 CHÍNH THỨC
QUA 20 NĂM

Tài liệu khóa LIVE-VIP


HocvoiOttoChannel
trunng@gmail.com
www.ottochannel.vn
Ho Chi Minh City
Otto Channel
SỞ GIÁO DỤC & ĐÀO TẠO KỲ THI OLYMPIC TRUYỀN THỐNG 30/4
TP. HỒ CHÍ MINH LẦN THỨ XXV – NĂM 2000
ĐỀ CHÍNH THỨC Môn thi: Anh văn - Khối: 10
Thời gian làm bài: 180 phút

➢ Thí sinh làm phần trắc nghiệm (MULTIPLE CHOICE) trên phiếu trả lời trắc nghiệm và phần tự
luận (WRITTEN TEST) trên phiếu trả lời tự luận.
➢ Trên phiếu trả lời trắc nghiệm, thí sinh tô thêm 2 số 00 vào trước số báo danh (bằng bút chì).
Phần mã đề thi trên phiếu trắc nghiệm, thí sinh tô vào ô 002.
 --------------------------------------------------------------------------------------------------------------------

CẤU TRÚC ĐỀ THI


SECTION A. MULTIPLE CHOICE (30PTS)...................................................................................................... 2
PART I. PHONOLOGY (15PTS)........................................................................................................................ 2
PART II. VOCABULARY (5PTS) ......................................................................................................................... 2
PART III. READING COMPREHENSION (10PTS) .......................................................................................... 3

SECTION B. WRITTEN TEST (70PTS) ............................................................................................................. 4


PART I. CLOZE TEST (10PTS) ......................................................................................................................... 4
PART II. GRAMMAR AND STRUCTURES (40PTS) ....................................................................................... 4
PART III. SENTENCE TRANSFORMATION (20PTS) ..................................................................................... 5
Otto Channel
SỞ GIÁO DỤC & ĐÀO TẠO KỲ THI OLYMPIC TRUYỀN THỐNG 30/4
TP. HỒ CHÍ MINH LẦN THỨ XXV – NĂM 2000
ĐỀ CHÍNH THỨC Môn thi: Anh văn - Khối: 10
Thời gian làm bài: 180 phút

➢ Thí sinh làm phần trắc nghiệm (MULTIPLE CHOICE) trên phiếu trả lời trắc nghiệm và phần tự
luận (WRITTEN TEST) trên phiếu trả lời tự luận.
➢ Trên phiếu trả lời trắc nghiệm, thí sinh tô thêm 2 số 00 vào trước số báo danh (bằng bút chì).
Phần mã đề thi trên phiếu trắc nghiệm, thí sinh tô vào ô 002.
 --------------------------------------------------------------------------------------------------------------------

SECTION A. MULTIPLE CHOICE (30PTS)


PART I. PHONOLOGY (15PTS)
A. Pick out the words whose underline part is pronounced differently from those of others.
1. A. ginger B. ginseng C. gipsy D. girl
2. A. uncle B. bugle C. huge D. latitude
3. A. bargain B. complain C. ascertain D. campaign
4. A. evasion B. evasive C. desuetude D. desultory
5. A. trays B. says C. bays D. days
6. A. emblem B. electrification C. exact D. entire
7. A. chick B. chic C. cheat D. machismo
8. A. brood B. naughty C. foolscap D. brooch
9. A. slaughter B. naughty C. draught D. plaudit
10. A. fathom B. feather C. anthem D. within
B. Group the following words into columns according to their stress patterns.
coincide speciality ecipe cylinder personify
spectrometer melancholy altogether hereditary index

1st syllable 2nd syllable 3rd syllable

PART II. VOCABULARY (5PTS)


Choose the correct answer.
1. When the police examined the house, they found that the lock had been with.
A. touched B. tampered C. broken D. hindered
2. David's married Elizabeth? - No, I don't believe it! You're pulling my .
A. toe B. leg C. mind D. hair
3. I'm afraid I haven't got the idea why he never turned up.
A. least B. smallest C. weakest D. faintest
4. He was a very man; one day he would be happy, the next miserable.
A. uncontrollable B. uneven C. temperamental D. dispirited
2|Page Tài liệu khóa học Live - VIP
Otto Channel
Otto Channel
5. Do you know, she's bought curtain material exactly the same as ours. She's a dreadful .
A parrot B. dog-in-the-manger C. ape D. copy cat
6. That book looks like an advanced text to me.
A. economic B. economical C. economics D. economists
7. He didn't share his secret with many people but he in her.
A. confessed B. concealed C. confided D. consented
8. I was woken up by the sound of sheep in the meadows.
A. neighing B. crowing C. bleating D. croaking
9. The accusation left him quite with rage.
A. speechless B. silent C. dumb D. mute
10. He bought a pair of sunglasses with silver .
A. rims B. brims C. edges D. boundaries
PART III. READING COMPREHENSION (10PTS)
Read the following passage and choose the correct answers.
A new British medical instrument is about to change dramatically our ability to recognise disease
hidden inside the body. It is called the magnetic scanner, and it gives information about the body
which current machines, such as the brain scanner and the more recent body scanner, cannot provide.
And unlike existing machines, the new machine does not use X-ray radiation waves, which makes it
much safer for patients.
Unlike existing scanners, a magnetic scanner can show the side or the back of a suspect part of
the body. Used on, for example, a lung with suspected damage, it can turn the image of the lung to
give a series of different views. The image is not simply worked out in the computer's "imagination",
the magnetic scanner actually examines every side of the lung. When examining the softer part of the
body with existing scanners, doctors frequently have to use a “contrast medium" to make a muscular
organ show up to X-ray waves, the usual "contrast medium" given to patients before a stomach X-ray
scan takes place is harmless, but for some brain scans so that it passes into the brain, this is unpleasant
and painful and it can be dangerous. The magnetic scanner needs no “contrast medium".
The new machine can also see in greater detail than ever before. Growths of all kinds show up
clearly on the brain and so do the dead patches which are the signs of the disease, multiple sclerosis.
Until now patients complaining of double vision or an inability to control their muscles from time could
be suffering from some much more easily cured diseases and a brain scanner could not distinguish
between these with certainty. The doctor can now definitely say whether or not cancer of multiple
sclerosis is present. Previously, these diseases have been missed and patients have been given false
information about their health. The technique is still improving with great speed: pictures produced a
couple of years ago look very simple compared with those of today.
1. What does the new magnetic scanner machine do?
A. It looks at a diseased part of the body from all directions.
B. It can look right through a diseased part of the body.
C. It sends sound waves to the diseased part of the body.
D. It uses a computer to "imagine" different views of the body.

3|Page Tài liệu khóa học Live - VIP


Otto Channel
Otto Channel
2. What disadvantage do existing machines have?
A. Patients need an injection before they are used. B. The contrasts recorded are too strong.
C. It is often necessary to use a contrast medium. D. The X-ray waves are sometimes painful.
3. In what way is the new machine an improvement?
A. It can cure some diseases much more easily.
B. It will never give a double view.
C. The techniques involved can be learned quickly.
D. It can show all the details of a diseased part of the body.
4. The machine is useful in identifying the disease multiple sclerosis, because it shows
A. dark spots in the eyes. C. cancer like growths.
B. dead areas in the brain. D. damage to muscles.
5. "Sclerosis " means .
A. a discase that causes an organ or soft part of the body to become hard.
B. a disease that causes indigestion.
C. tuberculosis.
D. bronchitis.

SECTION B. WRITTEN TEST (70PTS)


PART I. CLOZE TEST (10PTS)
Fill in each numbered space with one appropriate word.
According to an old (1) in the early seventeenth century a Penvian was cured (2) a terrible
fever by eating the bark of the cinchona tree. Quinine, the drug which can be (3) from
cinchona bark, was (4) widely used as medicine until 1816.
Quinine has proved invaluable to modern medicine. It is used in the (5) of malaria, a (6)
transmitted by the anopheles mosquito and common in the tropical (7) of the world.
Quinine preparation are also used to help cure typhoid fever, rheumatic fever, and (8) sicknesses.
The cinchona tree belongs to the evergreen family and, unlike most evergreens, (9) very
fragrant flowers. Cinchona trees are native to South America but are now (10) in such other places
as India, Ceylon and Java.
PART II. GRAMMAR AND STRUCTURES (40PTS)
A. Supply the correct form of the word in the parentheses.
1. You cannot enter the country without the documents. (require)
2. The old man collapsed after doing some exercises. (strain)
3. The of water is necessary. (pure)
4. She is training to be a . (beautiful)
5. This houses the largest telescope in the country. (observe)
6. Many beliefs of a culture are represented in rituals. (symbol)
7. I've never known such a person. (quarrel)
8. Thank you for your generosity; we shall always be to you. (debt)

4|Page Tài liệu khóa học Live - VIP


Otto Channel
Otto Channel
9. Professor Smith has joined the company in an capacity. (advise)
10. Please our letter of the 10th. We have not had a reply. (know)
B. Fill in each blank with one appropriate preposition or particle.
11. Her husband was courteous her.
12. You should comply the school rules.
13. Be calm and don't make a fuss such trifles.
14. John's very good his hands.
15. She knows how to get her father to buy her a new car.
16. The police are looking the past record of the suspect.
17. When she heard the news of the crash, she broke and cried.
18. She always sides her son against her husband.
19. In the warm sun, the grass dries quickly.
20. You shouldn't look down on people who aren't as well as you are.
C. Use the correct form of the verbs in brackets.
21. I can't bear the thought of you (go) home without someone (accompany) you.
22. I'd rather (live) in ancient Greece.
23. He insisted that the new baby (name) after his grandfather.
24. There (be) no further business, I declare the meeting closed.
25. The uneaten breakfast was still on the table. She (call) away in a hurry.
26. I (write) photographs every week, I promise, and I (try) to make them legible.
27. Were I (know) answer, I'd tell you right away.
28. It's no use crying over (spill) milk.
PART III. SENTENCE TRANSFORMATION (20PTS)
A. Rewrite the following sentences with the given words in such a way that the second sentence
has the same meaning as the first one. Do not change the form of the words in brackets.
1. Under no circumstances should you phone the police.
→ The last .................................................................................................................
2. Sam didn't celebrate until he received the prize.
→ It was not ..............................................................................................................
3. Nobody expected her to lose, but she did.
→ Against .................................................................................................................
4. You'll learn the basic rules. Then you'll find it easy to play.
→ Once ....................................................................................................................
5. If nothing unfortunate happens, I'll see you next week.
→ All being ................................................................................................................
6. The government have been reviewing their education policies recently.

5|Page Tài liệu khóa học Live - VIP


Otto Channel
Otto Channel
→ The government's .................................................................................................
7. When a policeman appeared at the door, I was pretty taken aback.
→ Imagine ................................................................................................................
8. My protests were ignored.
→ Nobody .................................................................................................................
9. It's what people eat that betrays their social background, not their table manners.
→ What gives ...........................................................................................................
10. The chances are a hundred to one against you.
→ It's most ...............................................................................................................
B. Rewrite the following sentences with the given words in such a way that the second sentence
has the same meaning as the first one. Do not change the form of the words in brackets.
11. Tasteful furniture made your room more attractive. (GRACED)
→ ............................................................................................................................
12. The train is 5 minutes late in leaving. (DUE)
→ ............................................................................................................................
13. We had a long discussion about the problem but we couldn’t solve it. (LENGTH)
→ .............................................................................................................................
14. The river Volta overflowed last year. (BURST)
→ .............................................................................................................................
15. You didn't think carefully enough before you decided. (OUGHT)
→ ............................................................................................................................
16. I think you should try and be optimistic as you can. (SIDE)
→ ............................................................................................................................
17. Don't misquote: I never said I hated ballet. (MOUTH)
→ ............................................................................................................................
18. While they were on leave, the sailors exploited their freedom fully. (MOST)
→ ............................................................................................................................
19. He shouldn't have told her such things. (UNWISE)
→ ............................................................................................................................
20. Teaching doesn't suit her (CUT)
→ ............................................................................................................................

THE END

6|Page Tài liệu khóa học Live - VIP


Otto Channel
Otto Channel
SỞ GIÁO DỤC & ĐÀO TẠO KỲ THI OLYMPIC TRUYỀN THỐNG 30/4
TP. HỒ CHÍ MINH LẦN THỨ XXV – NĂM 2001
ĐỀ CHÍNH THỨC Môn thi: Anh văn - Khối: 10
Thời gian làm bài: 180 phút

➢ Thí sinh làm phần trắc nghiệm (MULTIPLE CHOICE) trên phiếu trả lời trắc nghiệm và phần tự
luận (WRITTEN TEST) trên phiếu trả lời tự luận.
➢ Trên phiếu trả lời trắc nghiệm, thí sinh tô thêm 2 số 00 vào trước số báo danh (bằng bút chì).
Phần mã đề thi trên phiếu trắc nghiệm, thí sinh tô vào ô 002.
 --------------------------------------------------------------------------------------------------------------------

CẤU TRÚC ĐỀ THI


SECTION A. MUTIPLE CHOICE (30PTS) ..................................................................................................... 1
PART I. PHONOLOGY (10PTS) ..................................................................................................................... 1
PART II. VOCABULARY (5PTS) ...................................................................................................................... 1
PART III. PARAGRAGH UNDERSTANDING (10PTS) .................................................................................. 2
PART IV. READING COMPREHENSION (5PTS) .......................................................................................... 3
SECTION B. WIRTTEN TEST (70PTS) ........................................................................................................... 4
PART I. CLOZE GUIDE (10PTS) ................................................................................................................... 4
PART II. WORD FORMATION (10PTS) ........................................................................................................ 4
PART III. GRAMMAR AND STRUCTURES (30PTS)..................................................................................... 5
PART IV. SENTENCE TRANSFORMATION (20PTS) ................................................................................... 6
Otto Channel

SỞ GIÁO DỤC & ĐÀO TẠO KỲ THI OLYMPIC TRUYỀN THỐNG 30/4
TP. HỒ CHÍ MINH LẦN THỨ XXV – NĂM 2001
ĐỀ CHÍNH THỨC Môn thi: Anh văn - Khối: 10
Thời gian làm bài: 180 phút

➢ Thí sinh làm phần trắc nghiệm (MULTIPLE CHOICE) trên phiếu trả lời trắc nghiệm và phần tự
luận (WRITTEN TEST) trên phiếu trả lời tự luận.
➢ Trên phiếu trả lời trắc nghiệm, thí sinh tô thêm 2 số 00 vào trước số báo danh (bằng bút chì).
Phần mã đề thi trên phiếu trắc nghiệm, thí sinh tô vào ô 002.
 --------------------------------------------------------------------------------------------------------------------
SECTION A. MUTIPLE CHOICE (30PTS)
PART I. PHONOLOGY (10PTS)
A. Pick out the word whose underlined part is pronounced differently from those of the others.
1. A. hasty B. nasty C. tasty D. wastage
2. A. chooses B. houses C. rises D. horses
3. A. homonym B. dishonesty C. honorable D. hourly
4. A. slaughter B. draught C. naughty D. plaudit
5. A. whistle B. thirst C. skirt D. reptile
6. A. adventure B. advantage C. advertise D. addition
7. A. chain B. entertain C. bargain D. complain
8. A. naked B. coughed C. hatred D. sacred
9. A. pleased B. closed C. raised D. embarrassed
10. A. assist B. must C. island D. mist
B. Group the following words into columns according to their stress patterns.
create jeopardize preferential invalid guarantee
extravagant diametre elasticity misunderstand economics
accidental coincide recipe communism nevertheless
tuberculosis geneticist tattoo politics cinematographer

1st syllable 2nd syllable 3rd syllable 4th syllable

PART II. VOCABULARY (5PTS)


Choose the best answer.
1. For people with hearing difficulties, telephones with volume controls provide the best .
A. reply B. response C. solution D. result
2. I've had two teeth taken out. It was OK at the time, but now my mouth really .
A. pains B. harms C. hurts D. injures

1|Page Tài liệu khóa học Live - VIP


OTTO CHANNEL
Otto Channel

3. He got an excellent grade in his exam the fact that he worked particularly hard.
A. in spite of B. on account of C. because D. although
4. There was a great scandal when it turned out that the Bishop, who was meant to be , had
a son
A. singular B. celibate C. immaculate D. separated
5. It is a long from Tokyo to London.
A. tour B. track C. flight D. travel
6. The acrobats at the circus performed some very impressive .
A. deeds B. exploits C. actions D. feats
7. I'm afraid I'm rather about existence of ghosts.
A. skeptical B. partial C. adaptable D. incapable
8. His answer was so confusing that I could hardly make any of it at all.
A. interpretation B. meaning C. intelligibility D. sense
9. If she can make up such stories, she is certainly a very girl.
A. imaginable B. imaginative C. imaginary D. imagining
10. During the First World War, an 5 million people lost their lives.
A. assumed B. estimated C. envisaged D. approximated

PART III. PARAGRAGH UNDERSTANDING (10PTS)


Read carefully and decide whether each of the following statement is true (T) or false (F).
Arrival:
When the plane lands, you are requested to keep your seat belt fastened until the light goes off and
remain in your seat until the plane stops moving. This is for your own comfort and safety.
Leaving the airport:
If you are ending your journey at this airport, you should go up to the escalator to passport control
and customs. If you are travelling further by rail or coach, you will find an information desk outside
the customs hall, as well as a bank where you can change money.
Catching another flight:
If you are planning to change planes, you should follow the signs to the waiting lounge, where you
should check in and wait to board your connecting flight. There will be an announcement when your
flight is due to take off. Our ground staff will be happy to help you.
Leaving the plane:
All passengers leaving the plane should make sure that they take everything with them. If you have
checked in any luggage, you should remember to collect it from the baggage area inside the airport.
Continuing on this flight:
We regret that passengers who are continuing their journey on this flight may not get off the plane.
You may not smoke while the plane is on the ground. Thank you for flying with us. We hope to be able
to welcome you on board again soon.

2|Page Tài liệu khóa học Live - VIP


OTTO CHANNEL
Otto Channel

1. All passengers must leave the aeroplane.


2. You must wait until the light goes out before undoing your seat belt.
3. Passengers who are leaving the airport must first go to passport control.
4. Passengers continuing by another plane must go through customs.
5. You can change money before you go to passport control.
6. You must go up the escalator to the waiting lounge if you are changing
plane.
7. Passengers in the waiting lounge will hear an announcement when their
plane is ready to leave.
8. No one is allowed to smoke in the airport.
9. The aeroplanc is going on to another place after some of the passengers
get off.
10. Passengers who are leaving the plane should check that they have not left
anything behind.

PART IV. READING COMPREHENSION (5PTS)


Read the following passage and choose the correct answers.
Many countries face a somewhat more serious economic problem in the form of an unfavorable
trade balance with other nations. Such an imbalance exists when the total value of a country's imports
exceeds that of exports. For example, if a country buys 10 billion dollars in products from other
countries, yet sells 10 billion dollars of its own products overseas, its trade is 20 billion dollars. Many
underdeveloped nations find themselves in this position because they lack natural resources or the
industrial capacity to use these resources, and thus have to import raw materials or manufactured
goods.
One effect of a trade deficit is the flow of currency out of a country. In the case of an
underdeveloped nation, this can cause many financial difficulties, including failure to meet debt
payments and obstacles to creation of an industrial base. Even in the case of a fully developed nation
such as the United States, a large trade deficit is reason for alarm. American products, made by well-
paid workers in U.S. industries, cost more to produce than those made in places like Asia, where labor
and material costs are much lower. Money spent on foreign products is money not spent on items
produced by domestic industries.
1. What does the passage mainly discuss?
A. The causes and consequences of trade deficits. B. Several worldwide economic problems.
C. Lack of resources in underdeveloped countries. D. The value of exports versus imports.
2. According to the passage, when does a trade imbalance occur?
A. A country sells more products overseas than its imports.
B. The value of the products a country imports is greater than the value of the products it exports.
C. A country cannot develop its natural resources.
D. A country has a serious economic problem.
3. The passage states that many underdeveloped nations have trade deficits because
A. they find themselves in this position.
B. they have failed to meet debt payments.

3|Page Tài liệu khóa học Live - VIP


OTTO CHANNEL
Otto Channel

C. they export most of their natural resources to other


D. they have to import most of their natural resources or manufactured products. nations.
4. Which of the following is not mentioned as a possible cause of a trade imbalance?
A. The high cost of exported items. B. An underdeveloped industrial base.
C. A lack of natural resources. D. Low labor and material costs in Asian countries.
5. It can be inferred from this passage that American industries
A. provide a strong industrial base that prevents a trade deficit.
B. import labor and material from overseas.
C. are hurt by a trade imbalance.
D. do not pay their workers sufficient wages.

SECTION B. WIRTTEN TEST (70PTS)


PART I. CLOZE GUIDE (10PTS)
Fill in each numbered space with one appropriate word.
HOW TELEVISION WAS INVENTED
Television owes its origins (1) many inventors. But it was the single-minded determination
of an amateur inventor, John Logie Baird, (2) led to the first live television broadcast.
Born in Scotland in 1898 and educated in Glasgow, John Logie Baird (3) a living as a razor-
blade salesman. In (4) 1890s Guglicmo Marconi showed that sound could be sent by radio waves.
Baird became convinced that a similar system could transmit a (5) . He spent most of his
spare time working on his ideas in his tiny workshop (6) any commercial support. He had to use
his own earnings to convince his research. In 1924, Baird (7) transmitted the general outline of
a figure over more than 3 metres. He continued to experiment and on October 25, 1925 transmitted a
recognizable image of a doll. He ran down to the office on the ground floor and persuaded one of the
office boys to come upstairs. The boy became the first living image (8) by television. Overnight,
Baird became famous and the money which ho needed to convince his research was at (9) made
available. In 1927 he made a transmission from London to Glasgow and in 1928 he made another from
London to New York. He continued experimenting and spent his last year exploring the (10) of
colour television.
PART II. WORD FORMATION (10PTS)
Supply the correct form of the word in the parentheses.
1. I (end) wrote letters of application, but got no reply.
2. The thing I hate about John is his (rely) .
3. Barter, however, was a very (satisfy) system.
4. Although he was not (enthusiasm) about helping with the washing up, he did so all the
time.
5. In some areas water has to be boiled to (pure) it.
6. There was a heavy (pour) yesterday afternoon which completely ruined the church
garden party.
7. The man was found guilty of fraud and sentenced to 3-year (prison) .

4|Page Tài liệu khóa học Live - VIP


OTTO CHANNEL
Otto Channel

8. Some people claim to be able (tell) to the future.


9. While walking in the mountains in North Wales, we came across a (use) quarry.
10. Thank you for your (time) delivery of the parcel.
PART III. GRAMMAR AND STRUCTURES (30PTS)
A. Use the correct form or tense of the verbs in brackets.
• I would rather (1. not be) absent from class yesterday.
• An eyewitness described how ten people (2. kill) in the fire.
• Scarcely they (3. move) into the house when their friends (4. come) in.
• Trade has gone from bad to worse and staff (5. lay) off now.
• He became addicted to (6. smoke) . He really regretted (7. not take) his father's
advise.
• It is crucial that Dido (8. stop) using Quang Ha.
• She (9. not allow) anybody (10. read) her diary but yesterday she (11. find)
her Mum (12. read) it; she couldn't but (13. ignore) the incident.
• On the first of next month, he (14. be) in prison for five years.
• I (15. look) through my old photograph album. It's full of photos of people whose names I
completely (16. forget) . I wonder what (17. happen) to them all.
• He died of lung cancer. He (18. smoke) a lot.
• The last time he (19. see) in the public, he (20. wear) a gray suit.

B. Fill in each blank with one appropriate preposition or particle.


• I subscribe several magazines.
• The world is yearning peace but does not seem to know how to win it.
• When did you check out the hotel?
• He usually writes his essays ink ruled paper.
• I could hear the director rebuking Jim being late for work again.
• Because she kept putting going to the dentist, her toothache went worse.
• Are you fed with this job?
• How are you getting your study at school?
• Passengers mustn't get off the bus when it is still motion.
• Do you know of a cure baldness?
• Our education must put an end illiteracy.
• Without a fridge, fresh food will go very quickly.
• Something went wrong my car yesterday.
• Her parents don't approve her.

5|Page Tài liệu khóa học Live - VIP


OTTO CHANNEL
Otto Channel

• The completion of the new Town Hall ha's been held owing to a strike.
• It took me a long time to get my illness.
• The meeting broke after three hours and everyone went for a drink.
• He's independent his parents.

PART IV. SENTENCE TRANSFORMATION (20PTS)


A. Rewrite the following sentences in such a way that the second sentence has the same meaning
as the first one.
1. Everyone was surprised that the singer had very little money when he died.
→ The singer .............................................................................................................
2. John has not had his hair cut for over six months.
→ The last ................................................................................................................
3. Although both his legs were broken in the crash, he managed to get out of the car before it
exploded.
→ Despite .................................................................................................................
4. She got down to writing the report as soon as she returned from her walk.
→ No sooner .............................................................................................................
5. They were all arrested because of his incompetence.
→ Had ......................................................................................................................
6. I don't really like her even though I admire her achievement.
→ Much ....................................................................................................................
7. When they told the Minister about the strike, he was very angry.
→ On ........................................................................................................................
8. The patient recovered more rapidly than expected.
→ The patient ...........................................................................................................
9. Keeping cairn is the secret of passing your driving test.
→ As long as .............................................................................................................
10. It's extremely difficult for us to make ends meet these days.
→ We find ................................................................................................................
B. Rewrite the following sentences with the given words in such a way that the second sentence
has the same meaning as the first one. Do not change the form of the words in brackets
11. I just can't decide where to go on holiday. (MIND)
→ ............................................................................................................................
12. You could always stay with us for a few days, if necessary. (PUT)
→ ............................................................................................................................
13. It won't help if you worry about it. (POINT)
→ .............................................................................................................................

6|Page Tài liệu khóa học Live - VIP


OTTO CHANNEL
Otto Channel

14. He's really keen to start his new job. (FORWARD)


→ ............................................................................................................................
15. Everyone who comes to the city notices the beauty of its architecture (FAILS)
→ ............................................................................................................................
16. She was so beautiful that I couldn't stop looking at her (EYES)
→ ............................................................................................................................
17. I've lost interest in going to the same place all the time (FED UP)
→ ............................................................................................................................
18. Lucas is just working here temporarily. (BEING)
→ ............................................................................................................................
19. Everyone is sure that he will win the election (BOUND)
→ ............................................................................................................................
20. Ann was afraid the neighbours would despise her for not having a washing machine. (LOOK)
→ ............................................................................................................................

THE END

7|Page Tài liệu khóa học Live - VIP


OTTO CHANNEL
Otto Channel
SỞ GIÁO DỤC & ĐÀO TẠO KỲ THI OLYMPIC TRUYỀN THỐNG 30/4
TP. HỒ CHÍ MINH LẦN THỨ XXV – NĂM 2002
ĐỀ CHÍNH THỨC Môn thi: Anh văn - Khối: 10
Thời gian làm bài: 180 phút

➢ Thí sinh làm phần trắc nghiệm (MULTIPLE CHOICE) trên phiếu trả lời trắc nghiệm và phần tự
luận (WRITTEN TEST) trên phiếu trả lời tự luận.
➢ Trên phiếu trả lời trắc nghiệm, thí sinh tô thêm 2 số 00 vào trước số báo danh (bằng bút chì).
Phần mã đề thi trên phiếu trắc nghiệm, thí sinh tô vào ô 002.
 --------------------------------------------------------------------------------------------------------------------

CẤU TRÚC ĐỀ THI


SECTION A. MULTIPLE CHOICE (30PTS) ................................................................................ 1
PART I. PHONOLOGY (15PTS) ..................................................................................................................... 1
PART II. VOCABULARY (5PTS) ...................................................................................................................... 1
PART III. READING COMPREHENSION (10PTS) ........................................................................................ 2

SECTION B. WRITTEN TEST (70PTS) ....................................................................................... 4


PART I. WORD FORM (35PTS) ..................................................................................................................... 4
PART II. CLOZE TEST (10PTS) ...................................................................................................................... 5
PART III. ERROR INDENTIFICATION (5PTS) ............................................................................................. 6
PART IV. SENTENCE TRANSFORMATION (20PTS) ................................................................................... 6
Otto Channel
SỞ GIÁO DỤC & ĐÀO TẠO KỲ THI OLYMPIC TRUYỀN THỐNG 30/4
TP. HỒ CHÍ MINH LẦN THỨ XXV – NĂM 2002
ĐỀ CHÍNH THỨC Môn thi: Anh văn - Khối: 10
Thời gian làm bài: 180 phút

➢ Thí sinh làm phần trắc nghiệm (MULTIPLE CHOICE) trên phiếu trả lời trắc nghiệm và phần tự
luận (WRITTEN TEST) trên phiếu trả lời tự luận.
➢ Trên phiếu trả lời trắc nghiệm, thí sinh tô thêm 2 số 00 vào trước số báo danh (bằng bút chì).
Phần mã đề thi trên phiếu trắc nghiệm, thí sinh tô vào ô 002.
 --------------------------------------------------------------------------------------------------------------------
SECTION A. MULTIPLE CHOICE (30PTS)
PART I. PHONOLOGY (15PTS)
Pick out the word whose underlined part is pronounced differently from those of the others.
1. A. preferable B. prejudice C. premises D. preconception
2. A. significant B. cellar C. muscle D. cease
3. A. wicked B. sacred C. helped D. beloved
4. A. launch B. digestion C. pronunciation D. suggestion
5. A. donkey B. monkey C. tough D. hiccough
6. A. naval B. canal C. fatal D. mammal
7. A. subtle B. climb C. rib D. lamb
8. A. complain B. bargain C. ascertain D. campaign
9. A. laugh B. plough C. enough D. cough
10. A. honorable B. honesty C. historic D. heir
Group the following words into columns according to their stress patterns.
communism lemonade tuberculosis finite ingenuity
admirable police employee although tattoo

1st syllable 2nd syllable 3rd syllable 4th syllable

PART II. VOCABULARY (5PTS)


Choose the best answer.
1. Don't worry about the lunch! I'll to it.
A. succeed B. apply C. devote D. sec
2. After leaving school, Nigel decided to in the army.
A. enlist B. enroll C. register D. sign
3. He was a generous friend, but as a businessman he drove a hard .
A. bargain B. affair C. contract D. deal
4. Everybody is very anxious about the of the negotiations.
A. outbreak B. outcome C. output D. outlook

1|Page Tài liệu khóa học Live - VIP


OTTO CHANNEL
Otto Channel
5. How did Sheila to the news of her award?
A. react B. answer C. confess D. appeal
6. Although he was completely as a furniture maker, he produced the most beautiful chairs.
A. unable B. untrained C. incapable D. uneducated
7. Some people enjoy training; for others it is only a to an end.
A. means B. method C. way D. tool
8. She her neighbor’s children for the broken window.
A. accused B. complained C. blamed D. denied
9. If you want a good flat in London, you have to pay through the for it.
A. mouth B. ear C. nose D. teeth
10. I don't think we have very much , we are very different people.
A. together B. in common C. the same D. similar

PART III. READING COMPREHENSION (10PTS)


➢ Rearrange the following paragraphs into a meaningful passage. Begin the passage with paragraph
C.
A. Today, radio and television serve many purposes besides pleasure and entertainment. They help
us to find fish in the sea and to land airplanes in bad weather or at night. They control the work
of a large number of machines in industry, and they can hold spaceships in their orbits. With the
help of radio and television, Russian scientists photographed the far side of the moon.
B. Electrons do wonderful things in calculating machines, which besides working hundreds of
thousands of times faster than any mathematician, can feel, touch, smell, hear and see. It is
difficult to imagine how we could calculate the orbits of sputniks and spaceship without using
electronic calculating machines.
C. Our century has seen great changes in the life and living conditions of people. Less than a hundred
years ago, many of the things that we do now seemed impossible. No one could imagine that
people could fly, or that we could listen to music thousands of miles away.
D. More and more often, we hear of scientists using these machines in new fields. It was with the
help of an electronic calculating machine that scientists succeeded in reading the language of the
Mayas, the ancient people of Mexico. The machines can tell us what to explain in the fields under
different climatic conditions; they can tell us when an earthquake is coming and where. The
progress of science is leading to a better life for man.
E. The little electron is the giant of our modern times. Electronic means were used during World
War I to learn of coming airplane attacks, and - many lives were saved. F. Imagine a man of
Dickens's time in our modern world. In the sky, he sees people flying in tremendous "birds", we
turn a switch and the dark room immediately becomes light. But the greatest surprise is the radio
and the television set standing in the corner of the room.
Your answer
C → → → →

2|Page Tài liệu khóa học Live - VIP


OTTO CHANNEL
Otto Channel
➢ Read the following passage and choose the correct answers.
Amy Tan, the American-born daughter of Chinese immigrants, received the Commonwealth Club Gold
Award in 1989 for her first work of fiction, the best-selling Joy Luck Club. The sixteen interrelated
stories that constitute the work alternate between the tales of four Chinese immigrant mothers and
their Americanized daughters, in an exploration of the generational and cultural tensions experienced
by first-generation daughters of immigrants.
Tan's parents, like many, immigrants, had high expectations for their children and often .set confusing
standards, expecting Amy and her two brothers to think "like Chinese but to speak perfect English,
excel academically, and take advantage of every circumstance that might lead to success. Tan,
however, rebelled against her parent's expectations, which included such exalted professions as
neurosurgery and devoted herself to being thoroughly American and dreaming of being a fiction writer.
Tan obtained a bachelor's degree in English and linguistics and a master's degree in linguistics and
eventually established herself as a highly successful business writer. Tan, however, was not satisfied
despite her material success. Turning to her life-long dream, she wrote her first short, "Endgame"; and
then a second, "Waiting between the Trees". In 1987 Tan visited her half-sisters in China with her
mother, a trip that proved to be a turning point in her life and carcer. Tan felt a sense of completeness,
a bonding with the country and its culture that she had never expected. Returning from China, Tan
was surprised to learn that on the strength of her short stories she had received an advance from the
publisher. Tan closed her business and wrote the remaining stories for the Joy Luck Club. It was a
resounding success, well received by the critics and appearing on the New York Times bestseller list.
It has been translated into 17 languages, including Chinese, and was made into a movie in 1993.
1. What is the author's main purpose in this passage?
A. To analyze Amy Tan's literary works.
B. To support Amy Tan's decision to become a fiction writer.
C. To present biographical information about Amy Tan.
D. To criticize Amy Tan's rebellion against her parents.
2. According to the passage, Amy Tan's visit to China
A. was disappointing. B. had a profound effect on her.
C. was not surprising in the least. D. was a trip she had always dreamed of taking.
3. It can be inferred from the passage that
A. Tan's parents understood her dilemma but wanted the best for her.
B. It took Tan a while to summon the courage to pursue her dreams.
C. Tan started writing fiction in order to make more money.
D. Tan had always wanted to return to China.
4. The word "exalted" means
A. highly respected B. very difficult C. common D. established
5. It can be concluded from this passage that
A. parents don't know what is best for their children.
B. Tan did not use personal experience in her writing.
C. Tan made the right decision when she closed her business.
D. Tan always knew she was Chinese foremost and American only in her imagination.

3|Page Tài liệu khóa học Live - VIP


OTTO CHANNEL
Otto Channel
SECTION B. WRITTEN TEST (70PTS)
PART I. WORD FORM (35PTS)
A. Supply the correct forms of the words in brackets.
1. He's very generous and everyone admires his (self) .
2. Librarians spend a lot of their time (class) books.
3. There is an increasing (courage) to motorcycle racing.
4. We have a bed ready in the spare room in case visitors arrive (expect)
5. There was a heavy (pour) yesterday afternoon which completely ruined the garden party.
6. The manager handed in his (resign) after being accused of dishonesty.
7. She lost the case because of her (admit) evidence.
8. He completely went to pieces after the (break) of his marriage.
9. Nicotine is responsible for the (behave) effects of smoking.
10. Although the police suspected him of the crime, since they had no definite (prove) that
he was involved, they could not arrest him.
B. Use the correct form or tense of the verbs in brackets.
• On their next anniversary, Doris and Fred (1. be) married for forty years.
• Sorry. I didn't mean (2. step) on your foot.
• His greatest ambition is (3. choose, take) part in the Olympics.
• If Linda (4. not stay) up late last night, she (5. not be) tired now.
• How about your holiday? It was all right, but I’d rather (6. go) to Da Lat.
• He always talks as though he (7. address) a public meeting.
• Hardly he (8. pick) up the book when the phone (9. ring) .
• Do you feel like (10. walk) or shall we take a taxi?
• It's essential that she (11. arrive) before 6 o'clock.
• (12. Write) the letter, she put it in an envelope.
• A man answered the phone. I suppose it was her husband.
It (13. not be) her husband. He has been dead for ages.
• He will go to Ho Chi Minh City with a view to (14. give) a chance of promotion.
• I'm likely to pass the exam, (15. be) I?
• This time tomorrow, we (16. lie) on a beach.
• I think your garden needs (17. weed) , and you'd better have it (18. do)
tomorrow.
• The news about the earthquake in the Philippines (19. already broadcast) on radio
several times so far.
C. Fill in each blank with one appropriate preposition or particle.
1. He took it granted that they were happy.

4|Page Tài liệu khóa học Live - VIP


OTTO CHANNEL
Otto Channel
2. The unpleasant smell in the restaurant put me my dinner.
3. It's no use crying spilt milk.
4. Don't let a good chance go !
5. Many factories break the anti-pollution law and get, it.
6. She resembles her father looks.
7. We always make a point trying to make our guests feel comfortable.
8. absence of evidence, the police couldn't take action the man.
9. He didn't trust me. He was suspicious my intentions.
10. Three men have been arrested and charged robbery.
11. My neighbor’s garage, which is adjacent my house is full of rubbish.
12. I watched a film on TV last night, but I can't remember how it ended, I must have dropped
before the end.
13. She plays piano beautifully. She takes her mother, who used to be a concert pianist.
14. I arranged to meet Jim after work last night, but he didn't turn .
15. Hello, operator. Could you put me to extension 301, please?
16. We all know how wonderful you are. There's no need to show .
17. Our company has just been taken by a larger one and I think quite a few people are
going to lose their jobs.
18. When they got home from their vacation, they found that burglars had broken their house.
19. Jim's very tall. He really stands from the rest of his class.

PART II. CLOZE TEST (10PTS)


Fill in each numbered space with one appropriate word.
CLASSROOMS OF THE FUTURE: INTERNET WITHOUT WIRES
By Stephen A. Booth
Computers and the Internet are giving rise to virtual universities, and the effects of technology
are being felt (1) in grade schools. Teachers now have to rethink their approach to the
subject and may need retraining (2) . Here's how computer science is changing the (3) you
and your children learn. When students go to class at California's Stanford University, they can power
up their laptop computers by (4) them into floor sockets as numerous as the beanbags that
pass for furniture. But (5) retrofitting telephone cables into classrooms is not easy or
inexpensive feat, especially in older buildings, such case of access (6) only a dream for most
schools. One plan addresses the infrastructure of schools end would use low-power radio signals to
distribute online text and video information within buildings or across an entire campus. A good (7)
of this initiative is a program called Class Link, funded by the Cellular Telecommunications
Industry Association Foundation. These wireless links (8) the students with Internet access at (9)
of up to 1.5 million bits per second, roughly equivalent to expensive fiber-optic lines but (10)
high cost.

5|Page Tài liệu khóa học Live - VIP


OTTO CHANNEL
Otto Channel
PART III. ERROR INDENTIFICATION (5PTS)
Find the ten unnecessary words in the following passage. For each answer, underline the unnecessary
word together with the word before it.
1 The handwriting of school children could be spell the difference between success and failure
2 in examinations according to research was carried out by the Open University.
3 In a study by Dennis Briggs of the Faculty of Educational Studies, it was found that essays
4 which were written in different styles of handwriting attracted to different marks. "The
5 findings suggest that there is a borderline zone within examination marking where how an
6 essay is written may be nearly almost as important as what the essay is about. ", said Mr.
7 Briggs. Five essays which were double marked with the second marker be unaware of the
8 marks of the first marker. The essay scripts for the second marker had been copied out in
9 three writing styles. Two of the styles were ones which it had been the subject of continual
10 criticism at school.
11 All the markers were practicing teachers who were told that the effectiveness of double
12 marking was being checked. The results showed that a twelve-year-old who can present an
13 essay one way will do better, perhaps far better more than a friend who can present the
14 same standard in the terms of content but who does not or cannot make it look so that
15 attractive. The conclusion is that school children may not do as well as perhaps they could
16 if only their handwriting is untidy.

Lines Mistake Correction Lines Mistake Correction


1 could be X

PART IV. SENTENCE TRANSFORMATION (20PTS)


A. Rewrite the following sentences in such a way that the second sentence has the same meaning
as the first one.
1. Unless she's really hungry, she never eats snacks between meals.
→ Only .............................................................................................................................
2. I can't believe that she was the winner.
→ I find ............................................................................................................................
3. He must pass his English examination, which is obligatory.
→ It .................................................................................................................................
4. Not until the body was found did the police believe her.
→ It was only.....................................................................................................................
5. She never seems to succeed even though she studies much.
→ Much ............................................................................................................................

6|Page Tài liệu khóa học Live - VIP


OTTO CHANNEL
Otto Channel
6. My French friend finds driving on the left difficult.
→ My French friend isn't ....................................................................................................
7. The only thing that prevented the passing of the bill was the death of the Prime Minister.
→ Had it ............................................................................................................................
8. She is proud of being such a good cook.
→ She prides ....................................................................................................................
9. Betty is very happy to look after handicapped people.
→ Betty is devoted ............................................................................................................
10. Winning the football pools meant we could buy a new car
→ Winning the football pools enabled .................................................................................
B. Rewrite the following sentences with the given words in such a way that the second sentence
has the same meaning as the first one. Do not change the form of the words in brackets.
11. The fox was unsuccessful in reaching the grapes. (VAIN)
→ .....................................................................................................................................
12. The crops were badly affected by the storm. (EFFECT)
→ .....................................................................................................................................
13. Nobody could possibly believe the story he told us. (BEYOND)
→ .....................................................................................................................................
14. The project received unanimous approval of the committee. (FAVOUR)
→ .....................................................................................................................................
15. The personnel officer promised him that she wouldn't tell anyone that he had been in prison.
(WORD)
→ .....................................................................................................................................
16. I had lost his phone number, so I could not contact him before. (TOUCH)
→ .....................................................................................................................................
17. The bank robbers escaped in a stolen car. (GETAWAY)
→ .....................................................................................................................................
18. I enjoy being the boss of a small company. (FISH)
→ .....................................................................................................................................
19. The success of our local theatre has made our city famous. (MAP)
→ .....................................................................................................................................
20. She is not upset; she is only pretending. (ACT)
→ .....................................................................................................................................

THE END

7|Page Tài liệu khóa học Live - VIP


OTTO CHANNEL
Otto Channel
SỞ GIÁO DỤC & ĐÀO TẠO KỲ THI OLYMPIC TRUYỀN THỐNG 30/4
TP. HỒ CHÍ MINH LẦN THỨ XXV – NĂM 2003
ĐỀ CHÍNH THỨC Môn thi: Anh văn - Khối: 10
Thời gian làm bài: 180 phút

➢ Thí sinh làm phần trắc nghiệm (MULTIPLE CHOICE) trên phiếu trả lời trắc nghiệm và phần tự
luận (WRITTEN TEST) trên phiếu trả lời tự luận.
➢ Trên phiếu trả lời trắc nghiệm, thí sinh tô thêm 2 số 00 vào trước số báo danh (bằng bút chì).
Phần mã đề thi trên phiếu trắc nghiệm, thí sinh tô vào ô 002.
 --------------------------------------------------------------------------------------------------------------------

CẤU TRÚC ĐỀ THI


SECTION A. MULTIPLE CHOICE (30PTS) ................................................................................................... 1
PART I. PHONOLOGY (15PTS) ..................................................................................................................... 1
PART II. VOCABULARY (10PTS) .................................................................................................................... 1
PART III. READING COMPREHENSION (5PTS) .......................................................................................... 2
SECTION B. WRITTEN TEST (70PTS) ........................................................................................................... 3
PART I. GRAMMAR AND STRUCTURES (35PTS)..................................................................................... 3
PART II. CLOZE TEST (15PTS) ...................................................................................................................... 5
PART III. ERROR INDENTIFICATION (10PTS) ........................................................................................... 5
PART IV. SETENCE TRANSFORMATION (10PTS) ...................................................................................... 6
Otto Channel
SỞ GIÁO DỤC & ĐÀO TẠO KỲ THI OLYMPIC TRUYỀN THỐNG 30/4
TP. HỒ CHÍ MINH LẦN THỨ XXV – NĂM 2003
ĐỀ CHÍNH THỨC Môn thi: Anh văn - Khối: 10
Thời gian làm bài: 180 phút

➢ Thí sinh làm phần trắc nghiệm (MULTIPLE CHOICE) trên phiếu trả lời trắc nghiệm và phần tự
luận (WRITTEN TEST) trên phiếu trả lời tự luận.
➢ Trên phiếu trả lời trắc nghiệm, thí sinh tô thêm 2 số 00 vào trước số báo danh (bằng bút chì).
Phần mã đề thi trên phiếu trắc nghiệm, thí sinh tô vào ô 002.
 --------------------------------------------------------------------------------------------------------------------
SECTION A. MULTIPLE CHOICE (30PTS)
PART I. PHONOLOGY (15PTS)
Pick out the word whose bold part is pronounced differently from those of the others.
1. A. devotion B. question C. nation D. tradition E. action
2. A. betrays B. says C. plays D. pays E. lays
3. A. measure B. treasure C. dreadful D. breathe E. breakfast
4. A. garment B. geezer C. giggle D. genuine E. getaway
5. A. measure B. pleasure C. nasal D. television E. decision
6. A. naked B. sacred C. kicked D. wicked E. beloved
7. A. carriage B. voyage C. message D. massage E. dosage
8. A. south B. booth C. myth D. truth E. southern
9. A. extraordinary B. extreme C. exemplify D. expertise E. exaggeration
10. A. absorb B. debt C. comb D. bomb E. tomb
Group the following words into columns according to their stress patterns.
personnel marmalade ingenuity psychological patriotism
geneticist coincide experimental prerequisite infamous

1st syllable 2nd syllable 3rd syllable

PART II. VOCABULARY (10PTS)


A. Choose the best answer
1. This ring is only made of plastic, so it is quite .
A. valuable B. invaluable C. priceless D. valueless
2. Robert is completely in his new book on photography.
A. absorbed B. interested C. exhausted D. occupied
3. It is difficult for museums to find funds to protect the nation's .
A. inheritance B. heritage C. possessions D. legacy

1|Page Tài liệu khóa học Live - VIP


OTTO CHANNEL
Otto Channel
4. There is a constant of visitors to this important historic site.
A. current B. tide C. wave D. stream
5. You shouldn't have criticized him in front of his friends. It was extremely of you.
A. unfortunate B. insensitive C. insensible D. unconscious
6. The of living goes up and up. It'll never go down.
A. price B. value C. expense D. cost
7. Even after you move to New York, we'd really appreciate it if you would take the time to
with us.
A. keep in touch B. keep in mind C. drop a line D. stop over
8. To be a good short story writer one needs, among other things, a very imagination.
A. vivid B. living C. bright D. sparkling
9. We've got a very teacher who doesn't mind if we are a bit late.
A. broad-minded B. open-minded C. easy-going D. absent-minded
10. Stop about the bush, James! Just tell me exactly what the problem is.
A. rushing B. hiding C. beating D. moving
B. Supply the correct forms of the words in brackets.
11. We can't depend on him; he's a very (rely) person.
12. I have no excuses. My actions were (explain) .
13. Women who are slimming can never enjoy a meal without being afraid of (organize) their
diet.
14. The cat looked at the new type of food on its plate (suspect) .
15. He suffered from constant (sleep) .
16. The doctor gave him an injection to (death) the pain.
17. The people whose houses are to be demolished when the new road is built will be (house)
.
18. Although the splitting of the atom was one of the greatest scientific (break) of
this century, there are many people who don't welcome it.
19. Many parents today have their infants (vaccine) against hepatitis B.
20. My sister is very (know) about gardening but I'm afraid I don't know the first thing about it.
PART III. READING COMPREHENSION (5PTS)
Read the following passage and choose the correct answers.
Successful innovations have driven many older technologies to extinction and have resulted in
higher productivity, greater consumption of energy, increased demand for raw materials, accelerated
flow of material through the economy and increased quantities of metals and other substances in use
per capita. The history of industrial development abounds with examples.

2|Page Tài liệu khóa học Live - VIP


OTTO CHANNEL
Otto Channel
In 1870, horses and mules were the prime sources of power on US farms. One horse or mule
was required to support for human-beings, a ratio that remained almost constant for many decades.
At that time, had a national commission been asked to forecast the horse and mule population for
1970, its answer probably would have depended on whether its consultants were of an economic or
technological turn of mind. Had they been "economists", they would probably have projected the 1970
horses and mule population to be more than 50 million. Had they been "technologists" they would
have recognized that the power of steam had already been harnessed to industry and to land ocean
transport. They would have recognized further that it would be only a matter of time before steam
would be the prime source of power on the farm. It would have been difficult for them to avoid the
conclusion that the horse and mule population would decline rapidly.
1. According to the passage, what supplied most of the power on US farms in 1870?
A. Humans B. Animals C. Engines D. Water
2. Which of the following is not mentioned by the author as a consequence of new technological
developments?
A. More raw materials become necessary B. Overall productivity increased.
C. Older technologies die away D. The quality of life is improved.
3. It can be inferred from the passage that by 1870
A. technology began to be more economical.
B. the US horse population was about 10 million.
C. the steam engine had been invented.
D. a national commission on agriculture had been established.
4. In the second paragraph, the author suggests that “economists" would
A. fail to consider the influence of technological innovation.
B. value the economic contribution of farm animals.
C. plan the economy through yearly forecasts.
D. consult for the national commission on the economy.
5. What is the author's attitude towards changes brought on by technological innovation?
A. He is excited about them. C. He accepts them as natural.
B. He is disturbed by them. D. He questions their usefulness.

SECTION B. WRITTEN TEST (70PTS)


PART I. GRAMMAR AND STRUCTURES (35PTS)
A. Use the correct form or tense of the verbs in brackets.
Canning is a method of preserving food over extended periods of time. The process involves (1.
seal) food in containers and (2. heat) it in order to kill bacteria that could eventually
cause spoilage. While most canned food is produced by commercial companies, some (3. do)
at home.
Homegrown fruits and vegetables such as apples and tomatoes are the most popular foods (4.
can) . Even certain kinds of meats are suitable for canning. However, it is not advisable to heat
produce such as avocados because of the changes in taste and texture that (5. occur)

3|Page Tài liệu khóa học Live - VIP


OTTO CHANNEL
Otto Channel
Other foods, including cucumbers and peppers, can (6. can) only if they (7. first pickle)
and then cooked at very low temperatures.
Properly canned food can be saved for as long as three years. If (8. not seal/properly)
, the food can be spoiled by the growth of organisms. In severe cases, bacteria can cause a
fatal form of poisoning (9. call) botulism. For this reason, it is very important to check the seal of
the jar or bottle regularly to make sure it (10. remain) undisturbed.
B. Use the correct form or tense of the verbs in brackets.
• Were she (1. know) the truth, she would tell you about it.
• My boss is very angry with me. I didn't do all the work that I (2. do) last week.
• It looks as if this light (3. burn) all night. I (4. forget) (5. switch) it off
before I went to bed last night.
• I visit new countries every year. By the time I (6. be) sixty, I (7. visit) all the most
interesting countries in the world.
• Congress has decreed that gasoline tax (8. abolish) .
• I didn't see anyone but I felt as though I (9. watch) .
• If I'd taken his advice, I (10. still, not do) the same job.
C. Fill in each blank with one appropriate preposition.
• She's always (1) the go. I don't know where she gets her energy from.
• Samples will be provided (2) request.
• We have to go by car (3) account (4) the bus strike.
• This pullover is a bargain (5) $20.50.
• The case was dismissed (6) the absence (7) any definite proof.
• It is (8) the law to leave your car here.
• They can only cure him (9) his illness if they operate on him.
• His name is Nicholas, but we call him Nick (10) short.
D. Complete the following sentences using the verbs given in the box. You have to use the correct
tenses of the verbs and the appropriate particles.
give go let call fall
come make break wear bring

1. We felt we'd been badly when they told us our application had been rejected.
2. If you have an old blanket, it's advisable that it so that we have something to sit on at
the beach.
3. The gun in his hand when he was cleaning it.
4. He pretended to be an Englishman, but his foreign language him .

4|Page Tài liệu khóa học Live - VIP


OTTO CHANNEL
Otto Channel
5. We've made all the arrangements. Let's hope our plans don't at the last moment.
6. He a lot of money at a very early age, so he has never had to work.
7. To cover his absence, he decided a complete false story about being involved in a car
accident.
8. We a specialist when he grew worse.
9. The school for the holiday at the end of May.
10. Looking after six kids all day has completely me
PART II. CLOZE TEST (15PTS)
Fill in each numbered space with one appropriate word.
When we were in England last year, I (1) fishing with my friend, Peter. Early in the
morning we were sitting quietly by the (2) of a lake when we had an unpleasant surprise. We
saw a duck (3) with three ducklings (4) along cheerfully behind her. As we watched
them, (5) was a sudden swirl in the water. We caught a (6) of the vicious jaws of a pike - a
fish (7) is rather like a freshwater shark and one of the (8) was dragged below the surface.
This incident made Peter furious. He vowed catch the pike (9) three successive mornings, we
returned to the vicinity and used different (10) of bait. On the third day, Peter was lucky (11)
an artificial frog as bait, he (12) to hook the monster. There was a desperate fight but
Peter was determined to capture the pike (13) he succeeded. When he had got it ashore and
killed it, he (14) the fish and found that it scaled (15) thirty pounds a record for that district.
PART III. ERROR INDENTIFICATION (10PTS)
There are fifteen mistakes in the following passage. Find and correct them.
DREAMS
1 Dreams have usually fascinated human beings. The idea of dreams provides us with useful
2 informations about our life goes back thousands of years. For the greater part of human
3 history which was taken for granted that the sleeping mind was in touch with the super
4 natural world and dreams were to be interpreted as messages with prophetic or healing
5 functions. In the nineteen century, it was a widespread reaction against this way of thinking
6 and dreaming were widely dismissed as being very little more than jumbles fantasy about
7 memories of the next day.
8 It was not at the end of the nineteenth century that an Austrian neurologist, Sigmund Freud,
9 pointed out that people who has similar experiences on the day, and who are then subjected
10 by the same stimuli when they are asleep, produces different dreams. Freud went on to
11 develop the theory of the dream process which might enable him to interpret dreams as clue
12 to the conflicts taking place within the personality. It is by no means an exaggeration to say
13 that hardly any other theories have had such great an influence on subsequent thought.

5|Page Tài liệu khóa học Live - VIP


OTTO CHANNEL
Otto Channel
Lines Mistake Correction Lines Mistake Correction
1 usually always

PART IV. SETENCE TRANSFORMATION (10PTS)


A. Rewrite the following sentences in such a way that the second sentence has the same meaning as
the first one.
1. "That's a lovely new dress, Jean," said her mother.
→ Jean's mother complimented ...........................................................................................
2. Under no circumstances should you phone the police.
→ The last ..........................................................................................................................
3. I wasn't a bit surprised to hear that Karen had changed her job.
→ It came ..........................................................................................................................
4. The cost of living has gone up considerably in the last few years.
→ There .............................................................................................................................
5. I'm absolutely sure he took the money on purpose.
→ He couldn't possibly ........................................................................................................
6. He had no idea how difficult the task would be until he was half-way through it.
→ Only ...............................................................................................................................
7. They had such a fierce dog that nobody would visit them.
→ .......................................................................................................................................
8. His disabilities didn't prevent him from sailing around the world.
→ Although .........................................................................................................................
9. The presidential visit attracted such an enormous crowd that all traffic came to a standstill.
→ So many .........................................................................................................................
10. You can use it as long as you like, and it won't wear out.
→ No matter .......................................................................................................................
B. Rewrite the following sentences with the given words in such a way that the second sentence has
the same meaning as the first one. Do not change the form of the words in brackets.
11. Will you please stay with me for a while? (COMPANY)
→ .......................................................................................................................................
12. You shouldn't take his help for granted. (ASSUME)
→ .......................................................................................................................................
13. Jenny didn't feel like going to the party. (MOOD)
→ .......................................................................................................................................

6|Page Tài liệu khóa học Live - VIP


OTTO CHANNEL
Otto Channel
14. We delayed our departure because of the bad weather. (ACCOUNT)
→ .......................................................................................................................................
15. He is certainly not a reliable witness. (MEANS)
→ .......................................................................................................................................
16. Scientists say forests are being destroyed by air pollution. (BLAME)
→ .......................................................................................................................................
17. There's no way we can agree to this solution. (QUESTION)
→ .......................................................................................................................................
18. I can nearly remember his name. (TONGUE)
→ .......................................................................................................................................
19. I William tried to remain impartial in the quarrel between his two cousins. (SIDES)
→ .......................................................................................................................................
20. Final year students don't have to attend lectures. (OPTIONAL)
→ .......................................................................................................................................

THE END

7|Page Tài liệu khóa học Live - VIP


OTTO CHANNEL
Otto Channel
SỞ GIÁO DỤC & ĐÀO TẠO KỲ THI OLYMPIC TRUYỀN THỐNG 30/4
TP. HỒ CHÍ MINH LẦN THỨ XXV – NĂM 2004
ĐỀ CHÍNH THỨC Môn thi: Anh văn - Khối: 10
Thời gian làm bài: 180 phút

➢ Thí sinh làm phần trắc nghiệm (MULTIPLE CHOICE) trên phiếu trả lời trắc nghiệm và phần tự
luận (WRITTEN TEST) trên phiếu trả lời tự luận.
➢ Trên phiếu trả lời trắc nghiệm, thí sinh tô thêm 2 số 00 vào trước số báo danh (bằng bút chì).
Phần mã đề thi trên phiếu trắc nghiệm, thí sinh tô vào ô 002.
 --------------------------------------------------------------------------------------------------------------------

CẤU TRÚC ĐỀ THI


SECTION A. MULTIPLE CHOICE (35PTS)...................................................................................................... 1
PART I. PHONOLOGY (15PTS)........................................................................................................................ 1
PART II. VOCABULARY (10PTS) ...................................................................................................................... 1
PART III. READING COMPREHENSION (10PTS) .......................................................................................... 2

SECTION B. WRITTEN TEST (65PTS) ............................................................................................................. 3


PART IV. GRAMMAR AND STRUCTURES (25PTS) ....................................................................................... 3
PART V. CLOZE TEST (10PTS) ......................................................................................................................... 5
PART VI. ERROR INDENTIFICATION (10PTS) ............................................................................................. 5
PART VII. SETENCE TRANSFORMATION (20PTS) ........................................................................................ 6
Otto Channel
SỞ GIÁO DỤC & ĐÀO TẠO KỲ THI OLYMPIC TRUYỀN THỐNG 30/4
TP. HỒ CHÍ MINH LẦN THỨ XXV – NĂM 2004
ĐỀ CHÍNH THỨC Môn thi: Anh văn - Khối: 10
Thời gian làm bài: 180 phút

➢ Thí sinh làm phần trắc nghiệm (MULTIPLE CHOICE) trên phiếu trả lời trắc nghiệm và phần tự
luận (WRITTEN TEST) trên phiếu trả lời tự luận.
➢ Trên phiếu trả lời trắc nghiệm, thí sinh tô thêm 2 số 00 vào trước số báo danh (bằng bút chì).
Phần mã đề thi trên phiếu trắc nghiệm, thí sinh tô vào ô 002.
 --------------------------------------------------------------------------------------------------------------------
SECTION A. MULTIPLE CHOICE (35PTS)
PART I. PHONOLOGY (15PTS)
A. Pick out the word whose bold part is pronounced differently from those of the others.
1. A. liquor B. choke C. chord D. chaos
2. A. raised B. hatred C. sacred D. naked
3. A. food B. look C. took D. good
4. A. releases B. faces C. places D. advises
5. A. nature B. change C. gravity D. basis
6. A. exist B. exhibit C. exhibition D. exactly
7. A. enough B. cough C. though D. rough
8. A. geisha B. giraffe C. gin D. ginger
9. A. resign B. resound C. resonant D. resource
10. A. anchor B. uncle C. thankful D. anything
B. Group the following words into columns according to their stress patterns.
geologist ordinary contributory ingenuity understand
competent tuberculosis naive necessary guarantee

1st syllable 2nd syllable 3rd syllable 4th syllable

PART II. VOCABULARY (10PTS)


Choose the best answer.
1. Enough money has been raised to the hospital's survival.
A. ensure B. endow C. enable D. empower
2. , the people who come to this club are in their twenties and thirties.
A. by and large B. Altogether C. To a degree D. Virtually
3. Physics and mathematics interest him almost .
A. equally B. the same C. emptiness D. limit
4. I admit I suffer from a of patience with such people.
A. shortage B. lack C. similarly D. alike

1|Page Tài liệu khóa học Live - VIP


OTTO CHANNEL
Otto Channel
5. You haven't heard all the facts so don't to conclusions.
A. dash B. jump C. spring D. fly
6. The student's hard work was with success in his degree examination.
A. rewarded B. thanked C. awarded D. presented
7. He on the bed staring at the ceilings wondering what to do next.
A. laid B. lay C. lain D. lied
8. The between the rich and the poor countries of the world is increasing.
A. space B. gap C. distance D. interval
9. Picasso was a (n) cubist painter.
A. artistic B. celebrated C. colorful D. knowledgeable
10. Because of the dominance of retail chain-stores, most shopping centres show the same bland
and no imagination.
A. similarity B. likeness C. equality D. uniformity
PART III. READING COMPREHENSION (10PTS)
PASSAGE 1
Read the article below about the London Underground (the Tube). Choose the best phrases from A to
J to fill space 1-5. One answer has been given as an example.
UP THE TUBE
Instead of complaining about the Northern Line, we Londoners ought to feel grateful for the tube,
the envy of other cities. Liverpool and Newcastle have local lines of a faintly tube-like appearance but
they go underground for very short distances (0) J .
Thirty years ago, my friend Pearl from Peacchaven came up to London (11) . She had to
be coaxed on the escalator, and gave little shrieks of delighted horror (12) . Inside the train she
looked out of the window at blackness in alarm. Like all who gaze through underground windows, she
saw not only blackness (13) .
When the escalator first began, my family tells me, (14) hence the unmovable
steps that sometimes run parallel to the moving staircase. My aunt, straight from a village in Poland,
was struck with terror at the sight of the escalator, (15) . Those were chivalrous times-and two
London Transport officials heaved her on and helped on to her as she screamed, kicked and struggled.
I bet they were glad when they reached the top for she was a large aunt.
A. and refused point-blank to ascend
B. as if she were on a fairground roller-coaster
C. as there's nothing quite like it
D. but her own reflection
E. many adults were frightened of it
F. but because she was scared
G. and I showed her the sights
H. even though she had made up her mind
I. so I don't think they qualify as tubes

2|Page Tài liệu khóa học Live - VIP


OTTO CHANNEL
Otto Channel
PASSAGE 2
Read the following passage and choose the correct answers.
Noise, commonly defined as unwanted sound, is another environmental pollutant. Particularly in
congested urban areas, the noise produced as a by- product of our advancing technology causes
physical and psychological harm, and detracts from the quality of life for those who are exposed to it.
Unlike the eye, the ear has no lid; therefore, noise penetrates without protection. Loud noises
instinctively signal danger to any organism with a hearing mechanism, including human beings. In
response, heartbeat and respiration accelerate. In fact, there is a general increase in functioning
caused by the flow of adrenalin in response to fear.
Because noise is unavoidable in a complex, industrial society, we are constantly responding in
the same way that we would response to danger. Recently, researchers have concluded that noise and
our response may be much more than an annoyance. It may be a serious threat to physical and
psychological health and well-being, causing damage not only to the ear and brain but also to the heart
and stomach. We have known that hearing loss is America's number one nonfatal health problem, but
now we are learning that some of us with heart disease and ulcers may be victims of noise as well.
16. What is the author's main point?
A. Loud noises signal danger.
B. The ear is not like the eye.
C. Hearing loss is America's number one nonfatal health problem.
D. Noise may pose a serious threat to our physical and psychological health.
17. What is the author's definition of noise?
A. A by-product of technology. C. Unwanted sound.
B. Physical and psychological harm. D. Congestion
18. According to the passage, people respond to loud noises in the same way that they respond to
A. annoyance B. disease C. danger D. damage
19. It can be inferred from this passage that the eye
A. enjoys greater protection than the ear C. is damaged by noise.
B. responds to fear D. increases functions.
20. What is probably the topic of the paragraph that precedes this passage?
A. Urban areas B. Technology C. Environmental pollutants D. Disease

SECTION B. WRITTEN TEST (65PTS)


PART IV. GRAMMAR AND STRUCTURES (25PTS)
A. Use the correct form or tense of the verbs in brackets.
It (1. once, believe) that being overweight (2. be) healthy, but nowadays few
people (3. subscribe) to this viewpoint. While many people (4. fight) the battle (5.
reduce) weight, studies are being conducted (6. concern) the appetite and how it (7.
control) by both emotional and biochemical factors. Some of the conclusion of these studies may
give insight into how (8. deal) with weight problems. For example, when (9. ask) about
their eating habits in times of stress, several hundred people said they reacted to stressful situations

3|Page Tài liệu khóa học Live - VIP


OTTO CHANNEL
Otto Channel
by eating. Further investigations with both human and animals indicated that it (10. be, not)
food that relieves tension but rather the act of chewing.
B. Use the correct form or tense of the verbs in brackets.
1. Mary isn't here but she (not, take) the car because there's no petrol in it.
2. Hardly they (move) into the house when their friends came.
3. Trade has gone from bad to worse and staff (lay) off now.
4. Would they not rather you (seek) new sponsors?
5. By the time we get there now, I'm afraid the meeting (end) .
6. Jean, I'm so glad you've got here at last. I (expect) you all day.
7. None of the people (invite) to the party can come.
8. His demand was that his son (acquit) at once.
9. Had they not invented satellites, we (not, watch) live programmes on TV.
10. What a pity! If only I (listen) to my parents.
C. Supply the correct forms of the words in brackets.
1. The (electrify) of the agriculture in our country is very necessary.
2. She is training to be a (beauty) .
3. It's (character) of him to lose his temper like that he's usually very calm.
4. Politeness is one thing. Real kindness is another. You must learn to (different)
between the two.
5. It is (material) to me whether she stays or leaves.
6. I didn't believe him. His story was very (convince) .
7. We (vary) go to the pub before lunch on Sunday.
8. His disappearance is very strange, in fact, quite (explain) .
9. The new (pass) has certainly made things a lot easier in the town centre since it has
taken away all the through traffic.
10. The low wages and poor working conditions caused great (satisfy) among the workers.
D. Fill in each blank with one appropriate preposition.
1. He believes the figure should be more $10 million.
2. They were angry at being treated children.
3. I promise to admit my office.
4. These oranges are sold the dozen.
5. We met each other in Paris coincidence.
6. Our house has been the market for months.
7. I called the hope finding her at home.
8. I was the verge of phoning him when he came.

4|Page Tài liệu khóa học Live - VIP


OTTO CHANNEL
Otto Channel
9. Can you help me solve this puzzle?
10. Don't rely on me, I think we are the same boat.
E. Complete the following sentences using the verbs given in the box. You have to use the correct
tenses of the verbs and the appropriate particles.
set bring put come make
carry go get run
1. I unexpectedly an old friend in the shopping center last week.
2. He a new record for the 1,000 metres in the last Olympic Games.
3. Police have asked witness of the accident .
4. I couldn't what he was saying. It was beyond my understanding.
5. I'm sure this milk . It smells funny.
6. You may a brave front but you are really nervous.
7. Do you think I have any chance of my driving test?
8. Computer technology will a revolution in business administration.
9. He sitting for the test when he was told how difficult it would be.
10. The Vietnamese people successfully the August Revolution in 1945.

PART V. CLOZE TEST (10PTS)


Fill in each numbered space with one appropriate word.
Low level of literacy and numeracy have a damaging impact (1) almost very aspect
of adult (2) according to a survey published yesterday, which offers evidence (3) a developing
underclass.
Tests and interviews with hundreds of people born in a (4) week in 1958 graphically
illustrated the handicap of educational underachievement. The effects (5) seen in unemployment,
family breakdown, (6) incomes, depression and social inactivity.
Those who left school at 16 with (7) basic skills had been employed for (8) to four years
less than good readers by the time they reached 37. Professor John Bynner, of City University, who
carried (9) the research, said that today's unqualified teenagers would face even greater problems
because the supply of manual jobs had (10) up.
PART VI. ERROR INDENTIFICATION (10PTS)
There are ten mistakes in the following passage. Find and correct them.
1 Petroleum was formed on the ground many millions of years ago. Plants and animals live in
2 the sea sank to the sea bed when they died. Their remain mixed with sand and mud in layer
3 deposits, built up slowly over the years. A combined action of heat, pressure and bacteria
4 turns these deposits into petroleum. The petroleum moved slowly upwards to the porous
5 rocks nor became trapped there. In the passage of time, land that was once under water
6 rose to the face of the sea. As such, petroleum deposits are now found below or near dry
7 land.

5|Page Tài liệu khóa học Live - VIP


OTTO CHANNEL
Otto Channel

Lines Mistake Correction Lines Mistake Correction

PART VII. SETENCE TRANSFORMATION (20PTS)


A. Rewrite the following sentences in such a way that the second sentence has the same meaning
as the first one.
1. I only realized what I had missed when they told me about it later.
→ Not until ........................................................................................................................
2. My parents let me go abroad alone for the first time.
→ I was ............................................................................................................................
3. You're asking questions because you didn't pay enough attention.
→ Had it ...........................................................................................................................
4. It's sad, but unemployment is unlikely to go down this year.
→ Sad ..............................................................................................................................
5. She asked us to leave quietly so that we wouldn't disturb her.
→ So as ............................................................................................................................
B. Rewrite the following sentences with the given words in such a way that the second sentence
has the same meaning as the first one. Do not change the form of the words in brackets.
6. I think you should try and be as optimistic as you can. (SIDE)
→ .....................................................................................................................................
7. Local residents said they were against the new traffic scheme. (DISAPPROVAL)
→ .....................................................................................................................................
8. Fish for breakfast doesn't appeal to everyone. (TASTE)
→ .....................................................................................................................................
9. We agreed that each of us would do the washing up on alternative days. (TURNS)
→ .....................................................................................................................................
10. Jack and Rose never agree on how to bring up their children. (EYE)
→ .....................................................................................................................................

THE END

6|Page Tài liệu khóa học Live - VIP


OTTO CHANNEL
Otto Channel
SỞ GIÁO DỤC & ĐÀO TẠO KỲ THI OLYMPIC TRUYỀN THỐNG 30/4
TP. HỒ CHÍ MINH LẦN THỨ XXV – NĂM 2007
ĐỀ CHÍNH THỨC Môn thi: Anh văn - Khối: 10
Thời gian làm bài: 180 phút

➢ Thí sinh làm phần trắc nghiệm (MULTIPLE CHOICE) trên phiếu trả lời trắc nghiệm và phần tự
luận (WRITTEN TEST) trên phiếu trả lời tự luận.
➢ Trên phiếu trả lời trắc nghiệm, thí sinh tô thêm 2 số 00 vào trước số báo danh (bằng bút chì).
Phần mã đề thi trên phiếu trắc nghiệm, thí sinh tô vào ô 002.
 --------------------------------------------------------------------------------------------------------------------

CẤU TRÚC ĐỀ THI


SECTION A. MULTIPLE CHOICE (35PTS) ......................................................................... 1
PART I. PHONOLOGY (5PTS) ..................................................................................................................... 1
PART II. VOCABULARY AND GRAMMAR (15PTS) .................................................................................. 1
PART III. READING COMPREHENSION (15PTS) ..................................................................................... 3

SECTION B. WRITTEN TEST (65PTS) ............................................................................... 4


PART I. CLOZE TEST (20PTS) ................................................................................................................... 4
PART II. READING TEST (7PTS) ................................................................................................................ 5
PART III. WORD FORMATION (18PTS) ..................................................................................................... 7
PART IV. SETENCE TRANSFORMATION (20PTS) ................................................................................... 8
Otto Channel
SỞ GIÁO DỤC & ĐÀO TẠO KỲ THI OLYMPIC TRUYỀN THỐNG 30/4
TP. HỒ CHÍ MINH LẦN THỨ XXV – NĂM 2007
ĐỀ CHÍNH THỨC Môn thi: Anh văn - Khối: 10
Thời gian làm bài: 180 phút

➢ Thí sinh làm phần trắc nghiệm (MULTIPLE CHOICE) trên phiếu trả lời trắc nghiệm và phần tự
luận (WRITTEN TEST) trên phiếu trả lời tự luận.
➢ Trên phiếu trả lời trắc nghiệm, thí sinh tô thêm 2 số 00 vào trước số báo danh (bằng bút chì).
Phần mã đề thi trên phiếu trắc nghiệm, thí sinh tô vào ô 002.
 --------------------------------------------------------------------------------------------------------------------
SECTION A. MULTIPLE CHOICE (35PTS)
PART I. PHONOLOGY (5PTS)
Find the word that has its underlined part pronounced differently from the other three in each
question.
1. A. massage B. carriage C. voyage D. dosage
2. A. dimension B. expansion C. confusion D. tension
3. A. increase B. ink C. pink D. thank
4. A. apology B. classify C. testify D. verify
5. A. beloved B. naked C. ploughed D. learned
Find the word with the stress pattern different from that of the other three words in each question.
6. A. obvious B. notorious C. credulous D. numerous
7. A. dialect B. diagram C. diagonal D. diamond
8. A. Europe B. monument C. province D. minority
9. A. obsolete B. complete C. compete D. deplete
10. A. consent B. obstinacy C. condolence D. equality
PART II. VOCABULARY AND GRAMMAR (15PTS)
A. Choose the best answer
11. She loved tennis and could watch it till the came home.
A. she B. everyone C. horses D. cows
12. Could you close the window? There is a bit of a .
A. current B. wind C. draught D. breeze
13. Thousands of steel were used as the framework of the new office block.
A. beams B. girders C. stakes D. piles
14. The ceiling fans were on, but unfortunately they only the hot, humid air.
A. stirred up B. poured through C. turned into D. cut back
15. He set one alarm-clock for five o'clock and the other for five past so as to that he did
not oversleep.
A. nut B. ensure C. insure D. reassure
16. When Tim was eating a cherry, he accidentally swallowed the
A. assure B. stone C. seed D. core

1|Page Tài liệu khóa học Live - VIP


OTTO CHANNEL
Otto Channel
17. It was only he told me his surname that I realized that we had been to the same school.
A. then B. until C. as soon as D. when
18. He got an excellent grade in his examination the fact that he had not worked
particularly hard.
A. on account of B. because C. in spite of D. although
19. Their eventual choice of house was by the time Peter would take to get to the office.
A. related B. consequent C. determined D. dependent
20. It turned out that we rushed to the airport as the plane was delayed by several
hours.
A. hadn't B. should have C. mustn't D. needn't have
B. From the four underlined words or phrases A, B, C or D identify the one that is not correct.
21. Anthropologists agree that our primitive ancestors who inhabited the tropics probably have
A B C
natural protection against the sun.
D
22. A good exercise program helps teach people to avoid the habits that might shorten the lives.
A B C D
23. A thunder usually follows lightning by five seconds for every mile between the flash and the
A B C D
observer.
24. Forgery, in law, is the fabrication or altering of a written document with the intent to deceive
A B C
or defraud.
D
25. During the first half of the nineteenth century, immigrants to the United States were
A
predominant from Western Europe; after the Civil War, however, new arrivals came mainly
B
from Eastern and Southern Europe, as well as from Asia.
C D
26. Bill Gates built his microcomputer software company into one of the Largest in the nation, and
A B
in doing so became one of the country's wealthiest and most respected man.
C D
27. With his many theories, Albert Einstein did a great impact on physics, so much so that he is
A B
often called the greatest physicist of all time.
C D
28. Since rats are destructive and may carry disease, therefore many cities try to exterminate
A B C
them
D

2|Page Tài liệu khóa học Live - VIP


OTTO CHANNEL
Otto Channel
29. In that age of computers, it is difficult to imagine how tedious the work of accountants sand
A B C
clerks must have been in the past
D
30. Made up of more than 150 member countries, the organization known as the United Nations
A B
were established after World War II to preserve international peace and security.
C D
PART III. READING COMPREHENSION (15PTS)
A. Read the passage and then decide which word (A, B, C, or D) best fits each space.
WHALING
Rock carving suggest that Stone Age people were hunting whales for food as early as 2200 B.C.
Such (31) hunting is still practiced today in a number of (32) including the Inuit people of
Greenland and North America.
Whaling became big business from the seventh century as the (33) for whalebone and whale
oil rose, and humpback and sperm whales were hunted in (34) large numbers. But just as
stocks of these species began to fall, the explosive harpoon-gun was (35) . This weapon,
together with the development of steam-power ships, (36) the whalers to hunt the fast-moving fin
and blue whales.
In 1905 the whaling (37) moved to the waters of Antarctica. The introduction of massive
factory ships enabled the whales to be processed at sea. As a result, the blue whale had (38)
disappeared by the 1950s. In 1946 the International Whaling Commission was established to
maintain the declining whale populations. Quotas were (39) but these were often (40)
and numbers continued to fall. Hunting of many species continued until 1986 when the IWC
finally responded to international pressure and a ban on commercial whaling was introduced.
31. A. survival B. essential C. basic D. subsistence
32. A. groups B. societies C. races D. nationalities
33. A. demand B. desire C. request D. reliance
34. A. repeatedly B. frequently C. continually D. increasingly
35. A. invented B. discovered C. assembled D. applied
36. A. managed B. employed C. enabled D. empowered
37. A. lines B. troops C. staff D. fleets
38. A. virtually B. possibly C. uniquely D. commonly
39. A. made B. set C. placed D. done
40. A. refused B. denied C. ignored D. exempted
B. Read the passage and choose the correct answer for the following questions.
All at once Hazel was coming in through the French windows, pulling off gardening gloves, and
Bill was entering through the door, both at once. So I only had time to take one quick look at her
before I turned to face him. All very confusing. What that first glimpse showed me was that time had
thickened her figure but didn't seem to have made much difference to her face. It still had good skin
3|Page Tài liệu khóa học Live - VIP
OTTO CHANNEL
Otto Channel
and youthful outlines. She was holding a bunch of roses - been cutting them in the garden while
waiting for me. The gardening gloves lent a delightfully informal touch. It was quite an entrance,
though Bill spoilt it a bit by making his at the same time.
Bill seemed longer and thinner. His tightly massed hair had a tinge of grey Apart from that, twenty
years had done nothing to him, except deepen the lines of thoughtfulness that had already, when I
knew him, begun to spread across his face. Or was that all? I looked at him again, more carefully, as
he looked away from me at Hazel. Weren't his eyes different somehow? More inward looking than
ever? Gazing in not merely at his thoughts, but at something else, something he was keeping hidden
or perhaps protecting.
Then we were chattering and taking glasses in our hands, and I came back to earth. For the first
ten minutes we were all so defensive, so carefully probing, that nobody learnt anything. Bill had
forgotten me altogether, that much was clear. He was engaged in getting to know me from scratch,
very cautiously so as not to hit a wrong note, with the object of getting me to contribute a big
subscription to his African project. I kept trying to absorb details about Hazel, but Bill was talking
earnestly about African education, and the strain of appearing to concentrate while actually thinking
about his wife proved so great that I decided it would be easier just to concentrate. So I did. I let him
hammer away for about ten more minutes, and then the daughter, who seemed to be acting as
parlourmaid, showed in another visitor. Evidently, we were to be four at lunch.
41. What effect had time had on Hazel and Bill?
A. They had both lost weight. B. They were more withdrawn.
C. They hadn't changed at all. D. They had changed in subtle ways.
42. When they all started talking, the writer
A. relaxed at last. B. stopped dreaming.
C. spoke most to Hazel. D. began to remember things.
43. The writer found the first part of their conversation
A. sentimental B. irritating C. uninformative D. trivial
44. Why did Bill speak seriously?
A. He wanted some money from the writer. B. He did not remember the writer.
C. His wife was present. D. He was talking about the
45. In the end the writer found Bill's conversation past.
A. monotonous B. convincing C. thought-provoking D. instructive
SECTION B. WRITTEN TEST (65PTS)
PART I. CLOZE TEST (20PTS)
Read the text below and think of the word which best fits each space. Use only one word in each
space.
Although the rise in the global temperature by 4 per cent predicted by many scientists may not
sound like much, it is the difference between now and the last Ice Age, when huge glaciers covered
Europe and most of Britain. Nobody knows (1) what would happen in a warmer world, but we

4|Page Tài liệu khóa học Live - VIP


OTTO CHANNEL
Otto Channel
(2) know some things. Heat a kettle and the (3) inside it expands. The (4) of the
world has climbed more than half a degree this century, and the oceans have (5) by at least 10 cm.
But (6) as it takes several minutes for a kettle to begin warming, (7) it may have
taken the ocean thirty years to swell. This (8) that the global warming we are now experiencing is a
result only of the carbon dioxide we have dumped into the atmosphere (9) to the 1960s. Since
then, the (10) of fossil fuels has increased rapidly.
Scientists (11) for the United nations and European governments have been warning that
(12) the Dutch and the people of East Anglia will need to do will (13) to build more extensive
sea defenses. Many of the world's great cities are (14) risk, because they are (15) at
sea level. Miami, (16) entirely built on a sandbank, could be (17) away. But the effect of
rising sea levels will be much (18) for the developing countries. With a meter rise in sea
levels, 200 million could become homeless.
There are other fears too, (19) to a recent United Nations report. The plight of the hungry
in northern Africa could (20) as rainfall in the Sahara and beyond is reduced by 20 per cent.
PART II. READING TEST (7PTS)
• You are going to read a text about closed. circuit television (CCTV) in public places.
• Seven paragraphs have been removed from the text.
• Choose from the paragraphs A-H the one which fits each gap (1.7)
• There is one extra paragraph which you do not need to use.
WE'VE ALL BEEN FRAMED
Everybody's on television now. We are routinely filmed as we walk down the high street and enter
the shop to buy a newspaper. Police cameras take over as we drive down the road to drop our children
at school. Another hidden eye watches the playground for anything suspicious. And so it goes on - in
the office, at the cashpoint, at shopping malls, stations, airports, car parks, football grounds, public
squares, even public conveniences.
1. →
Do the claims for drastic crime reduction attributed to CCTV by the government and local authorities
stand up to independent analysis? Could the £1bn spent on monitoring and system costs over the past
decade have been used more effectively? If viewing surveillance is a form of power, what limits are
placed on its operation by the democratic and legal processes?
2. →
When we meet in Hull, Norris and I travel to his home, where there are 10 cameras focused on
various parts of the high street. While I pay the cab driver, Norris is switching off the burglar alarm.
Aha! So he's not against using modern technology to prevent crime? Of course not. Nor does he appear
enthusiastic when I ask if he would like to get rid of all CCTV cameras tomorrow.
3. →
I've never been convinced, though', he continues, that there could be a simple solution to crime.
One of my main complaints is that the last government invested 80% of the crime. prevention budget
on technology which was never properly evaluated.

5|Page Tài liệu khóa học Live - VIP


OTTO CHANNEL
Otto Channel
4. →
Norris and Armstrong felt it was high time to do some evaluating themselves. They spent days,
nights, and weekends in three different control rooms - one in a poor, multi-racial inner-city area, one
in a prosperous country town and one in a major city center. 'In a busy street, says Norris, 'there are
hundreds of issues to focus on. So how do you decide who's a likely trouble. maker and who's not?
The answer, in all cases, is that it's based on crude stereotypes.'
5. →
Norris is slightly surprised that a country where the concept of Big Brother has become part of the
language should accept so many 'little brothers and sisters' to the point where its citizens are, he says,
the most filmed in the world "without any democratic or legal controls'. To which I point out that most
people assume that if they've done nothing wrong then they have nothing to fear.
6. →
State concern? What has the state got to do with it? 'People think of a camera operator watching
over them kindly but all the information is being stored. Real-time images can be connected to
computers to be analyzed.'
7. →
What he sees as the possible long-term implications can best be summed up by the penultimate
paragraph of the book: The history of the 20th century should remind us that democratic institutions
are not assured. They can be, and have been, captured by totalitarian regimes of both left and right.
We should not trust in the myth of a benevolent government, for while it may be only a cynic who
questions the benign intent of their current rulers, it would surely be a fool who believed that such
benevolence is assured in the future.'
A. 'No, probably not,' he replies after a pause. They can be effective in limited circumstances - in
car parks, for instance. And with the new generation of speed cameras, we have a chance to
reduce pedestrian deaths in urban areas. Their use on railway crossings seems highly sensible
and when cameras allow the police to find a bomber, a mugger or a murderer then none of us
could say it wasn't a social good.
B. Norris disagrees. 'We all have something to hide,' he says. 'People have affairs. People hide
their true feelings about others. Are these really matters of state concern?"
C. Answers to these and many other questions are to be found in Norris and Armstrong's book,
The Maximum Surveillance Society: The Rise of CCTV. I decided to meet one of them in person.
D. So where is all this leading? Should we be alarmed about what is likely to happen in the future -
not tomorrow or the next day, perhaps, but some years from now?
E. In other words the targets are men rather than women, young men rather than middle. aged or
elderly men. If you're a young man in a baseball cap, then your every move is likely to be under
observation. 'Older men are largely ignored,' Norris says.
F. Occasionally, we catch sight of ourselves on a screen in one of these places. But the real
addicts of closed. circuit television are the ones who are paid to watch, says. day and night. Dr
Clive Norris and Dr Gary Armstrong have spent a total of 600 hours in control rooms watching

6|Page Tài liệu khóa học Live - VIP


OTTO CHANNEL
Otto Channel
the people who watch us. Both are lecturers in criminology and both are worried about the
phenomenal growth of CCTV surveillance in recent years. Accordingly, they set out to ask some
questions.
G. If the control room spots one of these crimes taking place, it doesn't mean that the police or
the security guards will respond, he says. They have their own agendas. In our 600 hours they
went into action just 43 times."
H. The present government, on the other hand, has begun a massive program of crime reduction
and they should be congratulated on providing a lot of money for evaluation. But while the use
of CCTV continues to spread, there still hasn't been a properly conducted survey into its
effectiveness.'
PART III. WORD FORMATION (18PTS)
A. Supply the correct form of the word provided in brackets in cook sentence.
1. His busy schedule made him completely (access) to his students.
2. He works for UNESCO in a purely (advice) role.
3. The sun and the moon are often (person) in poetry.
4. I've never known such a (quarrel) person.
5. I don't care if you had had too much to drink. Your behavior last night was (defend) .

B. Choose the right verbs provided in the box, then use the most suitable forms of the verbs to fill
in the numbered blanks.

break call slow take draw

6. The red car has just up in front of our house. Are we expecting anyone?
7. His condition is worse than before. I think we should in a doctor.
8. I've won a million pound! I don't believe it! I simply can't it in!
9. Could you down, please. I don't like driving so fast on country roads.
10. I'm sorry. I'm late. The car has down again. I've left it about a mile down the street.

C. Supply the correct tense of the verb in brackets.


• You must tell me the truth. I insist on (1. tell) the truth.
• (2. Thompson catch) the ball, we would have won the game.
• Don't worry. We (3. finish) the report by 11 o'clock.
• Fred was pleased (4. admit) to the college.
• There were some people (5. row) on the river.
• Alex has a test tomorrow that he needs to study for. He (6. not watch) TV right now,
• Were I (7. know) the answer, I (8. tell) you right away.

7|Page Tài liệu khóa học Live - VIP


OTTO CHANNEL
Otto Channel
• By the end of last year he (10. read) four Shakespeare plays and by next year he
(10. read) two more.
PART IV. SETENCE TRANSFORMATION (20PTS)
A. Finish each of the sentences in such a way that it means exactly the same as the sentence
printed before it.
1. Tim insisted on being told the complete story.
→ Nothing but ..................................................................................................................
2. She never seems to succeed, even though she works hard.
→ However .......................................................................................................................
3. Andrew is the most generous person I have ever met.
→ I've yet .........................................................................................................................
4. She never seems to succeed even though she studies much.
→ Much .............................................................................................................................
5. I never thought that I would win a prize It had
→ .....................................................................................................................................
B. Write a new sentence using the word given.
1. I don't think the television's likely to blow up at any minute. Likelihood
→ .....................................................................................................................................
2. This car only cost me five hundred pounds. Picked
→ .....................................................................................................................................
3. Someone paid five thousands pounds for the painting. Went
→ .....................................................................................................................................
4. We have made neither a profit nor a loss this year. Even
→ ......................................................................................................................................
5. In 1967 programs began to be transmitted in color. Advent
→ .....................................................................................................................................

THE END

8|Page Tài liệu khóa học Live - VIP


OTTO CHANNEL
Otto Channel
SỞ GIÁO DỤC & ĐÀO TẠO KỲ THI OLYMPIC TRUYỀN THỐNG 30/4
TP. HỒ CHÍ MINH LẦN THỨ XXV – NĂM 2008
ĐỀ CHÍNH THỨC Môn thi: Anh văn - Khối: 10
Thời gian làm bài: 180 phút

➢ Thí sinh làm phần trắc nghiệm (MULTIPLE CHOICE) trên phiếu trả lời trắc nghiệm và phần tự
luận (WRITTEN TEST) trên phiếu trả lời tự luận.
➢ Trên phiếu trả lời trắc nghiệm, thí sinh tô thêm 2 số 00 vào trước số báo danh (bằng bút chì).
Phần mã đề thi trên phiếu trắc nghiệm, thí sinh tô vào ô 002.
 --------------------------------------------------------------------------------------------------------------------

CẤU TRÚC ĐỀ THI


SECTION A. MULTIPLE CHOICE (40PTS) ............................................................................. 1
PART I. PHONOLOGY (5PTS) .......................................................................................................................... 1
PART II. ERROR IDENTIFICATION (5PTS) ................................................................................................... 1
PART III. VOCABULARY (5PTS) ......................................................................................................................... 2
PART IV. STRUCTURES (5PTS) ......................................................................................................................... 3
PART V. PREPOSITIONS (5PTS) ..................................................................................................................... 3
PART VI. CLOZE TEST (5PTS) ........................................................................................................................... 4
PART VII. READING COMPREHENSION (10PTS) .......................................................................................... 5

SECTION B. WRITTEN TEST (60PTS) ................................................................................... 7


PART I. OPEN CLOZE TEST (20PTS) ............................................................................................................ 7
PART II. WORD FORMS (20 PTS) ................................................................................................................... 8
PART III. SENTENCE TRANSFORMATION (20PTS) ..................................................................................... 9
Otto Channel
SỞ GIÁO DỤC & ĐÀO TẠO KỲ THI OLYMPIC TRUYỀN THỐNG 30/4
TP. HỒ CHÍ MINH LẦN THỨ XXV – NĂM 2008
ĐỀ CHÍNH THỨC Môn thi: Anh văn - Khối: 10
Thời gian làm bài: 180 phút

➢ Thí sinh làm phần trắc nghiệm (MULTIPLE CHOICE) trên phiếu trả lời trắc nghiệm và phần tự
luận (WRITTEN TEST) trên phiếu trả lời tự luận.
➢ Trên phiếu trả lời trắc nghiệm, thí sinh tô thêm 2 số 00 vào trước số báo danh (bằng bút chì).
Phần mã đề thi trên phiếu trắc nghiệm, thí sinh tô vào ô 002.
 --------------------------------------------------------------------------------------------------------------------
SECTION A. MULTIPLE CHOICE (40PTS)
PART I. PHONOLOGY (5PTS)
Choose the word whose underlined part is pronounced differently from those the others of the
group.
1. A. pizza B. lizard C. puzzle D. muzzle
2. A. devotion B. congestion C. suggestion D. question
3. A. comfortable B. intimacy C. apprehension D. disappointed
4. A. garbage B. massage C. language D. sewage
5. A. cement B. movement C. comment D. momentum
Choose the word whose stress pattern is different from the others of the ground.
6. A. superstitious B. melancholy C. obstacle D. sanguineness
7. A. regional B. respondent C. reservoir D. register
8. A. Aborigine B. appearance C. emergency D. contribute
9. A. triangle B. refugee C. commerce D. opportune
10. A. academic B. amphibian C. apartheid D. aquarium

PART II. ERROR IDENTIFICATION (5PTS)


Choose the underlined part of the sentence that needs correction.
11. If the water level had raised any higher, the dam would probably have broken.
A B C D
12. Though difficult to master, the guitar is embraced by many of beginning music students
A B C
because of its musical versatility, low cost, and extreme popularity.
D
13. As soon as 8000B.C., people began to experiment with methods of helping the sick.
A B C D
14. The Concorde can fly across the Atlantic without re-fueling and carrying 11 tons of freight.
A B C D
15. The problem of inadequate salaries for teachers not only exists in smaller communities but also
A B C D
in large cities.

1|Page Tài liệu khóa học Live - VIP


OTTO CHANNEL
Otto Channel
16. Investment banks do not accept deposits from the public and make loans to businesses or
A B C D
individuals.
17. The American soprano Mary Gardour, who had one of the greatest operatic voices of her era,
A B
retired at the height of the career.
C D
18. The art of tapestry-making involves forming a picture or design on fabric using coloring threads.
A B C D
19. Many scientists contributed to the development of television, so that a single person can be said
A B C
to have invented it.
D
20. White blood cells live only for a few days because they are poisoned by bacterium they capture.
A B C D

PART III. VOCABULARY (5PTS)


Choose the option that best fits the blank of the sentence.
21. During the rush hour the traffic in the city centre is terrible.
A. condensation B. congestion C. accumulation D. concentration
22. Children can be difficult to teach because of their short attention
A. limit B. duration C. span D. time
23. This is a example of what not to do.
A. critical B. pure C. simple D. prime
24. , the people who come to this club are in their twenties and thirties.
A. By and large B. Altogether C. To a degree D. Virtually
25. The little boy was left in of his grandmother during his parents' absence.
A. charge for B. the charge of C. care for D. care of
26. I am afraid a rise in salary is just now.
A. out of sight B. out of control C. out of date D. out of question
27. Some people feel that television should give less really to sport.
A. programs B. coverage C. concern D. involvement
28. Those campers are really . They have no idea how to set up a tent.
A. blue B. black C. green D. white
29. You're too old to carry on working. It's time you called it a .
A. year B. month C. week D. day
30. How could you just abandon me in the middle of London with no money and leave me and
dry.
A. cold B. out C. low D. high

2|Page Tài liệu khóa học Live - VIP


OTTO CHANNEL
Otto Channel
PART IV. STRUCTURES (5PTS)
Choose the option that best fits the blank of the sentence.
31. The play was not what we had expected.
A. just B. absolutely C. at all D. very
32. I'll be kind to her she decides to leave me.
A. in case B. so that C. for fear that D. lest
33. I don't suppose there is anyone there, ?
A. is there B. isn't there C. do I D. don't I
34. , I would give a party.
A. Were she to come next month B. She were to come next month
C. If she comes next month D. Should she come next month
35. The parcel I sent you by now.
A. is supposed to arrive C. should have arrived
B. should arrive D. would arrive
36. On the battle field .
A. the tanks lay B. did the tanks lie C. lay the tanks D. lied the tanks
37. Mr. Gump supposes, that he will retire at 60.
A. like most people did C. like most people do
B. as most of people D. as do most people
38. Ludwig Van Beethoven is considered one of the greatest composers
A. to live B. when living C. who ever lived D. while he was living
39. , the catfish is prized for its taste.
A. With ugly look B. As ugly looking
C. Ugly looking as it is D. As it is ugly looking
40. 'If it is a girl, she is going to be called Etheldreda.
"What name to give girl!"
A. Ø/ the B. a/a C. the / the D. the /Ø
PART V. PREPOSITIONS (5PTS)
Choose the option that best fits the blank of the sentence.
41. The head is proportion with the body.
A. out of B. outside C. off D. away from
42. How exactly did you set training the dogs to work well together?
A. up B. to C. about D. out
43. It really gets me having to work so much overtime.
A. up B. on C. by D. down
44. It seems to be your boss who is fault in this case.
A. under B. at C. with D. for

3|Page Tài liệu khóa học Live - VIP


OTTO CHANNEL
Otto Channel
45. The red light goes on outside the studio door to let people know that you are air
A. on the B. by C. in D. through the
46. Please and see us some time. You're always welcome.
A. come to B. come about C. come round D. come away from
47. The elaborate bridal costumes of the coastal Indians are mother to daughter.
A. taken after B. put by C. parted with D. handed down
48. I'm very tired. Joan invited me to dinner at her house, but I don't up to it. I shall go to bed
early.
A. look B. think C. feel D. come
49. Such relaxed days-were few and far in her hectic life.
A. off B. between C. beyond D. out
50. It's no good pretending; you've got to reality.
A. bargain for B. come up against C. get down to D. face up to

PART VI. CLOZE TEST (5PTS)


Choose the words or phrases that best fit the blanks to make a complete passage.
FUN AND GAMES
Do think no. computer games are just for kids? Then you should think again. You might be
surprised to learn that the games industry now makes more money than Hollywood. (51) a
family buy a new PC, all they really want to do is to play games.
It is (52) surprising that video gaming has become one of the most popular (53) of
entertainment today. A good game is like a good film; it will hold your (54) . (55) your
imagination and play with your emotions.
The big difference, however, is that watching a movie is a passive (56) You have no say in
how the (57) develops or which characters dominate the story. With computer games, you
direct the action and that is (58) makes them so exciting. Finding the (59) game is
likely to signal the beginning of a lasting love affair with the interactive world of make-believe.
It is wrong to think of gaming as something simply for children and teenagers. In fact, the (60)
growth area of the market is the 25-35 age group.
51. A. As soon as B. As well as C. As far as D. As long as
52. A. whatsoever B. no C. hardly D. barely
53. A. makes B. means C. sorts D. forms
54. A. feelings B. attention C. breath D. control
55. A. hold B. capture C. grasp D. clutch
56. A. pastime B. action C. passion D. fun
57. A. event B. content C. account D. plot
58. A. thing B. which C. what D. part that
59. A. accurate B. right C. complete D. proper
60. A. broadest B. widest C. biggest D. farthest

4|Page Tài liệu khóa học Live - VIP


OTTO CHANNEL
Otto Channel
PART VII. READING COMPREHENSION (10PTS)
PASSAGE 1
Read the following passage and choose the best answers to the questions.
Supernovas are the most powerful and spectacular outbursts known in nature. What is called a
Type II supernova is due to the collapse of a massive star, at least eight times as massive as the
sun, that has used up its main nuclear fuel and produced a nickel-iron core. When this core can no
longer support the pressure of the star's outer layer, it collapses to form a neutron star of immense
density. Over 2,500 million tons of neutron star material could be packed into a matchbox.
Its temperature is around 100,000 million degrees centigrade. Multitudes of neutrons are
produced in the collapsed star, which pass directly through the star into space, and this release of
neutrons causes the core to respond with a shock wave that moves outward. When it meets the
material that is falling inward, the result is a catastrophic explosion. Sometimes most of the star's
material is blown away, leaving only a small, incredibly dense remnant that may be a neutron star or,
in extreme cases, a black hole.
A supernova is often more than 500 million times as luminous as the sun. A supernova
remnant (SNR) may be detectable as a pulsar, an example of which is the Crab Nebula, known to
be a remnant of the supernova observed in the year 1054. The 1987 supernova in the Large Cloud
of Magellan had a low peak luminosity by supernova standards, only about 250 million times that of
the sun. At its brightest the supernova shone as a star between magnitudes 2 and 3, even though it
was 170,000 light-years away.
61. What is the main topic of the passage?
A. The heat of supernovas. B. The formation and power of a supernova.
C. The role of shock waves in a supernova. D. The density of a neutron star.
62. The word "it" refers to the
A. core of the collapsed B. neutron star
C. shock wave D. catastrophic explosion
63. According to the passage, which of the following is TRUE about the 1987 supernova?
A. It was the brightest supernova.
B. It was brighter than the sun.
C. It shone as a star between magnitudes 3 and 4.
D. It had the lowest peak luminously.
64. The word "detectable" is closest in meaning to
A. assumed B. known C. perceptible D. audible
65. Which of the following words can best be substituted for "remnant"?
A. characteristic B. relic C. specter D. remainder
66. The author of this passage is most likely .
A. an astronomer B. an economist C. a mathematician D. a botanist
67. The word "multitudes" is closest in meaning to
A. small numbers B. groups C. lots D. temperatures
5|Page Tài liệu khóa học Live - VIP
OTTO CHANNEL
Otto Channel
68. The word "luminous" is closest in meaning to
A. powerful B. bright C. hot D. distant
69. What can be inferred from the passage about supernovas?
A. They sometimes result in a black hole. B. The sun is a remnant of a supernova.
C. They occur when two stars collide. D. They only happen to pulsars.
70. According to the passage, what marks the beginning of a supernova?
A. The neutrons of a star become very dense. B. A massive star uses up its main nuclear fuel.
C. A star has grown too big. D. A star is born.
PASSAGE 2
Read the following passage and choose the best answers to the questions.
Marian Anderson's brilliant singing career began at age six when she sang spirituals at the
Union Baptist Church in her hometown of Philadelphia. She toured Europe in the 1920s, drawing vast
acclaim; however, when she returned to the United States she was still barred from performing on
the American operatic stage. Strict segregation laws were in force at the time, keeping many black
performers out of exclusively white theaters and concert halls. After she was prevented from singing
in Washington's segregated Constitution hall in 1939, Eleanor Roosevelt intervened and arranged
for Miss Anderson to perform at the Lincoln Memorial. Marian Anderson's beautiful contralto voice
broke down racial barriers, showing white Americans that blacks had a profound contribution to
make to America's cultural life. Eventually, in 1955, she became the first African-American singer to
perform at New York's Metropolitan Opera. In her many years of touring she had to endure a racism
that forced her to enter concert halls and hotels through service entrances. Her grace under this
stress showed a moral perseverance that paralleled that of the famous Marin Luther King, Jr.
71. We can conclude from the passage that Marian Anderson first toured Europe instead of the
United States because
A. she was paid more in Europe
B. she was not allowed to perform in the United States
C. there were better operatic facilities in Europe
D. it was too expensive to tour in the United States
72. The word "acclaim" could best be replaced by
A. publicity B. fund C. approval D. attention
73. The word "intervened" could best be replaced by
A appreciated B. supported C. interrupted D. interfered
74. The significance of Anderson's Lincoln Memorial performance was that
A. Eleanor Roosevelt arranged it C. 75,000 people came
B. her contralto voice was beautiful D. she was a black performer
75. The phrase “broke down racial barriers" means
A. disclosed opportunities C. revealed inaccuracies
B. shattered obstacles D. analyzed destinations

6|Page Tài liệu khóa học Live - VIP


OTTO CHANNEL
Otto Channel
76. Where is the best place in the passage to add the following sentence? "
A crowd of 75,000 people came to watch her sing before the Memorial.
A. after the word "Memorial" B. after the word “Stage”
C. after the word "Opera” D. after the word "entrances”
77. The word "grace" is similar in meaning to
A. awkwardness B. cruelty C. elegance D. saintliness
78. According to the passage, what did Marian Anderson have in common with Martin Luther King,
Jr.?
A. Moral perseverance C. a performance at the Lincoln Memorial
B. a clear strong voice D. singing in church
79. The author's tone in this passage is
A. instructive B. critical C. respectful D. regretful
80. What does the word "this" refer to?
A. touring for many years C. performing before thousands of people
B. racist attitudes toward her D. being like Martin Luther King, Jr

SECTION B. WRITTEN TEST (60PTS)


PART I. OPEN CLOZE TEST (20PTS)
Fill in each blank with ONE word to make a complete passage.
OPEN CLOZE TEST 1
Education is not an end, but a means (1) an end. In other words, we do not educate
children only for the purpose of educating them. Our purpose is to fit them for life.
In some modern countries it has for some time been fashionable to think that by free education
for all- (2) rich or poor, clever or stupid – one can solve all the problems of society and build
a perfect nation.
But we can already see that free education for all is not enough. We find in such countries a large
number of people with university (3) who refuse to do what they think "low" work. In fact, work
with hands is thought to be dirty and shameful in such countries. But we only have to think a (4)
to understand that the work of a completely uneducated farmer is (5) more important
than that of a professor. We can live (6) education, but we die if we have no food. If no one
cleaned our streets or (7) the rubbish from our house, we should get terrible diseases in our
towns.
In fact, when we say that all of us must be educated to be fitted for life, it means that we must
be educated in (8) a way each of us can do whatever work suited to our brains and ability and we
can realize that all jobs are necessary to society, and that it is very bad to be ashamed of one's work.
(9) such a type of education can be considered valuable (10) society.
OPEN CLOZE TEST 2
The blue whale is the biggest animal known to human beings. Bigger than (11) dinosaurs,
blue whales are (12) because they are warm-blooded and they breathe through their lungs.

7|Page Tài liệu khóa học Live - VIP


OTTO CHANNEL
Otto Channel
Every five to ten minutes, blue whales (13) from the sea to blow out the stale air from their lungs
through an (14) on the top of their heads. As the air is blown out, the water vapour that escapes
is visible at a distance as a (15) . The whales have to do this several times to fully clear their lungs
of the used air. Then they disappear into the sea again with the fresh air in their (16) .
For many years, the blue whale was hunted by men called (17) . They wanted a special
kind of bone found in the whale's mouth that is light and (18) easily without breaking.
This whalebone was used to make hoops for circular skirts that were fashionable for women then.
Today the blue whale is practically (19) . Other types of whales such as the killer whale and
the gray whale are (20) special protection. The only species of whale that seems to have survived
well is the dolphin.
PART II. WORD FORMS (20 PTS)
A. Complete the sentences with the correct forms of the given words.
1. It's when people won't believe things that are obviously true. FURY
2. This statue the soldiers who died in the war. MEMORY
3. The world champion was by a younger Russian challenger. THRONE
4. , we could still win, but it's not very likely. THEORY
5. Newts, frogs and toads are animals. AMPHIBIAN
6. They never dare to leave their only child for even a moment. ATTEND
7. The place where you can wash your clothes with coin-operated washing machines is called
. LAUNDRY
8. They live in a remote area, except by car. ACCESS
9. He was taken to court for . PAY
10. Many countries have agreed on the treaty. ARM
B. Complete the following passage with the correct forms of the words given in the box.

fly special apply geology operate


automatic computer quality analyse fill

(11) in cybernetics are working on the development of self-learning machines


capable of solving problems set by man. These self-learning cybernetic systems can collect various
data, analyze them and perform certain (12) as a result of the (13) . What is more,
they are able to produce (14) new information. For example, a cybernetic machine for chemists
will not only give information on any specific problem but will indicate the way to develop some new
substance, say, a new plastic that would be cheap and durable.

8|Page Tài liệu khóa học Live - VIP


OTTO CHANNEL
Otto Channel
A good number of these machines are already working in our industry, such as a cybernetic
system for the iron and steel industry, an electronic system for (15) prospecting, a cybernetic
designer which designs gas pipelines and automatizes the gas along these pipes.
(16) in our country has reached the stage of developing (17) automatized industries.
Electronic (18) techniques find wide (19) in and serve as a basis for the development of
modern programme- controlled machine tools, the controlling of spaceship (20) , weather
forecasts and planning work.
PART III. SENTENCE TRANSFORMATION (20PTS)
Rewrite the sentence with the given word or the given beginning so that the new sentence has the
same meaning as the previous one.
1. I'll be thinking of you and hoping that you have good luck on the day of your interview.
(fingers)
→ .....................................................................................................................................
2. 2. He maintained his position against his adversary. (ground)
→ .....................................................................................................................................
3. 3. I left without saying goodbye as I didn't want to disturb the meeting.
→ Rather ..........................................................................................................................
4. Doris tiptoed up the stairs because she didn't want to wake the baby up. (lest)
→ .....................................................................................................................................
5. Public opinion was so strong that the Prime Minister had to resign.
→ Such ..............................................................................................................................
6. The mistake in the accounts was not noticed until the figures were re-checked. (light)
→ .....................................................................................................................................
7. Andrew doesn’t claim to have a lot of musical talent. (pretense)
→ .....................................................................................................................................
8. The book interestingly describes the life of Marx as a young man. (account)
→ The book ............................................................................. as a young man.
9. The local council have considered mass tourism the cause of the environmental problems. (put)
→ .....................................................................................................................................
10. Don't conclude that learning English is easy. (come)
→ .....................................................................................................................................

THE END

9|Page Tài liệu khóa học Live - VIP


OTTO CHANNEL
Otto Channel
SỞ GIÁO DỤC & ĐÀO TẠO KỲ THI OLYMPIC TRUYỀN THỐNG 30/4
TP. HỒ CHÍ MINH LẦN THỨ XXV – NĂM 2009
ĐỀ CHÍNH THỨC Môn thi: Anh văn - Khối: 10
Thời gian làm bài: 180 phút

➢ Thí sinh làm phần trắc nghiệm (MULTIPLE CHOICE) trên phiếu trả lời trắc nghiệm và phần tự
luận (WRITTEN TEST) trên phiếu trả lời tự luận.
➢ Trên phiếu trả lời trắc nghiệm, thí sinh tô thêm 2 số 00 vào trước số báo danh (bằng bút chì).
Phần mã đề thi trên phiếu trắc nghiệm, thí sinh tô vào ô 002.
 --------------------------------------------------------------------------------------------------------------------

CẤU TRÚC ĐỀ THI


SECTION A. MULTIPLE CHOICE (20PTS) ............................................................................. 2
PART I. PHONOLOGY (5PTS) .......................................................................................................................... 2
PART II. GUIDED CLOZE (5PTS) ..................................................................................................................... 2
PART III. READING COMPREHENSION (10PTS) .......................................................................................... 3

SECTION B. WRITTEN TEST (80PTS) ................................................................................... 6


PART I. VERB TENSES /FORMS (10PTS) ..................................................................................................... 6
PART II. PREPOSITIONS & PHRASAL VERBS (20PTS) ............................................................................. 6
PART III. WORD FORMS (20PTS) ..................................................................................................................... 7
PART IV. OPEN CLOZE TEST (10PTS) ............................................................................................................ 8
PART V. ERROR IDENTIFICATION (10PTS) ................................................................................................ 9
PART VI. SENTENCE TRANSFORMATION (10PTS) ..................................................................................... 9
Otto Channel
SỞ GIÁO DỤC & ĐÀO TẠO KỲ THI OLYMPIC TRUYỀN THỐNG 30/4
TP. HỒ CHÍ MINH LẦN THỨ XXV – NĂM 2009
ĐỀ CHÍNH THỨC Môn thi: Anh văn - Khối: 10
Thời gian làm bài: 180 phút

➢ Thí sinh làm phần trắc nghiệm (MULTIPLE CHOICE) trên phiếu trả lời trắc nghiệm và phần tự
luận (WRITTEN TEST) trên phiếu trả lời tự luận.
➢ Trên phiếu trả lời trắc nghiệm, thí sinh tô thêm 2 số 00 vào trước số báo danh (bằng bút chì).
Phần mã đề thi trên phiếu trắc nghiệm, thí sinh tô vào ô 002.
 --------------------------------------------------------------------------------------------------------------------
SECTION A. MULTIPLE CHOICE (20PTS)
PART I. PHONOLOGY (5PTS)
Choose the word whose underlined part is pronounced differently from the other three

1. A. blessed B. curried C. crooked D. kicked


2. A. horrific B. horrible C. horizontal D. complex
3. A. pizza B. sizzle C. drizzle D. muzzle
4. A. layer B. prayer C. betrayer D. sprayer
5. A. decoy B. perishable C. benzene D. supreme
Choose the word whose stress pattern is different from that of the other three.

6. A. condolence B. obstinacy C. communism D. painstakingly


7. A. Aborigine B. aerobics C. theatergoer D. caretaker
8. A. Arabic B. agriculture C. lunatic D. politics
9. A. intimacy B. commentary C. preferable D. amphibian
10. A. volunteer B. absentee C. committee D. employee

PART II. GUIDED CLOZE (5PTS)


Read the following passage and choose the options that best complete the blanks.
Throughout our lives, right from the moment when as infants we cry to (11) our hunger,
we are engaged in social interaction of one form or another. Each and every time we encounter (12)
human beings, some kind of social interaction will take place, (13) it's getting on a bus
and paying the fare for the journey, or socializing with friends. It goes without saying, therefore, that
we need the ability to communicate. Without some method of (14) intentions, we would be at
a(n) (15) loss when it came to interacting socially.
Communication (16) the exchange of information which can be anything from a gesture to a
friend signaling boredom to the presentation of a university thesis which may only ever be read by a
(17) of others, or it could be something in between (18) . Our highly (19) languages set us
apart from animals. But for these languages, we could not communicate sophisticated or abstract ideas.
Nor could we talk or write about people or objects not immediately present. Were we restricted to
discussing objects already present, we would be unable to make abstract (20) about the world.

2|Page Tài liệu khóa học Live - VIP


OTTO CHANNEL
Otto Channel
11. A. show B. express C. inform D. communicate
12. A. fellow B. close C. neighbor D. companion
13. A. either B. even C. if D. whether
14. A. transferring B. giving C. spreading D. transmitting
15. A. utter B. complete C. absolute D. whole
16. A. requires B. needs C. means D. involves
17. A. lot B. group C. handful D. few
18. A. the two B. the others C. both D. them
19. A. advanced B. progressed C. developed D. constructed
20. A. guesses B. predictions C. questions D. generalizations

PART III. READING COMPREHENSION (10PTS)


PASSAGE 1
Read the following passage and choose the best option to complete the blank or answer the question.
Among the species of seabirds that use the windswept cliffs of the Atlantic coast of Canada in the
summer to mate, lay eggs and rear their young are common murres, Atlantic puffins, black-legged
kittiwakes, and northern gannets. Of all the birds on these cliffs, the black-legged kittiwake gull is the
best suited for nesting on narrow ledges. Although its nesting habits are similar to those of gulls that
nest on flat ground, there are a number of differences related to the cliff-nesting habit. The advantage
of nesting on cliff is the immunity it gives from foxes, which cannot scale the sheer rocks, and from
ravens and other species of gulls which have difficulty in landing on narrow ledges to steal eggs. This
immunity has been followed by a relaxation of the defenses, and kittiwakes do not react to predators
as fiercely as do ground-nesting gulls. A colony of Bonaparte's gulls responds to the of a predatory
herring gull by flying up as a group with a clamor of appearance alarm calls, followed by concerted
mobbing, but kittiwakes dimply ignore herring gulls, since they pose little threat to nests on cliffs.
Neither do kittiwakes attempt to conceal their nest. Most gulls keep the nest area clear of droppings,
and remove empty eggshells after the chicks have hatched, so that the location of the nest is not given
away. Kittiwakes defecate over the edge of the nest, which keeps it clean, but this practice, as well as
their tendency to leave the nest littered with eggshells, makes its location very conspicuous. On the
other hand, nesting on a narrow ledge has its own peculiar problems, and kittiwake behavior has
become adapted to overcome them. The female kittiwake sits when mating, whereas other gulls stand,
so the pair will not overbalance and fall off the ledge. The nest is a deep cup, made of mud or seaweed,
to hold the eggs safely, compared with the shallow scrape of other gulls, and the chicks are remarkably
immobile until fully grown. They do not run from their nests when approached, and if they should
come near to the cliff edge, they instinctively turn back.
21. What aspect of the kittiwake gull does the passage mainly discuss?
A. Its defensive behavior. C. Its nesting habits.
B. Its mating season. D. Its mating habit.
22. The word 'it' in line 6 refers to .
A. immunity B. cliff-nesting habit C. advantage D. nesting
23. The word 'scale' in line 6 is closest in meaning to
A. climb B. avoid C. approach D. measure

3|Page Tài liệu khóa học Live - VIP


OTTO CHANNEL
Otto Channel
24. The word ‘relaxation' in line 8 is closest in meaning to
A. negligence B. weakness C. liberation D. immobility
25. The word 'immunity' in line 8 is closest in meaning to
A. distance B. transition C. protection D. reminder
26. Why is it difficult for ravens to steal the kittiwakes' eggs?
A. The kittiwakes can see the ravens approaching the nest.
B. The ravens find it difficult to read the narrow ledges where kittiwakes nest.
C. The kittiwakes' eggs are too big for the ravens to carry.
D. The female kittiwakes rarely leave the nest.
27. The author mentions that eggshells litter the nests of kittiwakes in order to
A. demonstrate that kittiwakes are not concerned about predators
B. prove how busy kittiwakes are in caring for their offspring
C. show a similarity to other types of gulls
D. illustrate kittiwakes' lack of concern for their chicks
28. Which of the following statements is TRUE?
A. Kittiwakes cannot defend themselves from predators.
B. Gulls and kittiwakes have the same nesting habits.
C. Gulls and kittiwakes have the same mating habits.
D. Kittiwakes can raise their off-spring.
29. Which of the following things doesn't help to make the gull's nest less conspicuous?
A. Keeping its nest away from its droppings
B. Throwing away eggshells after its young have come into existence
C. Filling its nests with eggshells
D. Making its nest out of the areas with its waste
30. The word 'conspicuous' in line 15 is closest in meaning to
A. disordered B. suspicious C. noticeable D. appealing
PASSAGE 2
Read the following passage and choose the best option to complete the blank or answer the question.
Ralph Earl was born into a Connecticut farm family in 1751. He chose early to become a painter
and looked for what training was available in his home state and in Boston. Earl was one of the first
American artists to paint landscapes. Among his first paintings were scenes from the Revolutionary
War battles of Lexington and Concord. In 1778 Earl went to London to study with Benjamin West for
four years.
When Earl returned to the United States, he was jailed for fourteen months for outstanding debts.
While still a prisoner, he painted portraits of some of New York City's most elegant society women and
their husbands. After his release, be took up the trade of itinerant portrait painter, working his way
through southern New England and New York. Earl didn't flatter his subjects, but his portrait show a
deep understanding of them, perhaps because he had sprung from the same roots.
Among Earl's most famous paintings is his portrait of Justice Oliver Ellsworth and his wife, Abigail.
To provide counterpoint to the severity of the couple, he accurately details the relative luxury of the

4|Page Tài liệu khóa học Live - VIP


OTTO CHANNEL
Otto Channel
Ellsworth's interior furnishings. The view through the window behind them shows sunlit fields, well-
kept fences, and a bend of the Connecticut River. One of Earl's paintings is something of anomaly.
Reclining Hunter, which many years was attributed to Thomas Gainsborough, shows a well-
dressed gentleman resting beneath a tree. In the foreground, he displays a pile of birds, the results
of a day's hunt. The viewer can also see a farmer's donkey lying in the background, another of the
hunter's victims. This outrageously funny portrait couldn't have been commissioned no one would have
wanted to be portrayed in such a absurd way. However, this painting uncharacteristically show Earl's
wit as well as his uncommon technical skills
31. What is the author's main purpose?
A. To discuss the life and work of an American painter
B. To compare the art of Ralph Earl and Thomas Gainsborough
C. To trace Ralph Earl's artistic influences
D. To describe the art scene in New York in the late eighteenth century
32. Which of the following is NOT given in the passage as a subject of one of Earl's paintings?
A. People B. Landscapes C. Battle scenes D. Fruit and flowers
33. According to the passage, Benjamin West was Ralph Earl's
A. subject B. teacher C. student D. rival
34. Which of the following could be substituted for outstanding without changing the meaning of
the sentence?
A. Traveling B. Successful C. Talented D. Innovative
35. The word itinerant is closest in meaning to which of the following?
A. Excellent B. Shocking C. Unpaid D. Illegal
36. The author uses the phrase sprung from the same roots to indicate that Ralph Earl and his
subjects
A. lived in the same town B. were about the same age
C. were equally successful D. had the same background
37. According to the passage, one of the distinguishing features of the portrait of Oliver and Abigail
Ellsworth is the contrast between
A. the plainness of the figures and the luxury of the furnishings
B. the two styles used to paint the two figures
C. the sunlit fields and the dark interior
D. the straight fences and the curving Connecticut River
38. Why does the author refer to Reclining Hunter as "something of an anomaly?
A. It is so severe B. It is quite humorous
C. It shows Earl's talent D. It was commissioned
39. The word he refers to .
A. Ralph Earl B. the farmer C. the hunter D. Thomas Gainsborough
40. The author's attitude towards Ralph Earl is .
A. admiring B. antagonistic C. neutral D. unflattering

5|Page Tài liệu khóa học Live - VIP


OTTO CHANNEL
Otto Channel
SECTION B. WRITTEN TEST (80PTS)
PART I. VERB TENSES /FORMS (10PTS)
Use the correct forms /tenses of the given words.
• What a shame! I'd rather you (1. not/dismiss) like that.
• When I noticed the blank stare he (2. give) me, I (3. realize) that he didn't understand
me.
• Would you be so kind as (4. switch off) the lights when leaving?
• Just inside the outer later of the earth's atmosphere (5. be) the elements necessary to
protect it from ultraviolet rays, extreme temperatures, and threatening foreign substances.
• The Swedish scientist, Alfred B. Nobel, left money (6. award) to people who have done
something important to help humankind.
• I didn't see anyone but I (7. watch)
• It's imperative that the wildlife program (8. broadcast) tomorrow.
• The President (9. deliver) a speech, but in the end he (10. change) his mind.
• He has five outstanding students, each (11. deserve) (12. give) a scholarship.
• I can't understand why he (13.be) so selfish. He isn't usually like that. I hope you don't
mind my mentioning this but some rumors (14. circulate) about your engagement to a
banker.
• I'm sorry I wasn't in when you came round. I would like (15. see) you.
• Canning is a method of preserving food over (16. extend) periods of time. The process
involves (17. seal) food in containers and heating it in order to kill bacteria that could
eventually cause spoilage. While most canned food is produced by commercial companies, some
(18. do) at home.
• Homegrown fruits and vegetables such as apples and potatoes are the most popular foods (19.
can) . Even certain kinds of meats are suitable for canning. However, it is not advisable to
heat produce such as avocados because of the changes in taste and texture that occur, Other
foods, including cucumbers and peppers, can be canned only if they are first pickled and then
cooked at very low temperatures. Properly canned food can be saved for as long as three years.
If not properly (20. seal) the food can be spoiled by the growth of organisms. In
severe cases, bacteria can cause a fatal form of poisoning called botulism. For this reason, it is
very important to check the seal of the jar or bottle regularly to make sure it has remained
undisturbed.
PART II. PREPOSITIONS & PHRASAL VERBS (20PTS)
A. Complete each of the following sentences with a suitable preposition.
• Two months ago, I put (1) for a job as a dental receptionist.
• Mr. Brown tried (2) several cell phones before finding one entirely (3) his taste.
• The management has decided to lay (4) 50 employees.

6|Page Tài liệu khóa học Live - VIP


OTTO CHANNEL
Otto Channel
• He turned (5) for his appointment with the supervisor.
• Look! I'm sorry to butt (6) but I think I can help.
• We are (7) no obligation to change good which are not purchased here.
• It's not my fault. Don't take it (8) on me.
• In this season, most of the people in my area have gone (9) with flu.
• Everyone was eager to get a copy the day the book came (10) .
B. Complete the following passage with prepositions.
Many flowering plants woo insect pollinators and gently direct them to their most fertile blossoms
by changing the color of individual flowers from day to day. Through color cues, the plant signals (11)
the insect that it would be better (12) visiting one flower on its bush than another. The
particular hue tells the pollinator that the flower is full of far more pollen than are neighboring blooms.
That nectar-rich flower also happens to be fertile and ready to disperse its pollen or to receive pollen
the insect has picked (13) from another flower. Plants do not have to spend precious resources
maintaining reservoirs (14) nectar in all their flowers. Thus, the color-coded communication
system benefits both plants and insect.
For example, on the lantana plant, a flower starts (15) on the first day as yellow, when it is rich
with pollen and nectar. Influenced by an as-yet-unidentified environmental signal, the flower changes
color (16) triggering the production of the pigment erythromycin. It turns orange on the second
day and red on the third. By the third day, it has no pollen to offer insects and is no longer fertile. On
any given lantana bush, only 10 to 15 percent of the blossoms are likely to be yellow and fertile. But
(17) tests measuring the responsiveness of butterflies, it was discovered that the insects
visited the yellow flowers at least 100 times more than would be expected (18) haphazard
visitation. Experiments with paper flowers demonstrated that the butterflies were responding (19)
color cues rather than, say, the scent of the nectar.
In other types of plants, blossoms change from white to red, others from yellow to red, and so on.
These color changes have been observed (20) some families of plants.
PART III. WORD FORMS (20PTS)
A. Complete the sentences with the correct forms of the given words.
1. The brochures displayed very tempting photographs of the tourist attractions in Sweden. They
were so that it was hard for viewers to resist booking that tour immediately. (MOUTH)
2. What must be avoided at all costs is the suppression of anger, as feelings of resentment can lead
a relationship to break down . (RETRIEVE)
3. She is so that she won't let anything stand in her way of her ambition. (MIND)
4. Tourists can see many views of the ocean and mountain. (PANORAMA)
5. Though they are students, their made a good impression on the local audience. (THEATER)
6. The islands have been by the growth of tourism. (WEST)
7. Dolphins, species, sometimes jump above the surface of the water. (MAMMAL)

7|Page Tài liệu khóa học Live - VIP


OTTO CHANNEL
Otto Channel
8. We like Mary. She's very nice and . (LADY)
9. The Vietnamese people are happy to talk about their past and show an amazing resilience and
. (FORGIVE)
10. He is a bad manager in that factory and everyone is in an attempt to him. (FAME)
B. Complete the following passage with the correct forms of the given words.

coincidence solidify detail nature worship


basis antique enlargement time bury

One of the seven wonders of the ancient world, the Great Pyramid of Giza was a monument of
wisdom and prophecy built as a tomb for Pharaoh Cheops in 2720 B.C. Despite its (11) ,
certain aspects of its construction make it one of the truly great wonders of the world. The thirteen-
acre structure near the Nile River is a (12) mass of stone blocks covered with limestone. Inside
are a number of hidden passageways and the (13) chamber for the pharaoh. It is the (14)
single structure in the world. The four sides of the pyramid are aligned almost exactly on true
north, south, east and west an incredible engineering feat. The ancient Egyptians were sun (15)
and great astronomers, so computations for the Great Pyramid were (16) on
astronomical observations.
Explorations and (17) examinations of the base of the structure reveal of many interesting lines.
Further scientific study indicates that these represent a type of (18) events - past, present and
future. Many of the events have been interpreted and found to (19) with known facts of the
past. Others are prophesied for future generations and are currently under investigation. Many believe
that pyramids have (20) powers and this one is no exception. Some researchers even associate it
with extraterrestrial being of the ancient past. Was this superstructure made by ordinary being, or one
built by a race far superior to any known today?
PART IV. OPEN CLOZE TEST (10PTS)
Fill in each of the following blanks with ONE suitable word.
Most obviously, those exposed to weekly general music classes or private instrumental or vocal
lessons will find an outlet (1) their creativity and self-expression. However, a closer, more
scientific (2) at music will show that the advantages are indeed much greater than just increased
creativity.
Research has showed that learning a musical instrument or merely learning how to read music
assists a child in develop higher (3) skills, such as problem-solving and problem-finding, analysis,
and evaluation. A child who learns to understand the aspects of reading music, (4) notation, key
signatures, and other items found on a piece of music as well as the child who develops the ability to
follow the sequence of notes, is using the same portion of the brain that is used in mathematical
thinking. Gifted musicians, it's reported, are often gifted mathematicians as well.
Those who study music diligently also develop self-discipline. The serious music student who sets
(5) time to practice each day will develop similar positive habits in other subjects. Organizational
skills are better, grades are higher, and children learn what it takes to (6) at something.

8|Page Tài liệu khóa học Live - VIP


OTTO CHANNEL
Otto Channel
Participation in group musical activities build teamwork, and students learn that working together
as a group is essential (7) the production of a good finished product. They learn to rely on
others and to be relied upon. Teamwork also promotes responsibility; i.e. if you're the only trumpet in
the band, (8) you need to show up for rehearsals no matter what!
They'll also come to understand that music is the thread that (9) them together with the
world, as cliché as it may sound. Music is indeed the universal language but it also helps children to
learn about cultural heritage, their (10) as well as others', and gives them an insight into
history.
PART V. ERROR IDENTIFICATION (10PTS)
Identify 10 errors in the following passage and correct them.
1 In US, industries that generate hazardous wastes want to dispose of them as cheap as
2 possible. Private companies hiring to dispose of this waste compete with each other to offer
3 the lowest prices to these industries. The government does not get involved, beyond setting
4 minimum safety standards.
5 Unfortunately, the goal of companies that generates and disposes of waste to save money,
6 not to guarantee safety. These companies usually send waste to landfills so this is cheaper
7 than recycling or incineration. Disposal firms who want to increase their business must cut
8 corners to lower costs and gain customer. At the same time, relatively little was done to
9 reduce the number of waste generated, because disposal costs remain relatively modest.
10 Things are different from Denmark. There the government participates in the waste
11 disposal process beginning in the front end. Together with industry, the government formed
12 a corporation to establish and operate waste disposal facilities. This company, called
13 Kommunichem, has a monopoly on waste disposal. Generators of hazardous waste must
14 ship their waste to one of Kommunichem's disposal facilities. In this system, there is no
15 price competition in the waste disposal business.

Lines Mistake Correction Lines Mistake Correction

PART VI. SENTENCE TRANSFORMATION (10PTS)


Rewrite the sentences with the given words or beginning in such a way that their meanings remain
unchanged.
1. Most of the students ignored what the teacher was saying.
→ Few ...............................................................................................................................
2. He delayed writing the book until he had done a lot of research.
→ Only when ....................................................................................................................
3. We shall not announce the decision formally.
→ No formal .....................................................................................................................
9|Page Tài liệu khóa học Live - VIP
OTTO CHANNEL
Otto Channel
4. I'd be grateful if you could have a look at these figures. (CAST)
→ .....................................................................................................................................
5. You have said exactly the right thing. (NAIL)
→ .....................................................................................................................................
6. It is not certain that Jones will get the job.
→ It is open ......................................................................................................................
7. She furiously threw the book across the room.
→ Such .............................................................................................................................
8. You'll feel much healthier if you have a relaxing break. (DO)
→ .....................................................................................................................................
9. I didn't realize how much she was influenced by her mother. (EXTENT)
→ .....................................................................................................................................
10. He owed his rescue to a passer-by. (INDEBTED)
→ .....................................................................................................................................
THE END

10 | P a g e Tài liệu khóa học Live - VIP


OTTO CHANNEL
Otto Chanel
SỞ GIÁO DỤC & ĐÀO TẠO KỲ THI OLYMPIC TRUYỀN THỐNG 30/4
TP. HỒ CHÍ MINH LẦN THỨ XXV – NĂM 2010
ĐỀ CHÍNH THỨC Môn thi: Anh văn - Khối: 10
Thời gian làm bài: 180 phút

➢ Thí sinh làm phần trắc nghiệm (MULTIPLE CHOICE) trên phiếu trả lời trắc nghiệm và phần tự
luận (WRITTEN TEST) trên phiếu trả lời tự luận.
➢ Trên phiếu trả lời trắc nghiệm, thí sinh tô thêm 2 số 00 vào trước số báo danh (bằng bút chì).
Phần mã đề thi trên phiếu trắc nghiệm, thí sinh tô vào ô 002.
 --------------------------------------------------------------------------------------------------------------------

CẤU TRÚC ĐỀ THI

SECTION A. MULTIPLE CHOICE (20PTS) ...................................................................................................... 1


PART I. PHONOLOGY (5PTS) ........................................................................................................................... 1
PART II. GUIDED CLOZE (5PTS) ...................................................................................................................... 1
PART III. READING COMPREHENSION (10PTS) ........................................................................................... 2

SECTION B. WRITTEN TEST (80PTS) .............................................................................................................. 5


PART I. VERB TENSES/FORMS (10PTS) ....................................................................................................... 5
PART II. PREPOSITIONS & PHRASAL VERBS (10PTS) .............................................................................. 6
PART III. OPEN CLOZE TEST (20PTS) ............................................................................................................. 7
PART IV. WORD FORMS (20PTS) ...................................................................................................................... 7
PART V. ERROR IDENTIFICATION (10PTS) ................................................................................................. 9
PART VI. SENTENCE TRANSFORMATION (10PTS) ...................................................................................... 9
Otto Chanel
SỞ GIÁO DỤC & ĐÀO TẠO KỲ THI OLYMPIC TRUYỀN THỐNG 30/4
TP. HỒ CHÍ MINH LẦN THỨ XXV – NĂM 2010
ĐỀ CHÍNH THỨC Môn thi: Anh văn - Khối: 10
Thời gian làm bài: 180 phút

➢ Thí sinh làm phần trắc nghiệm (MULTIPLE CHOICE) trên phiếu trả lời trắc nghiệm và phần tự
luận (WRITTEN TEST) trên phiếu trả lời tự luận.
➢ Trên phiếu trả lời trắc nghiệm, thí sinh tô thêm 2 số 00 vào trước số báo danh (bằng bút chì).
Phần mã đề thi trên phiếu trắc nghiệm, thí sinh tô vào ô 002.
 --------------------------------------------------------------------------------------------------------------------
SECTION A. MULTIPLE CHOICE (20PTS)
PART I. PHONOLOGY (5PTS)
Choose the word whose underlined part is pronounced differently from the other three.
1. A. rhinoceros B. vehicle C. whale D. uninhabitable
2. A. unconcernedly B. ragged C. sacred D. hiccupped
3. A. archaic B. archive C. choir D. archery
4. A. suggestion B. congestion C. digestion D. devotion
5. A. dairy B. lair C. fair D. gait
Choose the word whose stress pattern is different from that of the other three.
6. A. credulous B. acropolis C. dialect D. obsolete
7. A. ecotourism B. compromise C. disposal D. neighborhood
8. A. beneficial B. detrimental C. understanding D. magnificent
9. A. zoology B. cement C. conquest D. duet
10. A. argumentative B. psychological C. contributory D. hypersensitive
PART II. GUIDED CLOZE (5PTS)
Read the following passage and choose the options that best complete the blanks.
Sylvia Earle, a (11) botanist and one of the (12) deep – sea explorers, has spent over 6,000
hours, more than seven months, under water. From her earliest years, she took her first plunge into the
open sea as a teenager. In the years since then, she has taken part in a(n) (13) of landmark
underwater projects, from exploratory expeditions around the world to her celebrated “Jim dive" in 1978,
which was the deepest solo dive (14) made without cable connecting the diver to a support vessel at
the surface of the sea.
(15) in a Jim suit, a futuristic suit of plastic and metal armor, which was secured (16) a
manned submarine, Sylvia Earle plunged vertically into the Pacific Ocean, at times at the speed of 100
feet per minute. (17) reaching the ocean floor, she was released from the submarine and from that
point her only connection to the sub was an 18-foot tether.
For the next two and a half hours, Earle (18) the seabed, taking notes, collecting (19) ,
and painting a U.S. flag. Consumed by a desire to descend deeper still, in 1981 she became involved in

1|Page Tài liệu khóa học Live - VIP


OTTO CHANNEL
Otto Chanel
the design and manufacture of deep-sea (20) one of which took her to a depth of 3000 feet. This
did not end Sylvia Earle's accomplishments.

11. A. marine B. underwater C. undersea D. submarine


12. A. furthest B. foremost C. greatest D. utmost
13. A. amount B. great deal C. average D. number
14. A. really B. later C. ever D. mostly
15. A. Covered B. Put C. Clothed D. Worn
16. A. to B. with C. from D. against
17. A. In B. On C. At D. For
18. A. walked B. roamed C. dived D. strolled
19. A. specimens B. models C. remains D. debris
20. A. subcontractors B. submariners C. submersions D. submersibles
PART III. READING COMPREHENSION (10PTS)
Read the following passage and choose the best option to complete the blank or answer the question.
PASSAGE 1
Since water is the basis of life, composing the greatest part of the tissues of all living things, the
crucial problem of desert animals is to survive in a world where sources of flowing water is rare. And since
man's inexorable necessity is to absorb large quantities of water at frequent intervals, he can scarcely
comprehend that many creatures of the desert pass their entire lives without a single drop.
Uncompromising as it is, the desert has not eliminated life but only those forms unable to withstand
its desiccating effects. No moist-skinned, water-loving animals can exist there. Few large animals are
found: the giants of the North American desert are deer, the coyote, and the bobcat. Since desert country
is open, it holds more swift-footed, running, and leaping creatures than the tangled forest. Its population
is largely nocturnal, silent, filled with reticence, and ruled by stealth. Yet they are not emaciated. Having
adapted to their austere environment, they are as healthy as animals anywhere in the world.
The secret of their adjustment lies in a combination of behavior and physiology. None could survive,
if, like mad dogs and Englishmen, they went out in the midday sun, many would die in a matter of minutes.
So most of them pass the burning hours asleep in cool, humid burrows underneath the ground,
emerging to hunt only by night. The surface of the sun-baked desert averages around 150 degrees, but
18 inches down the temperature is only 60 degrees.
21. What is the topic of the passage?
A. Desert plants B. Desert life C. Animal life D. Forest life
22. The phrase 'those forms' refers to all the following except
A. water-loving animals C. moist-skinned animals
B. the bobcat D. many large animals

2|Page Tài liệu khóa học Live - VIP


OTTO CHANNEL
Otto Chanel
23. The word 'inexorable' is closest in meaning to
A. relentless B. indispensable C. full D. demanding
24. The author mentions all of the following as examples of the behavior of desert animals except
A. animals sleep during the day C. animals are noisy and aggressive
B. animals dig homes underground D. animals are watchful and quiet
25. The word 'them' refers to
A. mad dogs and Englishmen C. behavior and physiology
B. desert animals D. minutes
26. The word emaciated' is closest in meaning to
A. wild B. cunning C. unmanageable D. unhealthy
27. The author states that one characteristic of animals living in the desert is that they.
A. are smaller and fleeter than forest animals
B. are less healthy than animals living in different places
C. can hunt in temperature of 150 degrees
D. live in an accommodating environment
28. Which of the following generalizations is supported by the passage?
A. Water is the basis of life. B. All living things adjust to their environments.
C. Desert life is colorful. D. Healthy animals live longer lives.
29. The word 'burrows' is closest in meaning to
A. underground nests C. underground caves
B. underground houses D. underground holes in the desert?
30. How is the temperature 18 inches underground compared to that on the surface
A. the same B. much higher C. less than half D. half
PASSAGE 2
Read the following passage and choose the best option to complete the blank or answer the question.
Even before the turn of the century, movies began to develop in two major directions: the realistic
and the formalistic. Realism and formalism are merely general, rather than absolute, terms. When used
to suggest a tendency toward either polarity, such labels can be helpful, but in the end they are just labels.
Few films are exclusively formalist in style, and fewer yet are completely realist. There is also an important
difference realism and reality, although this distinction is often forgotten. Realism is a particular type,
whereas physical reality is the source of all the raw materials of film, both realistic and formalistic. Virtually,
all movie directors go to the photographable world for their subject matter, but what they do with this
material- how they shape and manipulate it- determines their stylistic emphasis.
Generally speaking, realistic films attempt to reproduce the surface of concrete reality with a minimum
of distortion. In photographing objects and events, the filmmaker tries to suggest the copiousness of
life itself. Both realist and formalist film directors must select (and hence emphasize) certain details from
the chaotic sprawl of reality. But the element of selectivity in realistic films is less obvious. Realists, in

3|Page Tài liệu khóa học Live - VIP


OTTO CHANNEL
Otto Chanel
short, try to preserve the illusion that their film world is unmanipulated, an objective mirror of the actual
world. Formalists, on the other hand, make no such pretense. They deliberately stylize and distort their
raw materials so that only the very naive would mistake a manipulated image of an object or event for
the real thing.
We rarely notice the style in a realistic movie; the artist tends to be self-effacing. Some filmmakers
are more concerned with what is being shown than how it is manipulated. The camera is used
conservatively. It is essentially a recording mechanism that produces the surface of tangible objects with
as little commentary as possible. A high premium is placed on simplicity, spontaneity, and directness. This
is not to suggest that these movies lack artistry, however, for at its best the realistic cinema specializes
in art that conceals art.
31. What does the passage mainly discuss?
A. Acting styles B. Film plots C. Styles of filmmaking D. Filmmaking 100 years ago
32. With which of the following statements would the author be most likely to agree?
A. Realism and formalism are outdated terms.
B. Most films are neither exclusively realistic nor formalistic.
C. Realistic films are more popular than formalistic ones.
D. Formalistic films are less artistic than realistic ones.
33. The phrase" this distinction" in the first paragraph refers to the difference between
A. formalists and realists B. realism and reality
C. general and absolute D. physical reality and raw materials
34. Whom does the author say is primarily responsible for a style of a film?
A. The director B. The actors C. The producer D. The camera operator
35. The word "it" in the first paragraph refers to
A. the photographable world B. their subject matter
C. this material D. their stylistic emphasis
36. The word “copiousness" in the second paragraph is closest in meaning to
A. abundance B. greatness C. fullness D. variety
37. How can one recognize the formalist style?
A. It uses familiar images. C. It obviously manipulated images.
B. It is very impersonal. D. It mirrors the actual world.
38. The word "tangible" in the last paragraph is closest in meaning to
A. concrete B. complex C. various D. comprehensible
39. The word "self-effacing" in the last paragraph is closest in meaning to
A. modest B. shy C. egocentric D. introverted
40. Which of the following films would most likely use a realist style?
A. A travel documentary C. A musical drama
B. A science fiction film D. An animated cartoon

4|Page Tài liệu khóa học Live - VIP


OTTO CHANNEL
Otto Chanel
SECTION B. WRITTEN TEST (80PTS)
PART I. VERB TENSES/FORMS (10PTS)
Put the verbs in the brackets in the correct forms.
1. He (go) to the last meeting, but he didn't.
2. By the time you come here again, this palace (build) .
3. The yesterday accident is thought (cause) by human error.
4. We (cook) all day for the party that evening and by 8 o'clock we still weren't ready.
5. It is highly desirable that every effort (make) to reduce expenditure.
6. (There, be) any errors, let me know.
7. His (take) ill was quite unexpected.
8. In 20 hours' time, I (relax) on my yacht.
9. It was a boring show. I would rather (not go) there.
10. It was a close call. We (kill) .

In democratic countries, any efforts (11. restrict) the freedom of the press are rightly
condemned. However, this freedom (12. easily, abuse) . Stories about people often attract far more
public attention than political events. Though we may enjoy reading about the lives of others, it is
extremely doubtful whether we would equally enjoy reading about ourselves. (13. act) on the
contention that facts are sacred, reporters can cause untold suffering to individuals by publishing details
about their private lives. Newspapers exert tremendous influence that they cannot only bring about major
changes to the lives of ordinary people but (14. even, overthrow) a government.

The story of a poor family that (15. acquire) fame and fortune overnight, dramatically
illustrates the power of the press. The family lived in Aberdeen, a small town in South Dakota. As the
parents had five children, life was a perpetual struggle against poverty. They (16. expect) their
sixth child and (17. face) with even more pressing economic problems. If they had had only one
more child, the fact would have passed unnoticed. They (18. continue) to struggle against
economic odds and would have lived in obscurity. But they suddenly became the parents of quintuplets,
four girls and a boy, an event which radically changed their lives. The day after the birth of the five
children, a plane arrived in Aberdeen (19. bring) sixty reporters and photographers. The
news was of national importance, for the poor couple (20. become) the parents of the only
quintuplets in America.

5|Page Tài liệu khóa học Live - VIP


OTTO CHANNEL
Otto Chanel
PART II. PREPOSITIONS & PHRASAL VERBS (10PTS)
A. Complete each of the following sentences with (a) suitable preposition(s) or particle(s) from the box.

in for through down under off


round out over at up to

1. I wish you wouldn't fly me like that every time I make a mistake.
2. The piece of equipment is very well made and stands the roughest treatment. You won't
have any trouble with it.
3. The new office staff are shaking well.
4. I think a sip of wine can bring him .
5. We are no obligation to change goods which were not purchased here.
6. I am not friends with Peter any more. We have fallen .
7. I knew I was the hill when I started needing glasses to read.
8. Tom's family pulled him the difficult period following his wife's death.
9. The police came a great deal of criticism.
10. I have gone computer games. They are not as interesting as before.
B. Complete the following passage with prepositions or particles.
It is not easy trying to cope with fear. Most people (11) some stage in their lives feel afraid
of something; fear of the dark as children, afraid of spiders or flying. For the most part, these fears are
normal and do not interfere (12) our ability to get on with our lives.
However, some people are afraid of something (13) such an extent that it prevents them from
leading a normal life. For example, 'electrophobia' being afraid of electricity- makes life in today's world
extremely difficult.
In fact, you can be afraid of anything and there is almost certainly a name for it. You can be afraid
of clouds, certain colors, bicycles, rain, mushrooms and even sitting down. Apparently, the list of phobia
gets longer everyday, but for people who have a real terror of something, help is (14) hand.
Researchers are making enormous progress (15) understanding what a phobia is and what makes
it come (16) .
It is surprising how many people think they suffer from a phobia when actually all they are really
experiencing is a strong dislike or distaste (17) something.
You may think you are computer phobic and want to throw your machine out of the nearest window.
But that is not the same as being really 'mechanophobic', suffering from a racing heart and being short
of breath (18) the mere sight of a computer.
Experts say that you cannot take a true phobia (19) anything else as it affects the whole of your
nervous system. If you cannot run away from whatever is causing the fear, you feel that is inevitable. On

6|Page Tài liệu khóa học Live - VIP


OTTO CHANNEL
Otto Chanel
the other hand, it's natural for most people to be afraid if they are aboard an airplane which is flying into
a storm. Most psychologists agree that phobias can be described (20) three main ways: social
phobias, panic disorders and specific phobias.
PART III. OPEN CLOZE TEST (20PTS)
Fill each blank with ONE word.
PASSAGE 1
Although noise, commonly (1) as unwanted sound, is a widely recognized form of pollution, it
is very difficult to measure because the discomfort (2) by different individuals is highly subjective
and, therefore, variable. Exposure to lower levels of noise may be (3) irritating, whereas exposure
to higher levels may actually cause (4) loss. Particularly in congested urban areas, the noise produced
(5) a by-product of our advancing technology causes physical harm, and detracts from the quality of
life for those who are exposed to it.
Noise causes effects that the hearer cannot control and to (6) the body never becomes accustomed.
Loud noises instinctively signal danger (7) any organism with a hearing mechanism, including
human beings. (8) response, heartbeat and respiration accelerate, blood vessels constrict, the
skin pales, and muscles tense. In fact, there is a general increase in functioning (9) about by the
flow of adrenaline release in response to fear, and some of these responses persist even longer than the
noise, occasionally as long as 30 minutes (10) the sound has ceased.
PASSAGE 2
Unlikely (11) it may seem, there has now been expert confirmation that wild pumas and
lynxes are at (12) in parts of Britain, rather than being the figments of some wild imaginations. Previous
sightings of such large cats had been put (13) to exaggeration. After all, the argument went,
some people are prone to seeing flying saucers and Loch Ness monsters, particularly when under the
influence of one drink. Some newspapers were suspected of having made up stories such as (14) of
the Beast of Exmoor, an animal that is responsible for the deaths of hundreds of sheep over the past ten
years. (15) experts have now come up with proof that such stories were in earnest after all. The
animals are in all (16) pets which have escaped from small zoos, or been abandoned by their owners.
Because the keeping of such animals is severely restricted under the (17) of the Dangerous Wild
Animal Act of 1976, owners of unlicensed animals might not (18) an escape for fear of prosecution.
Britain's only surviving native species, the wild cat, is confined to Scotland. After examining hair samples,
experts now say that the Best of Exmoor in the south of England is without (19) a puma or lynx,
both of which are normally native (20) the Middle East and Asia.
PART IV. WORD FORMS (20PTS)
A. Complete the sentences with the correct forms of the given words.
1. His (speak) annoys some members of the committee because he is always expressing his ideas
frankly.
2. George is very disorganized and not very (business) .

7|Page Tài liệu khóa học Live - VIP


OTTO CHANNEL
Otto Chanel
3. She was fully aware of her own (short)
4. Low income and little administrative support make teachers (heart) with their profession.
5. A film about the (reptile) ancestors is available in the library.
6. There are people whose (mortal) begins from the moment of their death.
7. It's undeniable that the (diagnose) of the local incompetent healer was responsible for her
sudden death.
8. As the sole (benefit) of his uncle's will, he inherited a huge fortune.
9. This (repair) faulty washing machine should be returned to the manufacturer.
10. The first time I tried out my new bike I (balance) and fell off.
B. Complete the following passage with the correct forms of the given words.
band professor sale acquisition starry
lonely quarterly popularity invade origin

The Beatles became the most popular grouping rock music history. The (11) of
extraordinarily talented musicians generated a frenzy that transcended countries and economic strata.
While all of them sang, John Lennon and Paul McCartney wrote the majority of their songs. (12) ,
Lennon and five others formed a group called Quarrymen in 1956, with McCartney joining them later that
year. George Harrison, John Lennon and Paul McCartney, together with Stuart Sutcliffe, who played the
bass guitar, and Pete Best on the drums, performed together in several bands for a few years, until they
finally settled on the Silver Beatles in 1960. American Rock musicians, such as Chuck Berry and Elvis
Presley, influenced Lennon's and McCartney's music, whose first hits consisted of simple tunes and lyrics
about young love. The Beatles' U.S. tour propelled them to (13) and led to two movies filmed
in 1964 and 1965.
The so-called British (14) of the United States was in full swing when they took the top
five spots on the singles' charts, followed by the release of their first film.
During the 1960s, their music matured and (15) a sense of melody. The lyrics of their song became
deeper and gained in both imagination and meaning. Their popularity continued to grow as the Beatles
turned their attention to social problems and political issues in "Nowhere Man" and Eleanor Rigby". (16)
and nostalgia come through in their ballad "Michelle" and 'Yesterday", which fully displayed the
group's (17) development and sophistication. Lennon's sardonic music with lyrics written in the
first person, and McCartney's songs that created scenarios with off beat individuals, contributed to the
character of the music produced by the group. In addition to their music, the Beatles set a social trend
that (18) long hair, Indian music, and moss dress.
For a variety of reasons, the musicians began to drift apart, and their last concert took place in San
Francisco in 1966. The newspapers and tabloids publicized their quarrels and lawsuits, and the much
idolized group finally (19) in 1970. However, their albums had (20) those of any other
bands in history. Although all of the Beatles continued to performed solo or form new rock alone, none
could achieve the recognition and success that they had been able to win together.

8|Page Tài liệu khóa học Live - VIP


OTTO CHANNEL
Otto Chanel
PART V. ERROR IDENTIFICATION (10PTS)
Identify 10 errors in the following passage and correct them.
1 Between 1977 and 1981, three groups of American women, numbered 27 in all, between the
2 age of 35 and 65, were given month-long tests to determine how they would response to
3 conditions resembling those on the space shuttle. Carefully selected from many applicants,
4 the women were volunteers and pay was barely above the minimum wage. They weren't
5 allowed to smoke or drink alcohol during the tests, and they were expected to tolerate each
6 other's company at closed quarters for the entire period. Among other things, they had to
7 stand pressure three times of the force of gravity and carry out both physical and mental tasks
8 while exhausted from strenuous physical exercises. At the end of ten days, they had to spend
9 a further twenty days absolutely confined to bed, during that time they suffered backaches
10 and discomforts, and when they were finally allowed up, the more physically active women
11 were especially subject to pains due to a light calcium loss. Results of the tests suggest that
12 women have significant advantages on men in space. They need less food and less oxygen
13 and they stand up to radiation well. Men's advantages in terms of strength and stamina,
14 meanwhile, are virtually wiped out by the zero-gravity condition in space.

Lines Mistake Correction Lines Mistake Correction

PART VI. SENTENCE TRANSFORMATION (10PTS)


Rewrite the sentences with the given words or beginning in such a way that their meanings remain
unchanged.
1. The only reason the party was a success was that a famous film star attended it.
→ Had it ...........................................................................................................................
2. Government guidelines really do emphasize the importance of starting education early.
→ A lot of emphasis ..........................................................................................................
3. The trip was so amazing that we will never forget it.
→ It's too .........................................................................................................................
4. Your silly questions distracted me.
→ You drove .....................................................................................................................
5. She was so disgusted at the way her friend behaved that she refused to speak to him.
→ Such .............................................................................................................................
6. Mike is never reluctant to make tough decisions as a manager. (SHRINKS)
→ ....................................................................................................................................

9|Page Tài liệu khóa học Live - VIP


OTTO CHANNEL
Otto Chanel
7. The film is similar to Shakespeare's Hamlet in a number of ways. (RESEMBLANCE)
→ The film ........................................................................................................................
8. He is determined to become a doctor. (HEART)
→ .....................................................................................................................................
9. Alison bought the big house because she wanted to open a hotel. (VIEW)
→ Alison bought ................................................................................................................
10. We feel uncomfortable in the house. (FISH)
→ .....................................................................................................................................

THE END

10 | P a g e Tài liệu khóa học Live - VIP


OTTO CHANNEL
Otto Channel
SỞ GIÁO DỤC & ĐÀO TẠO KỲ THI OLYMPIC TRUYỀN THỐNG 30/4
TP. HỒ CHÍ MINH LẦN THỨ XXV – NĂM 2011
ĐỀ CHÍNH THỨC Môn thi: Anh văn - Khối: 10
Thời gian làm bài: 180 phút

➢ Thí sinh làm phần trắc nghiệm (MULTIPLE CHOICE) trên phiếu trả lời trắc nghiệm và phần tự
luận (WRITTEN TEST) trên phiếu trả lời tự luận.
➢ Trên phiếu trả lời trắc nghiệm, thí sinh tô thêm 2 số 00 vào trước số báo danh (bằng bút chì).
Phần mã đề thi trên phiếu trắc nghiệm, thí sinh tô vào ô 002.
 --------------------------------------------------------------------------------------------------------------------

CẤU TRÚC ĐỀ THI


SECTION A. MULTIPLE CHOICE (40PTS) ......................................................................... 1
PART I. PHONOLOGY (5PTS) ..................................................................................................................... 1
PART II. VOCABULARY & STRUCTURE (10PTS) .................................................................................... 1
PART III. GUIDED CLOZE TEST (5PTS) ..................................................................................................... 2
PART IV. READING COMPREHENSION (20PTS) ..................................................................................... 3

SECTION B. WRITTEN TEST (60PTS) ............................................................................... 6


PART I. WORD FORMATION (20PTS) ..................................................................................................... 6
PART II. PREPOSITIONS AND PHRASAL VERBS (10PTS) ................................................................... 7
PART III. OPEN CLOZE TEST (10PTS) ....................................................................................................... 7
PART IV. ERROR IDENTIFICATION (10PTS) ........................................................................................... 8
PART V. SENTENCE TRANSFORMATION (10PTS) ................................................................................ 8
Otto Channel
SỞ GIÁO DỤC & ĐÀO TẠO KỲ THI OLYMPIC TRUYỀN THỐNG 30/4
TP. HỒ CHÍ MINH LẦN THỨ XXV – NĂM 2011
ĐỀ CHÍNH THỨC Môn thi: Anh văn - Khối: 10
Thời gian làm bài: 180 phút

➢ Thí sinh làm phần trắc nghiệm (MULTIPLE CHOICE) trên phiếu trả lời trắc nghiệm và phần tự
luận (WRITTEN TEST) trên phiếu trả lời tự luận.
➢ Trên phiếu trả lời trắc nghiệm, thí sinh tô thêm 2 số 00 vào trước số báo danh (bằng bút chì).
Phần mã đề thi trên phiếu trắc nghiệm, thí sinh tô vào ô 002.
 --------------------------------------------------------------------------------------------------------------------
SECTION A. MULTIPLE CHOICE (40PTS)
PART I. PHONOLOGY (5PTS)
Choose the word which has the underlined part pronounced differently from the rest
1. A. ban B. inflation C. endangered D. landscapes
2. A. within B. fathom C. anthem D. with
3. A. brood B. broomstick C. foolscap D. brooch
4. A. contribute B. syndrome C. home D. microphone
5. A. mosaic B. conserve C. reserve D. poison
Choose the word that is stressed differently from the others in the list.
6. A. diversity B. amphibians C. ecotourist D. courageous
7. A. control B. severe C. install D. moonlight
8. A. hurriedly B. apartheid C. preparatory D. determine
9. A. humpback B. strengthen C. reduce D. rescue
10. A. mysterious B. preferential C. modernity D. historical
PART II. VOCABULARY & STRUCTURE (10PTS)
Choose the best options to complete the following sentences.
11. People in financial difficulties sometimes fall to unscrupulous money lenders.
A. prey B. fool C. scapegoat D. sacrifice
12. He was caught using forged bank notes to pay for goods and charged with .
A. deception B. fraud C. embezzlement D. theft
13. Don't stick your elbows out when you eat. them in by your sides.
A. Bend B. Place C. Tuck D. Turn
14. I was in no way prepared for the of criticism my play received.
A. onslaught B. onset C. offensive D. assault
15. She affection from her children but they neglected her shamefully.
A. yearned B. craved C. hungered D. desired
16. I've got such a headache that I can't concentrate on the lecture.
A. beating B. drumming C. hammering D. throbbing
17. Race relations in this country are unlikely to improve until people overcome their
feelings of hostility towards foreigners.
A. interior B. internal C. inverted D. innate
1|Page Tài liệu khóa học Live - VIP
OTTO CHANNEL
Otto Channel
18. I utterly your argument. In my opinion, you have distorted the facts.
A. confound B. refute C. dispute D. decline
19. I offer you my most apologies for offending you as I did.
A. repentant B. servile C. abject D. candid
20. When facing problems, it is important to keep a sense of .
A. proportion B. introspection C. relativity D. comparison
21. All the way along the winding street .
A. he came B. came he C. did he come D. comes he
22. , before, his first performance for the amateur dramatic group was a success.
A. Though having never acted C. As he had never acted
B. Despite he had never acted D. In spite of his never having acted
23. There's no point in telephoning him. He's certain by now.
A. to leave B. to have left C. left D. having left
24. The bank is reported in the local newspaper in broad daylight.
A. to be robbed B. robbed C. to have been robbed D. having been robbed
25. "Eric is really upset about losing his job." "Well, once myself, I can understand."
A. having been fired B. Fired C. having fired D. being fired
26. Clothing made of plastic fibers has certain advantages over made of natural fibers like cotton,
wool, or silk.
A. what B. the one C. that D. which
27. I'm not by a particularly ambitious man.
A. inclination B. habit C. character D. tendency
28. The government would be forced to use its emergency powers further rioting to occur.
A. should B. did C. were D. had
29. It is essential to be on the for any signs of movement in the undergrowth since there
are poisonous snakes in the area.
A. guard B. care C. alarm D. alert
30. , he remained optimistic.
A. Though badly wounded he was C. As he was badly wounded
B. Badly wounded as he was D. As badly wounded he was
PART III. GUIDED CLOZE TEST (5PTS)
Read the text below and decide which answer (A, B, C or D) best fits each space.
Whenever we read about the natural world nowadays, it is generally to be given dire predictions
about its imminent destruction. Some scientists go so far as to assert that from now on, the world can
no longer be called “natural", in so far as future processes of weather, (31) and all the
interactions of plant and animal life will no longer carry on in their time honored way, unaffected by
(32) . There will be never such a thing as "natural weather" again, say such writers, only weather

2|Page Tài liệu khóa học Live - VIP


OTTO CHANNEL
Otto Channel
manufactured by global warming. It is hard to know whether to believe such (33) of doom,
possibly because what they are saying seems to terrible to be (34) .
There are other equally influential scientists who argue that climate, for example, has changed
many times over the (35) , and that what we are experiencing now may simply be part of an
endless cycle of change, rather than a disaster on a global (36) . One cannot help wondering
these attempt to wish the problem be away simply underline the extent to which western industrialized
countries are to blame for upsetting the world's (37) . It is not our fault; they seem to be
saying, because everything is all right, really! One certain (38) which is chilling in its implications,
is that there is no longer anywhere on the earth's (39) whether in the depths of the oceans or
in the polar wastes, which is not (40) by polluted air or littered, with empty cans and bottles. Now
we are having to come to terms with understanding just what that means, and it is far from easy.
31. A. change B. atmosphere C. climate D. even
32. A. beings B. man C. people D. humans
33. A. prophets B. champions C. warriors D. giants
34. A. stopped B. true C. guessed D. here
35. A. top B. again C. centuries D. world
36. A. sense B. form C. scale D. existence
37. A. future B. ecology C. balance D. population
38. A. fact B. must C. fault D. and
39. A. planet B. atmosphere C. anywhere D. surface
40. A. full B. stained C. breathing D. only

PART IV. READING COMPREHENSION (20PTS)


Read the passage and choose the best answer for each of the questions below.
PASSAGE 1
Since the 20th century, magazines have been a major growth area of popular publishing. Specialist
magazines cater to every imaginable field and activity. In the United Kingdom, over 12,000
periodicals, magazines, bulletins, annuals, trade journals and academic journals are published on a
regular basis. There are some 40 women's magazines and over 60 dealing with particular sports, games,
hobbies, and pastimes. Although some U. S. magazines such as The Saturday Evening Post, has
succumbed to the competition of television, many continue to have enormous international
circulations, The Reader's Digest over 16 million, The National Geographic over 10 million. For many
people, magazines have been the most available and widely used form of continuing education,
providing information about history, geography, literature, science, and the arts, as well as guidance
on gardening, cooking, home decorating, financial management, psychology, even marriage and family
life.
Until the rise of television, magazines were the most available form of cheap, convenient
entertainment in the English-speaking world. Radio served a similar function, but it was more limited
in what it could do. Magazines and television, however, both address the more powerful visual sense.
During the third quarter of the 20h century, coincident with a dramatic rise in the popularity of
television, many general-interest, especially illustrated magazines went out of business. The shift in

3|Page Tài liệu khóa học Live - VIP


OTTO CHANNEL
Otto Channel
attention of a mass audience from reading such magazines to watching television has been a major
factor in this decline, but it is an implicit tribute from television to the older genre that its programs
are generally organized in a single format and content.
41. The word "it" in bold refers to
A. television B. publishing C. entertainment D. radio
42. From the passage, it can be inferred that
A. movies have replaced magazines B. the author is fond of magazines
C. almost all magazines are printed in English D. home decorating magazines are dramatic
43. According to the passage, which of the following magazines is no longer printed?
A. The Saturday Evening Post C. The Nation
B. The Reader's Digest D. The National Geographic
44. The word “circulations" in bold is closest in meaning to .
A. the number of blood banks selling magazines B. the number of readers of a magazine
C. the number of international magazines D. the number of sold copies of a magazine
45. The passage implies that magazines .
A. are less visual than radio C. influence television programs
B. put television out of business D. have a limited range of subjects
46. Which of the following does the author describe as limited in what it could do?
A. radio B. magazines C. movies D. television
47. The word “succumbed" in bold means .
A. set up for B. brought up for C. taken up by D. given in to
48. The passage mainly discusses .
A. the rise and fall of the radio business
B. the growth and decline of magazines in the 20h century
C. magazines and continuing education
D. the decline of international circulation
49. The phrase “every imaginable field" in bold is closest in meaning to .
A. all imaginary fields in stories and poems
B. all images in a camera's field of vision
C. all professions that one can think of
D. all trade journals about farming and psychology
50. What does the author say about mass audiences?
A. They have little influence on communications in the 20th century.
B. They have gone out of business.
C. They get information about gardening and psychology from radio.
D. They have shifted their attention from magazines to television.

4|Page Tài liệu khóa học Live - VIP


OTTO CHANNEL
Otto Channel
PASSAGE 2
The ruined temples of Angkor are perhaps one of the most impressive Seven Wonders of the World.
Located in modern day Cambodia near Lake Tonle Sap, the largest freshwater lake in Asia, Angkor was
the seat of power for the Khmer Empire from the ninth to the fifteenth century. The ruins of Angkor
are documented as some of the most impressive ones in the world, rivaling the pyramids of Giza in
Egypt. Why this mighty civilization died out is a question that archeologists are now only beginning to
ponder. The answer, it turns out. may be linked with the availability of fresh water.
One possible explanation for the downfall of the Khmer Empire has to do with the inhabitant's
irrigation system. The temples and palaces of Angkor were constructed around a series of artificial
reservoirs and canals which were annually flooded to capacity by the Mekong River. Once filled, they
were used to irrigate the surrounding paddies and farmland during the course of the year. Farmers
were completely dependent on the water for their crucial rice crop. Without consistent irrigation, the
farmers would have been unable to maintain functional crop production.
Scientists speculate that toward the end of the Khmer Empire the hydraulic system of the reservoirs
and canals broke down. The construction of hundreds of sandstone temples and palaces required an
enormous amount of physical labor. In addition, as the capital of the Khmer Empire, Angkor contained
upwards of one hundred thousand people who resided in and around Angkor. In order to feed so many
people, the local farmers were driven to grow food more quickly and more efficiently. After centuries
of continual use, the irrigation system was pushed beyond its capacity. Soil erosion, nutrient depletion,
and loss of water led to decrease in the food supply. With less food available, the people of Angkor
slowly began to migrate to other parts of Cambodia thus leaving the marvelous city of Angkor to be
swallowed by the jungle. Therefore, it is speculated that the Khmer Empire may have been fallen victim
to its own decrepit infrastructure.
51. What is the passage mainly about?
A. Modern day agricultural procedures in Cambodia
B. Religious temples of the ancient Khmer Empire
C. A possible explanation for the decline of a civilization
D. The essential role water plays in farming
52. The passage preceding most likely discusses
A. architecture of ancient Asian civilization
B. religious practices of the people of Angkor
C. the form of government practiced by the Khmer Empire
D. the other six wonders of the world
53. According to the passage, Lake Tonle Sap in Cambodia .
A. was unable to supply fish for the people of Angkor
B. is one of the Seven Wonders of the world
C. is an enormous body of fresh water in Asia
D. became polluted due to a population explosion
54. The word “seat" in the passage is closest in meaning to
A. battle B. summit C. location D. chief

5|Page Tài liệu khóa học Live - VIP


OTTO CHANNEL
Otto Channel
55. The hydraulic system of reservoirs .
A. supplied irrigation from the Indian Ocean
B. helped transport the sandstone for constructing temples
C. were destroyed by nearby warrior's tribes
D. became non-functional due to overuse
56. The word “artificial" in the passage is closest in meaning to
A. man-made B. numerous C. natural D. insincere
57. The word "they" in the passage refers to
A. reservoirs and canals B. rice paddies
C. temples and palaces D. farmland
58. It can be inferred from the passage that the Khmer Empire
A. supplemented their diets with meat hunted in the nearby jungles
B. were intentionally starved by the farmers
C. lost their food source due to excess rainfall
D. depended upon rice as their main source of food
59. All of the following are mentioned as events that can affect food supply EXCEPT
A. reduction of nutrients B. contamination of soil
C. loss of water supply D. erosion of soil
60. The word "decrepit" in the passage is closest in meaning to .
A. incomplete B. deteriorated C. beneficial D. disorganized

SECTION B. WRITTEN TEST (60PTS)


PART I. WORD FORMATION (20PTS)
A. Use the correct form of the verbs in brackets.
Man has made great strides in all the fields of science, particularly medicine. For instance,
research work in the laboratories at last (1. conquer) poliomyelitis, one of the most
devastating diseases. Although the Salk vaccine (2. not be) one hundred percent effective, it
(3. decrease) the cases of polio considerably.
Tuberculosis once (4. know) as the white plague (5. study) intensively. As a matter of
fact, it is curable if it is detected in its early stages. We still have cancer (6. deal) with, but
research workers and doctors all over the world are striving to find a way to prevent and cure it. (7.
Judge) from past experience we can expect that encouraging news (8. issue) from time to time.
Heart disease, the greatest killer of mankind, is now in the process of being overcome. An
example of the techniques that (9. develop) is that of heart massage: when a heart stops
nowadays while the patient is under anesthesia, the doctor opens the chest, massages the heart and
revises the patient. Even a few years ago, such an operation would (10. be) inconceivable.
B. Complete the following sentences with the correct form of the words in brackets.
1. My daughter left a half orange on the table. (EAT)
2. Most people who work feel that they are . (PAY)

6|Page Tài liệu khóa học Live - VIP


OTTO CHANNEL
Otto Channel
3. The teacher warned the children that if they again, they would be punished. (BEHAVE)
4. Are all those they put in food really necessary? (ADD)
5. The manager handed in his after being accused of dishonesty. (RESIGN)
6. A damage was caused by the earthquake. (WIDE)
7. Phil was sentenced to seven-year for his part in the armed robbery. (PRISON)
8. I was late because I how much lime I will need. (ESTIMATE)
9. How can you the fact that some people live in mansions while others live in slums?
(JUST)
10. Site is so that she won't let anything stand in the way of her ambition. (MIND)

PART II. PREPOSITIONS AND PHRASAL VERBS (10PTS)


Complete each of the following sentences with a suitable preposition(s) or particle(s).
1. Although Mark said that he'd be there at 8. 00, he didn't turn until 10.30.
2. She takes her father; she has the same gestures and mannerisms.
3. He said he would make me a rich man, but I saw him immediately.
4. We've run coffee. Could you go and buy some?
5. We put a sum of money each month for our summer holidays.
6. He looks his older brother and follows his example in everything.
7. She had to cancel her holiday when she went the flu.
8. I don't know if she'll get her husband's death.
9. He was told to cut sugar and fats or he would suffer serious health problems.
10. We don't know yet how we'll solve the problem but I'm sure someone will come a solution
soon.

PART III. OPEN CLOZE TEST (10PTS)


Fill each blank with ONE word.
Many of the countries surrounding the Pacific Ocean have helped to create an economic (1)
that has become known as the Pacific Rim.
In 1944 the geographer N. J. Spykman published a theory about the "rim" of Eurasia. He
proposed that the control of the rim land, as he called it, would effectively (2) control of
the world. Now, more than fifty years later we can see that part of his theory holds (3) since the
power of the Pacific Rim is quite extensive.
The Pacific Rim includes countries (4) the Pacific Ocean from North and South
America to Asia to Oceania. Most of these countries have experienced major economic change and
growth to become components of an economically (5) trade region.
Raw material and finished goods are shipped between Pacific Rim states for manufacture, (6)
and sale.

7|Page Tài liệu khóa học Live - VIP


OTTO CHANNEL
Otto Channel
The Pacific Rim continues to gain strength in the global economy. From the (7) of the
Americans to just a few years ago, the Atlantic Ocean had been the leading ocean for the shipment of
goods and material. Since the early 1990s, the (8) of goods crossing the Pacific Ocean has been
greater than that of goods crossing the Atlantic. Los Angeles is the American leader in the Pacific Rim
as it is the source for the most (9) flights and ocean- based (10) . Additionally, the value of
United States imports from Pacific Rim countries is greater than that of the imports from NATO (North
Atlantic Treaty Organization) members in Europe.

PART IV. ERROR IDENTIFICATION (10PTS)


Identify 10 errors in the following passage and correct them.
1 Quotations and sayings are part of our language and our way of life. As the poet Emerson
2 says, we use them by necessity, to remind ourselves to look before we leap or avoid crossing
3 our bridges before we come to them. We use them in habit, often not realize we are doing
4 so, and we all love to use an apt quotation to live conversation or score a point in an
5 argument. This booklet contains over a thousand quotations, proverbs and sayings.
6 Altogether, they offer a great deal of information, advice, amusement and comfort. Emerson
7 writes "I hate quotations" so it is undoubtful that he would have used this book – but we
8 hope that you will, That you use it to improve your knowledge, as an aid to solve crossword
9 puzzles, to enrich your own speech or simple for idle reading, in your spare time, it will put
10 you on touch with some of the cleverest minds of the past and present. Happy reading and
11 happy quoting!

Lines Mistake Correction Lines Mistake Correction

PART V. SENTENCE TRANSFORMATION (10PTS)


Rewrite the sentences with the given words or beginning in such a way that their meanings remain
unchanged.
1. The government shouldn't let this situation get worse.
→ This state .....................................................................................................................
2. The completion of the work was scheduled for last week.
→ The work was ...............................................................................................................
3. She is now leading a normal life as a result of all the support she received from social workers. –
→ Had ..............................................................................................................................
4. Just thinking about his face at that moment makes me laugh.
→ The very .......................................................................................................................

8|Page Tài liệu khóa học Live - VIP


OTTO CHANNEL
Otto Channel
5. He declared his disapproval of the behavior of some of his supporters.
→ He let ...........................................................................................................................
6. Everybody wants Pauline as an after-dinner speaker. (DEMAND)
→ .....................................................................................................................................
7. I don't think you mean what you say about helping me. (EARNEST)
→ .....................................................................................................................................
8. At first the new computer made me feel a bit afraid. (AWE)
→ .....................................................................................................................................
9. William decided that an actor's life was not for him. (CUT)
→ .....................................................................................................................................
10. I felt vaguely that something was wrong, but what was it? (BACK)
→ .....................................................................................................................................

THE END

9|Page Tài liệu khóa học Live - VIP


OTTO CHANNEL
Otto Channel
SỞ GIÁO DỤC & ĐÀO TẠO KỲ THI OLYMPIC TRUYỀN THỐNG 30/4
TP. HỒ CHÍ MINH LẦN THỨ XXV – NĂM 2012
ĐỀ CHÍNH THỨC Môn thi: Anh văn - Khối: 10
Thời gian làm bài: 180 phút

➢ Thí sinh làm phần trắc nghiệm (MULTIPLE CHOICE) trên phiếu trả lời trắc nghiệm và phần tự
luận (WRITTEN TEST) trên phiếu trả lời tự luận.
➢ Trên phiếu trả lời trắc nghiệm, thí sinh tô thêm 2 số 00 vào trước số báo danh (bằng bút chì).
Phần mã đề thi trên phiếu trắc nghiệm, thí sinh tô vào ô 002.
 -----------------------------------------------------------------------------------------------------------------

CẤU TRÚC ĐỀ THI


SECTION A. MULTIPLE CHOICE (30PTS) ............................................................................. 1
PART I. PHONOLOGY (5PTS) .......................................................................................................................... 1
PART II. VOCABULARY AND STRUCTURE (10PTS).................................................................................... 1
PART III. GUIDED CLOZE TEST (5PTS) .......................................................................................................... 2
PART IV. PARAGRAGH UNDERSTANDING (5PTS)....................................................................................... 3
PART V. READING COMPREHENSION (5PTS) ............................................................................................. 5
SECTION B. WRITTEN TEST (70PTS) ................................................................................... 7
PART I. VERB TENSES / FORMS (20PTS).................................................................................................... 7
PART II. WORD FORMS (20PTS) ..................................................................................................................... 8
PART III. GAP-FILLING (5PTS) .......................................................................................................................... 9
PART IV. PREPOSITIONS & PHRASAL VERBS (5PTS) .............................................................................. 10
PART V. ERROR IDENTIFICATION (10PTS) .............................................................................................. 10
PART VI. SENTENCE TRANSFORMATION (10PTS) ................................................................................... 11
Otto Channel
SỞ GIÁO DỤC & ĐÀO TẠO KỲ THI OLYMPIC TRUYỀN THỐNG 30/4
TP. HỒ CHÍ MINH LẦN THỨ XXV – NĂM 2012
ĐỀ CHÍNH THỨC Môn thi: Anh văn - Khối: 10
Thời gian làm bài: 180 phút

➢ Thí sinh làm phần trắc nghiệm (MULTIPLE CHOICE) trên phiếu trả lời trắc nghiệm và phần tự
luận (WRITTEN TEST) trên phiếu trả lời tự luận.
➢ Trên phiếu trả lời trắc nghiệm, thí sinh tô thêm 2 số 00 vào trước số báo danh (bằng bút chì).
Phần mã đề thi trên phiếu trắc nghiệm, thí sinh tô vào ô 002.
 -----------------------------------------------------------------------------------------------------------------
SECTION A. MULTIPLE CHOICE (30PTS)
PART I. PHONOLOGY (5PTS)
Choose the word whose underlined part is pronounced differently from the other three.

1. A. canal B. casino C. canary D. canon


2. A. teenage B. dosage C. voyage D. carriage
3. A. ecosystem B. knowledge C. technology D. commodity
4. A. counterfeit B. courtesy C. drought D. ouster
5. A. danger B. landscape C. hand D. nature
Choose the word whose stress pattern is different from that of the other three.
6. A. prerequisite B. necessity C. European D. synonymous
7. A. compromise B. correspond C. dominate D. educate
8. A. metropolis B. descendant C. impetus D. perpetuate
9. A. vigorous B. scandalous C. victorious D. dangerous
10. A. necessary B. infamous C. automobile D. technique

PART II. VOCABULARY AND STRUCTURE (10PTS)


Choose the best answer
11. He tries to himself with everyone by paying them compliments.
A. gratify B. please C. ingratiate D. commend
12. Assembly lines are useful for producing a large of identical products.
A. quality B. quantity C. quandary D. qualification
13. Everyone knows about pollution problems, but not many people have any solutions.
A. thought over B. come up with C. looked into D. got round to
14. Mr. Nixon refused to answer the questions on the that the matter was confidential.
A. reasons B. excuses C. grounds D. foundations
15. The VCTV tries to for all tastes with its 4 national programs.
A. cater B. suit C. furnish D. regard
16. When his alarm went off, he shut it off and slept for 15 minutes.
A. other B. others C. another D. the others

1|Page Tài liệu khóa học Live - VIP


OTTO CHANNEL
Otto Channel
17. Every woman who has enough criteria can join the beauty contest irrespective of their background.
A. regardless of B. can have gone C. must have been D. was
18. , dolphins have no sense of smell.
A. As known as far B. As far as is known
C. It is known as far D. Known as far as it is
19. “Another cup of coffee?" – "No, but thanks “
A. not at all B. for all C. all the same D. you for all
20. I've told him not to go out with those people, but he wouldn't listen. Just let him face
the music now.
A. Many a time B. Many the time C. Quite a time D. For a time
21. Most crimes that are committed are no more than theft.
A. slight B. small C. unimportant D. petty
22. This is the most difficult job I've ever had to do.
A. by heart B. by chance C. by far D. by myself
23. No matter how angry he was, he would never to violence.
A. resolve B. recourse C. exert D. resort
24. He had to retire from the match suffering from a ligament.
A. torn B. broken C. slipped D. sprained
25. , he felt so unhappy and lonely.
A. Despite of his riches C. Rich though he was
B. Rich as was he D. Despite of the fact that he was rich
26. as taste is really a composite sense made up of both taste and smell
A. That we refer to B. What we refer to
C. To which we refer D. What do we refer to
27. The police a good deal of criticism over their handling of the demonstration.
A. came in for B. brought about C. back out D. back up
28. Paul's been in Alice's bad ever since he offended her at the party.
A. eyes B. books C. likes D. treats
29. The photocopier in our office needs a complete . These copies are terrible.
A. maintenance B. repair C. overhaul D. renovation
30. The burglar's presence was betrayed by a floorboard.
A. crackling B. crunching C. groaning D. creaking
PART III. GUIDED CLOZE TEST (5PTS)
LETTER TO THE EDITOR
The Prime Minister's comments yesterday on education spending (31) the point, as the
secondary education system also needs a major overhaul. Firstly, the system only views the weakest
learners as having special needs. The brightest and most conscientious students are not encouraged

2|Page Tài liệu khóa học Live - VIP


OTTO CHANNEL
Otto Channel
to develop to their full (32) . Secondly, there's too much testing and not enough learning. My
fifteen-year-old daughter, for example, has just spent the last month or so (33) for exams.
These aren't even real, important exams, as her GCSES will be next year. They're just (34) exams.
Is the work she's been doing really going to make her more knowledgeable about her subjects, or will
she forget it all tomorrow? I suspect the (35)
Thirdly, the standard (36) doesn't give students any tuition in developing practical
work-related, (37) and social skills, or in skills necessary for higher education. How many
students entering university have the first idea what the difference is between (38) someone
else's work and (39) good use of someone else's ideas? Shouldn't they have been taught this at
school? How many of them are really able to go about (40) – a skill that's essential at
university because there are no teachers to tell you what to do – in an efficient way? Indeed, how
many students graduate from university totally unable to spell even simple English words correctly?
The system is letting our children down.
31. A. lose B. escape C. miss D. fail
32. A. capacity B. ability C. achievement D. potential
33. A. cramming B. lecturing C. reading D. practicing
34. A. false B. mock C. fake D. artificial
35. A. latter B. frontier C. later D. former
36. A. timetable B. lecture C. seminar D. curriculum
37. A. life B. alive C. living D. live
38. A. writing B. going over C. plagiarizing D. repeating
39. A. taking B. making C. having D. creating
40. A. reviewing B. revision C. distance learning D. self-study
PART IV. PARAGRAGH UNDERSTANDING (5PTS)
Fill in each gap in the passage with one suitable sentence from A-J
WHAT YOU NEED TO KNOW ABOUT CULTURE SHOCK
Most people who move to a foreign country or culture may experience a period of time when they
feel very homesick and have a lot of stress and difficulty functioning in the new culture. (41) .
First of all, it's important to know that culture shock is normal. (42) If you go, for
example, to a culture that is far different from your own, you're likely to experience culture shock more
sharply than those who move to a new culture knowing the language and the behavioral norms of the
new culture. (43) .
The first stage is usually referred to as the excitement stage or the 'honeymoon' stage. (44)
. During this stage you are merely soaking up the new landscape, taking in these impressions
passively, and at this stage you have little meaningful experience of the culture.
But it isn't long before the honeymoon stage dissolves into the second stage - sometimes called
the withdrawal stage. The excitement you felt before changes to frustration as you find it difficult to
cope with the problems that arise. (45) It is at this stage that you are likely to feel anxious
and homesick, and you will probably find yourself complaining about the new culture or country. (46)
.

3|Page Tài liệu khóa học Live - VIP


OTTO CHANNEL
Otto Channel
Culture shock is only temporary, and at some point, if you are one of those who manage to stick
it out, you'll transition into the third stage of cultural adjustment, the 'recovery' stage. At this point,
you'll have a routine, and you'll feel more confident functioning in the new culture. (47)
Customs and traditions are clearer and easier to understand. At this stage, you'll deal with new
challenges with humor rather than anxiety.
The last stage is the 'home' or 'stability' stage - this is the point when people start to feel at home
in the new culture. (48) .
There is, in a sense, a fifth stage to this process. (49) . This means that you may find
aspects
of your own culture 'foreign' because you are so used to the new culture that you have spent so long
adjusting to. (50) . Reverse culture shock rarely lasts for very long.
A. At this stage, you'll function well in the new culture, adopt certain features and behaviors from
your new home, and prefer certain aspects of the new culture to your own culture.
B. Upon arriving in a new environment, you'll be interested in the new culture, everything will seem
exciting, everyone will seem friendly and helpful and you'll be overwhelmed with impressions.
C. Reverse culture shock is usually pretty mild - you may notice things about your home culture that
you had never noticed before, and some of the ways people do things may seem odd.
D. There are four general stages of cultural adjustment, and it is important that are aware of these
stages and can recognize which stage you are in and when so that you will understand why you
feel the way you do and that any difficulties you are experiencing are temporary, a process you
are going through rather than a constant situation.
E. You'll start to feel less isolated as you start to understand and accept the way things are done
and the way people behave in your new environment.
F. This feeling is often called 'culture shock' and it is important to understand and learn how to cope
with culture shock if you are to adapt successfully to your new home's culture.
G. If you decide to return home after a long period in a new culture, you may experience what is
called 'reverse culture shock'.
H. It seems that everything is difficult, the language is hard to learn, people are unusual and
unpredictable, friends are hard to make, and simple things like shopping and going to the bank
are challenges.
I. This is the stage which is referred to as 'culture shock'.
J. Everyone in a new situation will go through some form of culture shock, and the extent to which
they do is determined by factors such as the difference between cultures, the degree to which
someone is anxious to adapt to a new culture and the familiarity that person has to the new
culture.
Your answer
41. 42. 43. 44. 45.
46. 47. 48. 49. 50.

4|Page Tài liệu khóa học Live - VIP


OTTO CHANNEL
Otto Channel
PART V. READING COMPREHENSION (5PTS)
Read the following passage and choose the best answer for each of the questions below.

THE WORLD AND ITS GLOBAL ECONOMY


The world as man knows it today is getting smaller and smaller because of technology such as the
Internet and high speed modems. In fact, on March 3, 2005, a man flew entirely around the globe
without refueling or stopping in a one-person jet. The world is changing the world, and as the 21st
century continues, the global economy will play a larger and larger role. As Thomas Friedman so
eloquently put it in Lexus and the Olive Tree, globalization is “the inexorable integration of markets,
nation-states, and technologies to a degree never witnessed before."
[A] With today's technology it is possible for people to solicit business from the far side of the globe.
[B] A company like Dell Computers can order parts from several different countries, take shipment in
North Carolina where the new computer will be assembled, and then ship them to all regions of the
globe. [C] An American oil firm can do a satellite survey in Siberia for oil deposits and then contract
with a Russian oil firm to drill the actual well, while the petroleum engineer, acting as the project
supervisor, remains in the US and runs the project by using a computer, a high speed Internet
connection, and a cellular telephone for quick questions. [D]
A global economy poses some serious problems. If the company doesn't act fast enough, it can lose,
and if the company loses heavily, what will happen to the employees working for the company, and
then in turn what will happen to the stores that depend on those employees buying their goods? As
can be seen, there is a definite trickle-down effect. How is the child who is about to graduate from
high school supposed to decide on what career field to enter? A career field that is here today might
gone long before the child can graduate from college, so not only does it become vital that a person
gain the needed knowledge to enter a given career field, but the person also needs to learn how to
learn. Leaning how to learn may prove to be given more necessary than the knowledge needed to
enter a given career field. A person who is good at learning how to learn can quickly adapt to changes
in the global economy by quickly preparing to enter other career fields if his job is here today, but
gone in the morning.
If the world turns into a global economy, a person will need to be able to get along and work with
people from different cultural backgrounds. However, unless a person has spent time living in different
parts of the world, this might be hard to do. While many students from foreign countries, especially
the Asian countries, come to the US to earn a graduate degree, how many students from the US spend
even a semester abroad studying in another country? The answer to this question is of course a very
small percentage.
While individuals from some of foreign countries and some individuals from the US and the US
economy will adjust to globalization, will the rest of the world? Mass media are more than willing to
continue to stir the pot of controversy as they not only have to learn how to report the positive news,
but also dole out a continuous stream of negative news. When a person in a developing country sees
all the cars on the streets of Bejing, of course that person wants a car so he can show his neighbor
how wealthy he is, and all this does is promote unneeded consumption. Why does the person who has
nowhere to go and no money to spend for travel want to own a car? The simple answer is because

5|Page Tài liệu khóa học Live - VIP


OTTO CHANNEL
Otto Channel
the media paint owning a car as a symbol of wealth and it is human nature to want to become wealthy
or at least to appear wealthy.
On the positive side, as prices rise due to increased demands on scarce resources, there will be an
incentive to find affordable alternatives. For example, as the price of oil rises and along with it the
price of a gallon of gasoline, a point will be reached at which people are no longer willing to purchase
gasoline so they can drive their cars, and they will demand both alternative transportation methods
and cars which use another source of energy. A current online survey says $ 2.50 per gallon of gasoline
is the point at which the people in the US will start making demands on the auto manufacturer, which
will open up new career fields in a few countries that have the technology needed to meet the
demands; however, people around the globe will work together on it.
51. The word inexorable in the passage is closest in meaning to
A. upward B. recent C. inevitable D. preposterous
52. The word solicit in the passage is closest meaning to
A. lure B. sell C. help D. ask for
53. The word them in the passage refers to
A. parts B. the new computers
C. all regions D. several different countries
54. According to paragraph 2, what can be inferred about the role of a project supervisor?
A. to make sure the project is finished correctly and on time
B. to hand-check each of the steps in a project
C. to assign each of the items that needs to be done in a project
D. to talk to everyone in a daily basis
55. Based on the information in paragraph 3, which of the following best explains the trickle-down
effect?
A. Water runs downhill.
B. What happens at the top eventually affects those at the bottom.
C. The people who have things get more and those without anything continue to go without.
D. If a person gets yelled at when he is at work, he should go home and kick the furniture since
it will make him feel better.
56. According to the passage, people need to learn how to learn, otherwise
A. they will not move from the first grade to the second grade in elementary school
B. they will not be able to work in the same job throughout college
C. they will not be able to switch from one career to another as the global economy changes
D. they will not be able to graduate from college and become a member of the global economy
57. In paragraph 3, why does the author mention the phrase "If his job is here today, but gone in
the morning"?
A. To suggest that he is going to lose his job in the morning
B. To give an example of losing the job when he has reached the mandatory retirement age.
C. To explain that he is a part time worker and only works when the company needs him
D. To support the claim that learning how to learn is important to a person.

6|Page Tài liệu khóa học Live - VIP


OTTO CHANNEL
Otto Channel
58. Which of the sentences below best expresses the essential information in the sentence in italics
in the passage?
A. The mass media are like a witch stirring her cauldron.
B. The mass media always report the truth since this keeps the ratings up and brings in the
most advertising revenue.
C. The mass media will report half-truths if it will keep their ratings up so they can sell advertising.
D. The mass media report every angle of a story since reports are apolitical and never present
only one viewpoint.
59. In paragraph 5, what can be inferred from the description of the media about owning a car?
A. A car needs to be painted certain colors if it is going to show others a person is wealthy.
B. The media are so biased that they will provide paint if a person needs to paint his car so as
to project the car as a symbol of wealth
C. The media slant the stories, so it will appear to viewers that only wealthy people own a car.
D. The members of the media don't own cars, so they are jealous of those wealthy people who
do own cars.
60. Look at [A] [B] [C] [D] that indicate where the following sentence could be added to the
passage:
E - businesses will be the lucrative businesses in the future since they are available to everyone with
an Internet connection.
Where would the sentence best fit? [A] [B] [C] [D]
A. [A] B. [B] C. [C] D. [D]

SECTION B. WRITTEN TEST (70PTS)


PART I. VERB TENSES / FORMS (20PTS)
A. Supply the correct forms or tenses of the verbs given.
1. If there aren't any tickets left when we reach the front of the queue, we (wait) all
the time for nothing.
2. I haven't decided yet about whether to buy a new car or a second hand one. But I (think)
about it.
3. There (be) any errors, let me know.
4. We have just got to the top in time. The sun (rise) in a minute.
5. By midnight he (be) unconscious for forty-eight hours.
6. It (not be) Anna who cooked that food. She was out all day yesterday.
7. It is highly desirable that from every product in regular production, samples (withdraw)
periodically.
8. The portrait is widely known (paint) by an Italian.
9. Can you keep calm for a moment? You (always make) noise in class.
10. You (visit) Aunt Mary while you were in Paris.

7|Page Tài liệu khóa học Live - VIP


OTTO CHANNEL
Otto Channel
B. Complete the following passage with the correct forms or tenses of the verbs given.
A year ago, Laura reached her lowest point. (11. utterly exhaust) after a short walk, she was
picked up off the pavement and driven home by the police. "My feet felt (12. nail) to the
ground," she recalls. With permanent flu symptoms panic stricken and confined to a wheelchair, she
was eventually diagnosed as (13. have) chronic fatigue syndrome, the term doctors now use
for her illness. Laura, a 30-year-old marketing manager, (14. since make) a remarkable
recovery in her health. She is now able to walk for an hour, swims twenty lengths three times a week
and (15. contemplate) back to work. She puts her new-found sense of well-being down to a
technique called cognitive behavior therapy (CBT). CBT (16. aim) to help people understand
how the beliefs they hold about themselves and others influence mood and behavior and how re-
evaluating negative beliefs can help them to feel and behave differently. It (17. be) successful in
dealing with problems ranging from depression to chronic, unexplained pain. Research shows that CBT
can also help people like Laura. Yet many sufferers are bitterly opposed to the treatment, (18. argue)
that their condition is physical not psychological.
Psychological therapy, they believe, implies that they are to blame and their disorder is not
genuine. Laura says CBT (19. help) her change the way she thought about her problems without
implying that they were her fault "I was an "all-or-nothing" thinker and felt I had to be perfect in
everything. The therapy (20. encourage) a less perfectionist approach to life and helped me
gradually to build up more physical activity."
PART II. WORD FORMS (20PTS)
A. Complete the following sentences with the correct forms of the words given
1. It's undeniable that the of the local incompetent healer was responsible for her
sudden death (DIAGNOSE)
2. A few jokes can up a lecture. (LIFE)
3. He is late for meetings. He is always on time. (VARY)
4. Nothing wrong will happen to you as long as you follow the strict set by the inspector.
(GUIDE)
5. I'm thinking of giving Anna and Mathew a vase for her silver wedding. (CRYSTAL)
6. A renewable resource is one that may be replaced overtime by natural process or is
(EXHAUST)
7. It is said that the problem of rapid climate change has been caused by too drastic (FOREST)
8. The mother of the child hurried to her neighborhood drugstore. (BREATH)
9. The workers who were during the recession badly need help from the government.
(SIZE)
10. His performance in the match today his reputation as a great player. (LIE)
B. Complete the following passage with the correct forms of the words given

8|Page Tài liệu khóa học Live - VIP


OTTO CHANNEL
Otto Channel
As Philadelphia grew from a small town into a city in the first half of the important eighteenth
century, it became an (1. increase) marketing center for a vast and growing agricultural
hinterland. Market days saw the crowded city even more crowded, as farmers from within a radius of
24 or more kilometers brought their sheep, cows, pigs, vegetables, cider, and other products for direct
sale to the (2. town) The High Street Market was continuously (3. large)
throughout the period until 1736, when it reached from Front street to Third. By 1745 New Market
was opened on Second Street between Pine and Cedar. The next year the Callowhill Market began
operation. Along with market days, the institution of twice-yearly fairs persisted in Philadelphia even
after similar trading days had been (4. continue) in other colonial cities.
The fairs provided a means of bringing (5. hand) goods from outlying places to would-be
buyers in the city. Linens and stockings from Germantown, for example, were popular items. Auctions
were another popular form of occasional trade. Because of the competition, retail merchants opposed
these as well as the fairs. Although (6. government) attempts to eradicate fairs and
auctions were less than successful, the ordinary course of economic development was on the
merchants' side, as increasing business specialization became the order of the day. Export merchants
became (7. different) from their importing counterparts, and (8. special) shops
began to appear in addition to general stores selling a variety of goods. One of the reasons
Philadelphia's merchants generally (9. prosperity) was because the surrounding area was
(10. go) tremendous economic and demographic growth. They did their business, after all, in the
capital city of the province. Not only did they cater to the governor and his circle, but citizens from all
over the colony came to the capital for legislative sessions of the assembly and council and meetings
of the courts of justice.
PART III. GAP-FILLING (5PTS)
Fill in each gap with one suitable word.
The knowledge and eloquence that people gain through travelling is usually perceived (1)
the best fulfillment in life. It is the inquisitive human nature (2) impels people to
seek thrilling experiences and to set out on an exploration trip. Those who travel frequently and to
diverse places benefit from establishing new relationships and (3) a better knowledge about
other cultures and lifestyles. However, there is a grain of truth in the assumption that people are prone
to cherishing clichés and unfounded prejudices about other nations and their characteristics.
Sometimes, it is only the first-hand encounter that can help change the approach towards the (4)
'inferior communities'. This direct (5) with a different civilization enables travelers to drop
their baseless assumptions and get acquainted with the real concept of life in all four (6) of the
globe. Beyond question, travelling facilitates friendship and makes (7) easier for many individuals
to acknowledge the true value of different traditions and customs. Yet, it does not always (8)
enjoyment. It (9) also involve coming close with the atrocities of real existence as well as
becoming aware of the challenges and hardships that other people have to struggle with. Hence, a
true voyage is the one with a good deal of experience to reminisce about, (10) often combined
with exposure to abhorrent sights and incredible ordeals. The learning to be complete, thus, requires
an ability to observe and analyze the surroundings, both their glamour and brutality.

9|Page Tài liệu khóa học Live - VIP


OTTO CHANNEL
Otto Channel
PART IV. PREPOSITIONS & PHRASAL VERBS (5PTS)
Fill in each gap with one suitable preposition or particle.
1. The couple broke their engagement after they had a huge argument.
2. At school, Luis got into a lot of trouble for something I did, and now he holds it me.
3. Do you have to go already? Can't you stick for a few minutes?
4. Josh can while a whole day playing online computer games with his mates.
5. The company's announced it is laying over 1,000 workers.
6. The music blared and it was impossible to have a conversation.
7. My plans for starting a restaurant fell
8. After a week on the run, he gave himself to the police.
9. The students were slow to catch but gradually they began to understand.
10. We made that we had forgotten Jane's birthday, though it was not true.
PART V. ERROR IDENTIFICATION (10PTS)
Identify 10 errors in the following passage and correct them
1 Water scarcity is fast becoming one of the major limited factors in world crop production. In
2 many areas, poor agricultural practices have led to increasing desertification and the loss of
3 formerly arable lands. Consequently, those plants species that are well adapted with survival
4 in dry climates are being looked at for an answer in development more efficient crops to
5 grow on marginally arable lands.
6 Plants use several mechanisms to ensure their survival in desert environments. Some involve
7 pure mechanical and physical adaptations, such as the shape of the plant's surface, smaller
8 leafed size, and extensive root systems. Some of the adaptations are related to chemical
9 mechanisms. Many plants, such as cacti, have internal gums and mucilages which give them
10 water-retaining properties. Other chemical mechanism is that of the epicuticular wax layer.
11 This wax layer acts as an impervious cover to protect the plant. It prevents excessive loss
12 from internal moisture. It also protects the plant from external aggression, which can come
13 from inorganic agents such as gases, or organic agents which include bacteria ant plant pets.
14 Researchers have proposed that synthetic waxes with similar protective abilities could be
15 prepared based on knowledge of desert plants. If is successfully developed, such a
16 compound could be used to greatly increasing a plant's ability to maintain health in so
17 adverse situations as inadequate water supply, limited fertilizers availability, attacked by
18 pets, and poor storage after harvesting.

Lines Mistake Correction Lines Mistake Correction

10 | P a g e Tài liệu khóa học Live - VIP


OTTO CHANNEL
Otto Channel
PART VI. SENTENCE TRANSFORMATION (10PTS)
1. Bruce said that the situation at work was like a family argument. (LIKENED)
→ .....................................................................................................................................
2. When she sold the jewelry at such a low price, she was cheated. (RIDE)
→ .....................................................................................................................................
3. Suppose she make no changes at all for the time being?
→ What ............................................................................................................................
4. It's rumored that we will have a new manager.
→ Rumor has ....................................................................................................................
5. They arrived at their destination alive and kicking. (SOUND)
→ .....................................................................................................................................
6. At this moment I only have time to think about my university thesis. (PREOCCUPIED)
→ .....................................................................................................................................
7. Do you any have idea about how Jack made enough money to buy his new house? (LIGHT)
→ Can you ..................................................... Jack made enough money to buy his new house?
8. My salary is half what I would be in the job I was offered in January.
→ If .................................................................................................................................
9. Our boss is absolutely determined not to give us that pay rise. (INTENT)
→ Our boss ..................................................... the pay rise.
10. The value of sterling has fallen considerably in the past week.
→ There ............................................................................................................................

THE END

11 | P a g e Tài liệu khóa học Live - VIP


OTTO CHANNEL
Otto Channel
SỞ GIÁO DỤC & ĐÀO TẠO KỲ THI OLYMPIC TRUYỀN THỐNG 30/4
TP. HỒ CHÍ MINH LẦN THỨ XXV – NĂM 2013
ĐỀ CHÍNH THỨC Môn thi: Anh văn - Khối: 10
Thời gian làm bài: 180 phút

➢ Thí sinh làm phần trắc nghiệm (MULTIPLE CHOICE) trên phiếu trả lời trắc nghiệm và phần tự
luận (WRITTEN TEST) trên phiếu trả lời tự luận.
➢ Trên phiếu trả lời trắc nghiệm, thí sinh tô thêm 2 số 00 vào trước số báo danh (bằng bút chì).
Phần mã đề thi trên phiếu trắc nghiệm, thí sinh tô vào ô 002.
 --------------------------------------------------------------------------------------------------------------------

CẤU TRÚC ĐỀ THI


SECTION A. MULTIPLE CHOICE (40 PTS) ............................................................................ 1
PART I. PHONOLOGY (5PTS) .......................................................................................................................... 1
PART II. WORD CHOICE (5 PTS) .................................................................................................................... 1
PART III. GRAMMAR AND STRUCTURES (5PTS).......................................................................................... 2
PART IV. PHRASAL VERBS AND PREPOSITIONS (5PTS) .......................................................................... 2
PART V. GUIDED CLOZE (10PTS) ................................................................................................................... 3
PART VI. READING COMPREHENSION (10PTS) .......................................................................................... 5

SECTION B. WRITTEN TEST (60PTS) ................................................................................... 9


PART I. CLOZE TEST (20PTS) ......................................................................................................................... 9
PART II. WORD FORMATION (20PTS)......................................................................................................... 10
PART III. ERROR CORRECTION (10PTS) ..................................................................................................... 11
PART IV. SENTENCE TRANSFORMATION (10 PTS) .................................................................................. 11
Otto Channel
SỞ GIÁO DỤC & ĐÀO TẠO KỲ THI OLYMPIC TRUYỀN THỐNG 30/4
TP. HỒ CHÍ MINH LẦN THỨ XXV – NĂM 2013
ĐỀ CHÍNH THỨC Môn thi: Anh văn - Khối: 10
Thời gian làm bài: 180 phút

➢ Thí sinh làm phần trắc nghiệm (MULTIPLE CHOICE) trên phiếu trả lời trắc nghiệm và phần tự
luận (WRITTEN TEST) trên phiếu trả lời tự luận.
➢ Trên phiếu trả lời trắc nghiệm, thí sinh tô thêm 2 số 00 vào trước số báo danh (bằng bút chì).
Phần mã đề thi trên phiếu trắc nghiệm, thí sinh tô vào ô 002.
 --------------------------------------------------------------------------------------------------------------------
SECTION A. MULTIPLE CHOICE (40 PTS)
PART I. PHONOLOGY (5PTS)
Choose the word whose underlined part is pronounced differently from the others.
1. A. choreograph B. christianity C. archaic D. chivalry
2. A. slaughter B. borough C. drought D. laughter
3. A. humane B. locate C. strategy D. rabies
4. A. wretched B. blessed C. allegedly D. knackered
5. A. realm B. cleanse C. heather D. meager
Choose the word which is stressed differently from the other three.
6. A. architecture B. comparison C. championship D. communism
7. A. propaganda B. influential C. mediocre D. obligatory
8. A. addressee B. referee C. employee D. nominee
9. A. malice B. leopard C. phenom D. cohort
10. A. unaffected B. unanimous C. unaccountable D. unambiguous
PART II. WORD CHOICE (5 PTS)
Choose the best options to complete the following sentences.
11. When I finish writing this composition, I'm going to and go to bed.
A. make time B. hit the hay C. hit the big time D. call it a day
12. Mary usually buys her clothes . It's cheaper than going to a dressmaker.
A. in public B. on the shelf C. off the peg D. on the house
13. You are at to do what you like
A. freedom B. odds C. disposal D. liberty
14. Without written evidence, we don't have a on.
A. leg to stand B. foot to stand C. leg to lean D. foot to lean
15. It was very strange but I had a(n) that the plane would crash.
A. intuition B. omen C. premonition D. prediction
16. Although he spoke slowly, I found it difficult at times to follow the of his argument.
A. spool B. track C. thread D. path
17. I intend to an official complaint to the Director.
A. write B. lodge C. place D. take

1|Page Tài liệu khóa học Live - VIP


OTTO CHANNEL
Otto Channel
18. The transport strikes a real , which will make it very difficult for me to get to work.
A. pain in the back B. nuisance C. last straw D. frustration
19. benefits include a new car and free health insurance.
A. Well B. Fringe C. Edge D. Verge
20. Mind that the baby shouldn't touch the knife; it's as sharp as a .
A. blade B. sword C. cut D. razor
PART III. GRAMMAR AND STRUCTURES (5PTS)
Choose the best options to complete the following sentences.
21. He prefers to attend Economics University rather than .
A. going to Polytechnic C. studying Polytechnic
B. to be accepted to Polytechnic D. to attend Polytechnic
22. It pays some professional advice before you make a decision.
A. get B. getting C. to getting D. to get
23. It able to finish it in an hour.
A. can't have been too hard if you had been C. couldn't be too hard if you are
B. can't have been too hard if you were D. couldn't be too hard if you had been
24. It could have been a lot worse there.
A. when he had not been C. had he not been
B. for he had not been D. whether or not he had been
25. believed to be over 300 species of trees in El Yunque rain forest in Puerto Rico.
A. There are B. They are C. It is D. Is has been
26. classified as a carnivore, the North American Grizzly bear eats berries and even grass.
A. Just as B. Because of C. Although D. Either
27. I'm my brother is.
A. nothing near as ambitious B. nothing as ambitious than
C. nowhere like so ambitious D. nowhere near as ambitious as
28. On the island the only representation of the Indians' handicraft.
A. does it remain B. did it remain C. remains D. remains it
29. It is advised that not only Tam but also his friends punctual.
A. be B. are C. is D. were
30. Many a boy come up with many new ideas.
A. have B. have been C. has D. has been
PART IV. PHRASAL VERBS AND PREPOSITIONS (5PTS)
31. If you pay the restaurant bill with your credit card, I'll with you later.
A. settle down B. pay back C. settle up D. pay up
32. Demand for the products is expected to peak 5 years from now and then to .
A. taper off B. fall down C. set back D. drift away

2|Page Tài liệu khóa học Live - VIP


OTTO CHANNEL
Otto Channel
33. It was an embarrassing situation, but she managed to .
A. laugh at it B. laugh on it C. laugh it off D. laugh it out
34. The police a good deal of criticism over their handling of the demonstration.
A. came in for B. brought about
C. went down with D. opened up
35. The rain was simply down on the deserted street.
A. pelting B. spraying C. showering D. dripping
36. As we were waiting on the pavement, a black Mercedes beside us.
A. pulled up B. pulled down C. pulled off D. pulled through
37. I was the impression that you liked Indian food.
A. on B. with C. over D. under
38. Ms. Dung, your form teacher, as a very sympathetic person. She can be a shoulder to cry on.
A. comes out B. comes at C. comes over D. comes about
39. She brought the deal even though nobody thought she was capable of doing it.
A. in B. off C. out D. down
40. It took the parents a long time to their children's games.
A. catch with B. catch up C. catch on to D. catch out
PART V. GUIDED CLOZE (10PTS)
Read the text below and decide which answer best fits each space.
PASSAGE 1
(41) Christmas evolved in the United States, new customs were (42) and many
old ones were reworked. The legend of Santa Claus, for example, had (43) in Europe
and was brought by Dutch settlers to New York in the early 18 century. Traditionally, Santa Clause -
th

from the Dutch Sinter Klaas – was depicted (44) a tall, dignified, religious figure riding a
white horse (45) the air.
Known as Saint Nicholas in Germany, he was usually accompanied by Black Peter, an elf who
punished disobedient children. In North America, he (46) developed into a fat, jolly old gentleman
who had neither the religious (47) of Saint Nicholas nor the (48) disciplinarian character of
Black Peter.
Santa's transformation began in 1823, when a New York newspaper published the poem "A Visit
from Saint Nicholas", which Clement Clark Moore had written to amuse his daughter. The poem
introduced many Americans to the story of a kindly saint who flew over housetops in a reindeer-drawn
sleigh. Portraits and drawings of Santa Claus by American illustrator Thomas Nast further (49)
the legend during the second half of the 19th century. Living at the North Pole and
assisted by elves, the modern Santa produced and delivered toys to all good children. By the late 19th
century, he had become such a (50) figure of American folklore that in 1897, when Virginia
O'Hanlon wrote to the New York Sun newspaper asking if Santa was real, she received a direct answer:
"Yes, Virginia, there is a Santa Claus."

3|Page Tài liệu khóa học Live - VIP


OTTO CHANNEL
Otto Channel
41. A. As B. Since C. Through D. Now that
42. A. adapted B. acclaimed C. adopted D. attributes
43. A. roots B. stems C. origins D. assumed
44. A. by B. as C. for D. backgrounds
45. A. up B. on C. at D. into
46. A. eventually B. at last C. finally D. through
47. A. hopes B. roles C. temperaments D. ultimately
48. A. severe B. strict C. harsh D. austere
49. A. strengthened B. consolidated C. confirmed D. bettered
50. A. prominent B. prime C. domineering D. principle
PASSAGE 2
CARNIVOROUS PLANTS
All plants rely on nutrients taken from the soil in order to survive. However, in areas where the soil
does not contain enough (51) nutrients, some plants have adapted to (52) their diets
from another source: living organisms.
Though they are few in number, carnivorous plants are (53) fascinating beings that
"eat" anything from one-celled organisms to insects in order to survive. They are commonly found in
marshlands. Carnivorous plants feature one of several types of "traps" to ensnare prey, which they
consume to make up for nutrients that may be missing from the soil. While there are over 400 species
of carnivorous plants in the world today, some are more (54) than others.
The most well-known of these plants are the snap traps, which include the Venus flytrap. Snap
traps are easily identified by their leaves, which are separated into two lobes that have the ability to
fold together. Inside the lobes, the surface is covered with tiny hairs that are (55) to
movement. When the plant's prey brushes against the hairs, it triggers a closing mechanism that rapidly
brings the two lobes together, trapping the prey (56) inside. The response of the traps is
phenomenal (57) speed: the time between triggering the hairs and snapping shut is less than a
second. As the prey struggles inside the trap, it only triggers more hairs, causing the leaves to tighten
their (58) . The plant then secretes liquid chemicals from special glands into the trap to dissolve
the prey and absorb all of its nutrients. Besides the Venus fly trap, only one other type of snap trap
exists today, (59) to as the waterwheel plant. The two share a common ancestor and differ
only in a few ways. For instance, the waterwheel is an aquatic plant, while the fly trap is exclusively
terrestrial. In addition, the fly trap feeds primarily on arthropods like spiders, while the waterwheel
lives (60) simple invertebrates, like certain types of plankton.
51. A. vital B. essential C. crucial D. indispensable
52. A. enlarge B. modify C. supplement D. augment
53. A. nevertheless B. nonetheless C. though D. contradictorily
54. A. prevailing B. prevalent C. current D. domineering
55. A. liable B. prone C. sensitive D. vulnerable
56. A. closely B. irreplaceably C. steadily D. securely
57. A. in regard to B. in accordance with C. in preference to D. on merits of
58. A. hold B. fist C. grip D. seizure
59. A. denoted B. indicated C. referred D. implicated
60. A. off B. onto C. with D. through

4|Page Tài liệu khóa học Live - VIP


OTTO CHANNEL
Otto Channel
PART VI. READING COMPREHENSION (10PTS)
Read the text below and choose the best answer to each question.
PASSAGE 1
GERTRUDE STEIN
One of the most influential literary figures of the twentieth century was American author Gertrude
Stein. Her literary style and vision was often a radical departure from traditional methods, which relied
on a more linear plotline. Instead, Stein focused on language itself by employing methods of repetition
and spontaneity in an attempt to mirror human consciousness. But, Stein's influence did not stop with
her writing. As an expatriate in Paris, she was responsible for bringing some of the greatest minds in
art and literature together at her apartment, Salon 27. The Spanish painter Pablo Picasso and American
writer Ernest Hemingway were two frequent visitors. Indeed, Stein was the one who coined the phrase
"Lost Generation" to denote up-and-coming American writers living in Paris at the time mainly due to
their disillusionment with art as a whole back home. While Gertrude Stein may not be the most
recognizable figure in literature, her personal and literary influence on artists was invaluable.
Stein began living in Paris in 1903. Eventually, her flat, Salon 27, became a center of intellectual
exchange for writers and artists. Before long, she became an integral part of the artistic and literary
scene in the city and befriended numerous important figures, such as Picasso and Matisse. They would
meet there weekly, where they could expound on new theories of art, philosophy, literature, politics,
and social issues in the stimulating, productive environment provided by Stein. Without Stein's Salon
27, it is quite possible that many of the artists and writers of the day never would have crossed paths,
and the individual disciplines would not have become as diverse or fully developed as they eventually
did. These interactions also became a major influence on Stein's own literary style.
Stein became enamored with Picasso's cubist style, and, as a result, many of his earliest works
adorned the walls of her apartment. But they were not simply decorative. Cubism attempts to reduce
the subject form its natural form into an abstract, geometrical shape capable of numerous angles of
perception. In a similar fashion, Stein attempted to interpret cubism through literature and writing.
Like cubist painters, Stein wrote in a style that took into consideration every possible angle of her
subject matter. She wanted to give readers the opportunity to view her work on many different levels,
not just a single, flat surface. But, unlike the cubists who relied on formal structure to some degree.
Stein took it a step further and placed less emphasis on formal writing structures such as grammar
and syntax and focused on what she called "automatic" language, which was spontaneous and
repetitive and relied more on the spoken word.
In many ways, Stein's style was a reaction against nineteenth century authors' tendency to place
order and structure above all other considerations. Stein revolutionized the style of the twentieth
century by undermining traditional, expected methods that came before the modern era. Stein
focused on the process of writing, not its ultimate result. She also believed that all parts of a sentence
were equally important. For example, to her, an article was as important as a noun, and a conjunction
was as vital as a verb. In essence, every part of a sentence was related. She learned this from painters,
who believed every brushstroke on a canvass contributed to the whole and could not be left out.
Furthermore, the act of perception, whether it be art or literature, was crucial, not the ultimate outcome
because to Stein, consciousness never ceases. It is continuous, so it became her task to try to embody

5|Page Tài liệu khóa học Live - VIP


OTTO CHANNEL
Otto Channel
the movement of consciousness in her work through techniques such as the repetition of words or
spontaneity rather than depicting events or a plotline in a linear fashion.
In many ways, Stein's writing mirrors the chaotic, detached atmosphere of post- Word War I Paris,
yet it also marks the moment when literature began to leave the nineteenth century behind in favor of
a more modernist style of imagination and innovation. It is also a clear precursor of stream of
consciousness, which dominated writing styles during the rest of the twentieth century in America. For
instance, Hemingway's style in many of his early novels is clearly influenced by Stein. While Stein was
able to capitalize on artistic renditions of life and translate them into literature, other artists benefited
from her willingness to reject the accepted traditions and create her own.
61. The word "radical" is closest in meaning to
A. gradual B. abrupt C. swift D. drastic
62. The author's description of Gertrude Stein mentions which of the following?
A. She was originally from Paris and later moved to the US.
B. Her writing style was centered on a structured plotline.
C. She attempted to focus on the linguistic side of writing.
D. Her work was quite controversial due to its extreme nature.
63. Which of the following can be inferred from Paragraph 1 about American expatriates in Paris?
A. They did not completely agree with Picasso's style at first.
B. They were more satisfied with the creative capacity of Europe.
C. They attempted to forge a new style of literary consciousness.
D. They rarely met together in Stein's apartment to converse.
64. The author discusses Salon 27 in paragraph 2 in order to
A. illustrate its relevance as a place of artistic interaction of the time.
B. note it is where the painters Matisse and Picasso first met one another.
C. indicate it was an adequate place for such great minds to mingle.
D. show that artists of the day lacked a meeting place at which to gather.
65. According to paragraph 3, Stein followed the cubist style because
A. it focused on the geometric forms inherent in most subject matter
B. it gave her a way to present her reader with various perspectives.
C. it was the simplest form with which to interpret and understand her work.
D. it eschewed the secondary parts in favor of the major parts of art.
66. The word "undermining" in the passage is closest in meaning to
A. simplifying B. overshadowing C. promoting D. weakening
67. The author's description of Stein's revolutionary style mentions all of the following EXCEPT
A. She placed reason, order and logic above all other facets of her technique.
B. She considered each part of the sentence to be essential to its weight and meaning.
C. She did not believe that the finality of a work of literature was very important.
D. She attempted to capture the movement of consciousness through various techniques.
68. The word "embody" in the passage is closest in meaning to
A. evade B. express C. simplify D. replicate

6|Page Tài liệu khóa học Live - VIP


OTTO CHANNEL
Otto Channel
69. According to paragraph 5, Stein's style is a product of the era because
A. it translates the sentiment of Europeans during a time of reconstruction.
B. it is more imaginative and innovative than anything the world had seen.
C. it continues the former structural traditions present before World War II.
D. it rejects the past and is fashioned out of the sentiment of post-war Europe.
70. The word "renditions" in the passage is closest in meaning to
A. interpretations B. outlooks C. perspectives D. concepts
PASSAGE 2
According to accounts, when the first astronauts in space looked down and saw the Earth floating
in the vast black void, they had what can only be described as a profound spiritual experience; in an
instant they had attained a 'global consciousness' in which all national and international boundaries
disappeared, and they were left with the awesome realization that they were mere 'planetary citizens'.
To the astronauts, the planet looked as if it were some huge single living system. The photographs
they brought back touched us all in some way, and the blue sphere in space came to symbolize the
oneness of all humanity and life on Earth. The idea that the planet might be alive, strange though it
sounds, was soon to gain credence, even among the scientific community.
Not long afterwards in the 1970s, the hypothesis that the Earth's biosphere actually functions as a
single living system was put forward by Dr James Lovelock, a British scientist and inventor who had
been commissioned by NASA to help determine whether or not there was life on Mars. By comparing
the atmospheres of both planets, he soon realized that, while Mars had a stable, unchanging, 'dead'
atmosphere, Earth had no such equilibrium, and that there were some complex processes going on. It
was this imbalance that made the planet suitable for sustaining life. He postulated that: 'the physical
and chemical condition of the surface of the Earth, of the atmosphere and of the oceans has been,
and is, actively made fit and comfortable by the presence of life itself... in contrast to the conventional
wisdom which held that life adapted to the planetary conditions as it, and they, evolved their separate
ways.'
Suffice it to say, Lovelock knew that when looking at the Earth in this way, what he was seeing
was not so much a planet that just happened to be suitable for sustaining life, but a self-evolving and
self-regulating system that adjusted itself to support life. This seemed to qualify the Earth as a living
entity in her own right, so he named her 'Gaia' after the Greek goddess who was said to have drawn
the living world forth from Chaos - and the 'Gaia hypothesis' was born.
Lovelock first published his idea in 1979 in his book, Gaia, a New Look at Life on Earth, although
the science behind the hypothesis was still imprecise. The ideas in the book provoked a storm of
criticism, but also generated a lot of research, which has since led to profound new insights about life
on Earth. For instance, Lovelock knew that the heat of the sun has increased by 25% since life began
on Earth, yet he did not understand by which process the temperature on the surface had been kept
at the optimum conditions suitable for sustaining life.
Since that time, many of the mechanisms by which Gaia regulates her systems have been identified.
For example, it has been shown that cloud formation over the open ocean is almost entirely a function
of the metabolism of oceanic algae. Previously, it was thought that this cloud formation was a purely
chemical phenomenon. Further research suggested that Gaia has automatically been controlling global

7|Page Tài liệu khóa học Live - VIP


OTTO CHANNEL
Otto Channel
temperature, atmospheric content, ocean salinity, and other factors in order to 'maintain the conditions
suitable for its own survival', in much the same way that any individual organism regulates its body
temperature, blood salinity etc.
Similarly, all the life forms on the planet are a part of Gaia, in a way analogous to the different
organs in a body, each with its own function. The oceans and atmosphere act as the planet's circulatory
and temperature control systems, while the tropical rainforests could be compared to the liver,
cleansing the body of toxins. In their diversity, the myriad life forms of earth co-evolve and contribute
interactively to produce and sustain the system as a whole.
Some of Lovelock's critics took his hypothesis to imply that the Earth was behaving with a sense
of purpose, that it was being a teleological* being, actively controlling the climate and so on. However,
Lovelock had never stated that planetary self-regulation was purposeful, only that it was a living, highly
complex system. No one doubts that plants or bacteria are alive, yet they do not produce processes
nearly as complicated as the Earth's.
The Gaia Theory has already had a huge impact on science and has inspired many leading figures
of the past 20 years, who have written and spoken eloquently about how we can model human
activities that are beneficial to the living systems of our planet. By making us more aware of the
damage we are doing to the eco- system, Gaia theory may also help us to survive. We are just one
part of a larger system, and are reliant on that system for our continued existence. As Lovelock said:
'if we see the world as a superorganism of which we are a part not the owner, nor the tenant, not
even a passenger - we could have a long time ahead of us and our species might survive for its 'allotted
span'. It all depends on you and me.
* A being with an ultimate purpose/design towards an end.
71. The first astronauts in space were me.
A. conscious of the lack of physical boundaries between nations.
B. forced to adjust their perspective of their place in the cosmos.
C. profoundly affected by the symbolism of the Earth.
D. made aware of the life-forces operating on Earth.
72. The word "credence" is closest in meaning to
A. acceptance B. concordance C. protection D. rejection
73. Dr. James Lovelock had originally
A. been an inventor in Britain. C. been employed to compare Mars with Earth.
B. been looking for Martian life. D. proved Mars was a dead planet.
74. The word "they" in paragraph 2 refers to
A. life on Earth B. planetary conditions
C. oceans D. atmosphere and temperatures
75. Lovelock surmised that .
A. Earth's inconstant atmosphere was a bi-product of life on the planet.
B. the chemical condition of the Earth had come about by accident.
C. the imbalance of gases on Earth had created life.
D. life had evolved to survive in Earth's planetary conditions.

8|Page Tài liệu khóa học Live - VIP


OTTO CHANNEL
Otto Channel
76. Research has shown that .
A. the various planetary systems are regulated by different mechanisms.
B. clouds are formed by metabolic chemical changes in the sky.
C. the saltiness of the seas is due to the presence of oceanic algae.
D. Gaia can ultimately control her own survival.
77. The word "analogous" is closest in meaning to
A. consistent B. comparable C. related D. traceable
78. According to Gaia theory
A. the planet has physical biological organs similar to a person's.
B. the oceans control the temperature of the Earth's atmosphere.
C. the rainforests can remove all the pollution from the Earth.
D. each species on Earth has a part to play in the planet's survival.
79. According to Lovelock,
A. higher forces are at work behind the Earth's existence and survival.
B. the Earth had developed senses and was conscious of its purpose.
C. the complex life-forces on earth are equal to the sum of a living being.
D. bacteria and plants are alive but can only produce simple processes.
80. The Gaia hypothesis may ultimately ensure the immediate survival of
A. the planet Earth C. the ecosystem
B. the human species D. all life on Earth

SECTION B. WRITTEN TEST (60PTS)


PART I. CLOZE TEST (20PTS)
Read the text below and think of the word which best fits each space. Use only ONE WORD for each
space
OPEN CLOZE 1
Quite different from storm surges are the giant sea waves called tsunamis, which derive their (1)
from the Japanese expression for "high water in a harbor".
These waves are also referred to by the general public as tidal waves, although they have relatively
(2) to do with tides. Scientists often refer to them as seismic sea waves, far more appropriate in
(3) they do result from undersea seismic activity.
Tsunamis are caused when the sea bottom suddenly moves and the water above the moving earth
is suddenly displaced. This sudden shift of water sets off a series of waves, which can travel great
distances at speeds close to 700 kilometers per hour. In the (4) ocean, tsunamis have little
noticeable amplitude, often no more than one or two meters. It is when they hit the shallow water (5)
they increase in (6) possibly up to 40 meters. Tsunamis often occur in the Pacific
because the Pacific is an area of (7) seismic activity. Two areas of the Pacific (8)
accustomed to the threat of tsunamis are Japan and Hawaii. Because the seismic activity that
causes tsunamis in Japan often (9) on the ocean bottom quite close to the islands, the tsunamis
that hit Japan often come with little (10) and can therefore prove disastrous.

9|Page Tài liệu khóa học Live - VIP


OTTO CHANNEL
Otto Channel
OPEN CLOZE 2
The response of most animals when suddenly faced with a predator is to flee. (11)
selection has acted in a variety of ways in different (12) to enhance the efficacy of the
behaviors, known as "flight behaviors" or escape behaviors that are used by prey in fleeing predators.
Perhaps the most direct adaptation is enhanced flight (13) and agility.
Adaptations for speed, however, are likely to require sacrifices, so we might expect only some
species to adopt a simple fast flight strategy. Another way of enhancing the effectiveness of flight is
to move in an erratic and unpredictable (14) Many species, like ptarmigans, snipes, and
various antelopes and gazelles, flee from predators in a characteristic zigzag fashion. Rapid unexpected
changes in flight direction (15) it difficult for a predator to track (16) . In some
species, like the European hare, erratic zigzag flight might be more effective in the (17) of
predators that are faster than they are and straight flight more effective against predators that are
(18) . A quite different way of enhancing escape by flight is to use so-called "flash" behavior.
Here, the alarmed grey flees for a short distance and then "freezes". Some predators are unexcited by
(19) prey, and a startling flash of activity followed by immobility may confuse them. "Flash
behavior" is used in (20) by frog and orthopteran insects.
PART II. WORD FORMATION (20PTS)
A. Complete each sentence, using the correct form of the word in parentheses.
1. This involved some dealings with the chief of the police. (HAND)
2. Ensure your screen colors are not causing eye strain. Avoid fluorescent screens.
(COLOR)
3. You can travel from one end of the park to the other on a railway. (MINIMIZE)
4. Mr. Madill stated that the against him were unfounded. (ALLEGE)
5. The lingering war in the country has brought about nothing but the effects of poverty and
squalor. (HUMAN)
6. The between blacks and whites are more and more common. (MARRY)
7. I don't think Tom's getting too much sleep lately. His eyes are terribly . (BLEED)
8. He has unwavering belief in something unreal such as forces. (NORMALITY)
9. can be dangerously close to racism. (NATION)
10. The world champion was by a younger Russian challenger. (THRONE)
B. Complete the passage with appropriate forms from the words given in the box.
kick leaver ikelihood ecstasy relation
adventurous nurture count attend remark

Over the years, there have been (11) fans of the classic Hanna-Barbera cartoon character
Yogi Bear. The cartoon series enjoyed by young and old alike revolved mostly around the (12) of
this loveable bear and his (13) Boo-Boo as they tried unsuccessfully to snag "pic-a-nic"
baskets in the made-up land of Jellystone Park. It is not often that people think about where the ideas
for these cartoon characters come from, which brings up an interesting point: do bears actually search
for food (14) in picnic baskets and (15) campsites?

10 | P a g e Tài liệu khóa học Live - VIP


OTTO CHANNEL
Otto Channel
(16) enough, bears have been known to seek out food from some (17)
sources, including picnic baskets, on top of their usual diet of berries, insects, and fish. Bears work
throughout the summer and fall to build up fat stores so as to have energy enough to last them through
their winter hibernations. (18) to this is their need to replenish their depleted reserves when
they wake up in the spring. Food is generally scarce in the early spring, and consequently they will
(19) indulge in any foods that are (20) . This is the main reason for many incidents involving
bears entering campsites in search of food.
PART III. ERROR CORRECTION (10PTS)
The following passage contains 10 errors. Identify and correct them.
1 Many flowering plants woo insect pollinators and gently direct them to their most fertile
2 blossoms by changing the colors of individual flowers from day to day.
3 Through color cues, the plant signals for the insect that it would be better off visiting one
4 flower on its bush than other. The particular hue tells the pollination that the flower is full
5 of far more pollen than are neighboring blooms. That nectar- rich flower also happens to
6 be fertile and ready to disperse its pollen or to receive pollen the insect has picked out from
7 another flower. Plants do not have to spend precious resources remaining reservoirs of
8 nectar in all their flowers. Thus, the color-code communication system benefits both plants
9 and insects.
10 For example, on the lantana, a flower starts out on the first day as yellow, when it is rich
11 with pollen and nectar. Influenced by an as-yet-unidentified environmental signal, the
12 flower changes its color of triggering the production of the pigment anthramycin. It turns
13 orange on the second day and red on the third. By the third day, it has no pollen to offer
14 insects and is no longer fertile. On any given lantana bush, only 10 to 15 percent of the
15 blossoms are likely to be yellow and fertile. But in the tests measuring the response of
16 butterflies, it's discovered that the insects visited the yellow flowers at least 100 times more
17 than would be expected from the haphazard visitation. Experiments with paper flowers
18 demonstrated that the butterflies were responding with color cues rather than, say, the
19 scent of the nectar. In other types of plants, blossoms change from white to red, others
20 from yellow to red, and so on. These color changes have been observed in 74 families of
21 plants.

Lines Mistake Correction Lines Mistake Correction

PART IV. SENTENCE TRANSFORMATION (10 PTS)


Rewrite the following sentences using the words given.
1. This liver condition is common among those who drink a lot.
→ This liver condition .................................................................................. drinkers.

11 | P a g e Tài liệu khóa học Live - VIP


OTTO CHANNEL
Otto Channel
2. "I think the whole idea is ridiculous," he said.
→ He dismissed ..................................................................................................................
3. He threatened the officers with violence.
→ He made ........................................................................................................................
4. It shouldn't have surprised me that my children didn't like the new, cheaper ice- cream.
→ I might ............................................................................................................................
5. Her lateness made the boss angry. (BANANAS)
→ It was because she .........................................................................................................
6. She wore headphones in order not to disturb anyone.
→ She wore headphones lest ...............................................................................................
7. She is not very good at arranging flowers. (FLAIR)
→ .......................................................................................................................................
8. Bill changed his ways when he came out of prison. (LEAF)
→ Bill has ...........................................................................................................................
9. You must not, I repeat not, open this box until Christmas Day. (TO)
→ Under ........................................................................... opened until Christmas Day.
10. Her early success has made her very conceited. (HEAD)
→ .......................................................................................................................................
THE END

12 | P a g e Tài liệu khóa học Live - VIP


OTTO CHANNEL
Otto Channel
SỞ GIÁO DỤC & ĐÀO TẠO KỲ THI OLYMPIC TRUYỀN THỐNG 30/4
TP. HỒ CHÍ MINH LẦN THỨ XXV – NĂM 2014
ĐỀ CHÍNH THỨC Môn thi: Anh văn - Khối: 10
Thời gian làm bài: 180 phút

➢ Thí sinh làm phần trắc nghiệm (MULTIPLE CHOICE) trên phiếu trả lời trắc nghiệm và phần tự
luận (WRITTEN TEST) trên phiếu trả lời tự luận.
➢ Trên phiếu trả lời trắc nghiệm, thí sinh tô thêm 2 số 00 vào trước số báo danh (bằng bút chì).
Phần mã đề thi trên phiếu trắc nghiệm, thí sinh tô vào ô 002.
 -----------------------------------------------------------------------------------------------------------------

CẤU TRÚC ĐỀ THI


SECTION A. MULTIPLE CHOICE (40PTS) ................................................................................................ 1
PART I. PHONOLOGY (5PTS) ..................................................................................................................... 1
PART II. WORD CHOICE (5PTS)................................................................................................................. 1
PART III. GRAMMAR AND STRUCTURES (5PTS) .................................................................................... 2
PART IV. PHRASAL VERBS AND PREPOSITIONS (5PTS) ..................................................................... 3
PART V. GUIDED CLOZE (10PTS) .............................................................................................................. 3
PART VI. READING COMPREHENSION (10PTS) ..................................................................................... 5

SECTION B. WRITTEN TEST (60PTS) ........................................................................................................ 9


PART I. CLOZE TEST (20PTS) ................................................................................................................... 9
PART II. WORD FORMATION (20PTS) ................................................................................................... 10
PART III. ERROR CORRECTION (10PTS) ................................................................................................ 11
PART IV. SENTENCE TRANSFORMATION (10PTS) .............................................................................. 12
Otto Channel
SỞ GIÁO DỤC & ĐÀO TẠO KỲ THI OLYMPIC TRUYỀN THỐNG 30/4
TP. HỒ CHÍ MINH LẦN THỨ XXV – NĂM 2014
ĐỀ CHÍNH THỨC Môn thi: Anh văn - Khối: 10
Thời gian làm bài: 180 phút

➢ Thí sinh làm phần trắc nghiệm (MULTIPLE CHOICE) trên phiếu trả lời trắc nghiệm và phần tự
luận (WRITTEN TEST) trên phiếu trả lời tự luận.
➢ Trên phiếu trả lời trắc nghiệm, thí sinh tô thêm 2 số 00 vào trước số báo danh (bằng bút chì).
Phần mã đề thi trên phiếu trắc nghiệm, thí sinh tô vào ô 002.
 -----------------------------------------------------------------------------------------------------------------
SECTION A. MULTIPLE CHOICE (40PTS)
PART I. PHONOLOGY (5PTS)
Choose the word whose underlined part is pronounced differently from the others.
1. A. wretchedly B. ruggedly C. confusedly D. determinedly
2. A. slumber B. doubter C. dumbly D. debtor
3. A. expansion B. conversion C. precision D. explosion
4. A. athlete B. ethnic C. asthma D. breath
5. A. finale B. machete C. facsimile D. denote
Choose the word which is stressed differently from the other three.
6. A. magnifier B. preferable C. interpolate D. jeopardy
7. A. horizontal B. adolescent C. panorama D. infrastructure
8. A. photography B. advantageous C. proverbial D. magnificent
9. A. lunatic B. nomadic C. aroma D. heroic
10. A. constituency B. constitution C. presentiment D. subsidiary
PART II. WORD CHOICE (5PTS)
Choose the best options to complete the following sentences.
11. The news of his death was like a bolt
A. from the red B. from the blue C. from the black D. from the white
12. Environmental groups are locked in with the council over the proposed new bypass
through parts of Charmy Wood.
A. argument B. battle C. endowed D. controversy
13. He was with an extraordinary ability.
A. entrusted B. ensured C. debate D. entreated
14. "There is no further treatment we can give", said Dr. John. “We must let the disease take its
.
A. end B. term C. way D. course
15. What on earth made you risk your life and by driving that fast?
A. death B. health C. limb D. liberty
16. During the evening football match, the stadium was illuminated by .
A. flashlights B. floodlights C. highlights D. spotlights

1|Page Tài liệu khóa học Live - VIP


OTTO CHANNEL
Otto Channel
17. Tom admitted that he couldn't concentrate on his work due to this irritating noise.
A. in all B. the bit least C. in the least D. at the least
18. You can buy fake designer clothes much more cheaply than the articles.
A. exact B. real C. true D. genuine
19. Could I pick your on the subject before the meeting?
A. brains B. mind C. head D. intellect
20. Four people drowned when the yacht in a sudden storm.
A. inverted B. overflowed C. upset D. capsized
PART III. GRAMMAR AND STRUCTURES (5PTS)
Choose the best options to complete the following sentences.
21. Everything looks very positive for the company, the current investors do not default on
their agreements.
A. assuming that B. whether C. whereas D. as if
22. came as the injured cat was so weak.
A. Afterwards/dying B. Death/X C. Soon/death D. Then/to die
23. Probably no man had more effect on the daily lives of most people in the United States
.
A. as Henry Ford, a pioneer in automobile production.
B. rather than Henry Ford, a pioneer in automobile production.
C. than did Henry Ford, a pioneer in automobile production.
D. more than Henry Ford, a pioneer in automobile production.
24. The picking of the fruit, takes about a week.
A. whose work they receive no money
B. as they receive no money for that work
C. for which work they receive no money
D. they receive no money for it
25. It was not until they sent me the invitation how meaningful| was to them.
A. that I realized B. I realized C. did I realize D. had I realized
26. I would rather my holiday in Egypt than in Turkey. I really couldn't stand the food.
A. take B had taken C. to have taken D. have taken
27. that we decided to cancel the polo match.
A. Such was a weather C. So terrible a weather was it
B. Such terrible weather was it D. The weather was such
28. Why didn't you tell me you needed help? I you.
A. must have helped C. could help
B. could have helped D. was able to help
29. , he remained optimistic.
A. Though badly wounded he was C. As he was badly wounded
B. As he was Badly wounded D. As badly wounded he was

2|Page Tài liệu khóa học Live - VIP


OTTO CHANNEL
Otto Channel
30. Mark Twain began his career on a newspaper and to be a journalist.
A. long considering himself C. long considered himself
B. himself long considered D. was long himself considered
PART IV. PHRASAL VERBS AND PREPOSITIONS (5PTS)
Choose the best options to complete the following sentences.
31. I can't come to your birthday party because something has .
A. done away with B. come off C. dropped off D. cropped up
32. Many students couldn't , but the teacher the problem by going into a detailed
explanation.
A. catch up/cleared away B. catch up with/cleared off
C. catch on/cleared up D. catch at/clear out
33. The robbers packed diamonds into a suitcase and in a van that waited for them in
the street.
A. put through B. rolled over C. carried away D. made off
34. It is not surprising that he became a writer because he always longed to see his name .
A. in type B. in print C. in letters D. in edition
35. At first they refused but I managed to them round to my way of thinking.
A. put B. force C. push D. bring
36. We put his rude manner ignorance of our British customs.
A. off at B. up with C. up to D. down to
37. Bert is a bit down in the . He was sacked yesterday.
A. dumps B. sad C. low D. bad
38. Thanks to her regular workouts and sensible diet, she certainly strikes me as in the
.
A. blue B. green C. pink D. red
39. What's all this crying ?
A. without the aid of B. with the aid of C. in aid of D. within the aid of
40. Could you reach me that vase from the top shelf?
A. down B. for C. forward D. up for
PART V. GUIDED CLOZE (10PTS)
PASSAGE 1
Read the text below and decide which answer best fits each space.
Red Rock Canyon, part of the Red Rock Recreation Lands in Nevada, is an escarpment of crimson
Aztec sandstone cliffs and canyon walls that (41) the geologic history of the area. Bands of
sediment layers tell a deep-sea bed that 400 million years ago (42) eastward to shoreline in
present - day western Utah. As the ancient sea grew (43) more shallow, about 225 million
years ago, marine limestone and shales were overlain by sediments washed in from emerging land
areas. As the water in the shallow inland sea evaporated, salts and minerals were deposited in thick

3|Page Tài liệu khóa học Live - VIP


OTTO CHANNEL
Otto Channel
beds and fluctuating shorelines created intermixed beds of limestone, shales and minerals. Sediments
from this period gave the canyon its name. Their red color was created from the weathering of iron
(44) within. About 180 million years ago the area was arid and was covered in sand dunes
more than 2.000 feet deep, which became cemented into the Aztec sandstone that is (45) in the
canyon today. Its alternating hues of red, yellow and white are believed to have resulted from
groundwater percolating through the sand and leaching (46) the oxidized iron.
The most significant geologic (47) of the area is the Keystone Thrust Fault, a fracture in
the earth's crust. Sixty- five million years ago, (48) pressure thrust one rock plate over another,
a(n) (49) that can clearly be seen in the contrasting bands of gray limestone and red sandstone
beneath it. The Keystone is one of the most easily identifiable thrust faults to be (50) anywhere.
41. A. enclose B. give C. mention D. reveal
42. A. grew B. evolved C. rose D. stood
43. A. progressively B. painstakingly C. constantly D. precedingly
44. A. complexities B. compounds C. combinations D. composites
45. A. dominating B. sufficient C. prominent D. principal
46. A. out B. into C. against D. with
47. A. character B. feature C. trait D. property
48. A. tense B. intensive C. intense D. extreme
49. A. event B. incident C. phenomenon D. occurrence
50. A. found B. located C. noted D. realized
PASSAGE 2
Read the text below and decide which answer best fits each space.
(51) popular belief, one does not have to be a trained programmer to work online. Of course,
there are plenty of jobs available for people with high-tech computer skills, but the growth of new
media has (52) up a wide range of Internet career opportunities requiring only a minimal level of
technical (53) . Probably one of the most well-known online job opportunities is the job of
webmaster. However, it is hard to define one basic job description for this position. The qualifications
and responsibilities depend on what tasks a particular organization needs a webmaster to (54) .
To specify the job description of a webmaster, one needs to identify the hardware and software
that the website will manage to run (55) . Different types of hardware and software require different
skill sets to manage them. Another key factor is whether the website will be running internally or
externally. Finally, the responsibilities of a webmaster also depend on whether he or she will be working
independently, or whether the firm will provide people to help. All of these factors need to be
considered before one can create requiring (56) knowledge of the latest computer applications.
(57) , there are also online jobs available for which traditional skills remain in high (58)
. Content jobs require excellent writing skills and a good sense of the web as a “new media".
The term "new media" is difficult to define because it encompasses a (59) growing set
of new technologies and skills. Specifically, includes websites, email, Internet technology, CD-ROM,
DVD, streaming audio video, interactive multimedia presentations, e-books, digital music, computer
illustration, video games, (60) reality and computer artistry.

4|Page Tài liệu khóa học Live - VIP


OTTO CHANNEL
Otto Channel
51. A. Apart from B. Contrary to C. Prior to D. In contrast to
52. A. taken B. sped C. set D. opened
53. A. expertise B. master C. efficiency D. excellency
54. A. conduct B. perform C. undergone D. overtake
55. A. on B. over C. in D. with
56. A. built-in B. up-market C. in-service D. in-depth
57. A. However B. Therefore C. Moreover D. Then
58. A. content B. demand C. reference D. requirement
59. A. constantly B. continually C. increasingly D. invariably
60. A. fancy B. imaginative C. illusive D. virtual
PART VI. READING COMPREHENSION (10PTS)
PASSAGE 1
Read the text below and choose the best answer to each question.
THE BALANCE IN THE OCEANS
The oceans’ predators come in all shapes and sizes. For example, one of less infamous ones is the
colorful starfish, which feeds on plant life, coral, or other shellfish such as mussels for sustenance. A
more bloodcurdling example, especially to human beings and most other species of fish, is the shark,
though most scientists agree that only ten per cent of the 450 plus species of sharks have been
documented as actually attacking a human. Still, there is another predator lurking invisibly in the bodies
of water of the world, one which poses one of the greatest threats to all species of ocean life – bacteria.
Though many types of fish are continually stalking and evading one another for survival, they all band
together in an attempt to keep bacteria levels at bay in order to allow their own existence to continue.
Bacteria play a dual role in the ecosystems of the oceans. On the one hand, they are beneficial as they
stimulate plant life through food decomposition, which releases the necessary chemicals for the growth
of plant life. This is called nutrient recycling and helps keep the oceans alive. But, on the other hand,
bacteria are a major predator for all fish because they attack fragile, weaker individuals. If they are
allowed to run rampant and not kept in check, they could virtually suffocate the oceans. In water,
bacteria prove to be an even greater threat than on land because, as they proliferate, they reduced
the oxygen levels necessary for organisms in the oceans to live. Further, when fish populations become
depleted due to factors like overfishing, microbes such as algae expand and threaten the fragile
ecosystems of the ocean. Therefore, ocean predators play a critical role by thwarting bacteria growth
and maintaining the oceans equilibrium by reducing vulnerable links in the food chain.
In many ways, the balance within the oceans’ ecosystems mirrors the human body. That is, all of their
components must work in harmony for them to stay healthy, efficient, and alive. If one of them is
missing or deficient, an entire system can be placed in jeopardy. In both the human body and the
ocean, bacteria play a vital role because, at manageable levels, they aid in protecting and cleaning
each system of foreign agents that can be of harm. On the other hand, if bacteria levels increase and
become out of control, they can take hold of a system, overrun it, and become debilitating. Therefore,
both oceans and the human body have a kind of custodian that maintains bacteria levels. In the human
body, it is called a phagocyte. Phagocytes eat up sick, old, or dying cells, which are more prone to

5|Page Tài liệu khóa học Live - VIP


OTTO CHANNEL
Otto Channel
bacterial invasion, and thus keep the body healthy. Like in the human body, bacteria can prove fatal
to the living organisms in the ocean.
Like phagocytes in the human body ocean predators work as antibacterial custodians of the seas. In
essence, they are the immune system and a vital link in the food chain because they remove small,
injured, or sickly fish from the ocean environment before bacteria can become too comfortable and
multiply. By ridding the ocean of weaker fish, predators allow the stronger ones to multiply, making
their species stronger and more resilient. Without their services and with their declining numbers,
bacteria will blossom to levels that will eventually overpower and kill even the strongest species of fish
because of the depletion of their number one source of life, all important oxygen.
While the greatest battle in the ocean may seem on the surface to be the survival of the fittest fish, a
closer look reveals something completely different: fish versus microorganisms. Clearly, most living
organisms in the oceans are hunters by nature, but this way of life does not merely provide a food
source for a dominant species. It also maintains a healthy level of bacteria in an ocean's ecosystem,
thus ensuring the continuation of all species of life within. Major predators are necessary, like the
antibacterial cells of the human body, to keep this delicate balance in synch. If their numbers continue
to decline and humans ignore their vital role in the ocean, dire consequences will definitely result.
61. The word lurking in the passage is closest in meaning to
A. attacking B. increasing C. waiting D. approaching
62. According to paragraph 1, which of the following is true of ocean predators?
A. The shark is the deadliest one for all other kinds of life in the oceans.
B. One of the most threatening to all fish populations is bacteria.
C. Starfish do little damage to the population of mussels and shellfish.
D. Most of the killers that hide in the oceans are unknown to humans.
63. Which of the following can be inferred from paragraph 1 about bacteria?
A. They can be extremely detrimental to fish if their numbers increase.
B. They are able to feed off themselves when other food sources are limited.
C. They stimulate plant life, which in turn releases oxygen into the water.
D. They present themselves in numerous shapes and forms as well as colors.
64. Which of the sentences below best expresses the essential information in the sentence in bold
("Though... continue")?
A. Evasion tactics help fish escape from the threats posed by an increasing on number of
bacteria.
B. Various species of fish prey upon one another in order to lower bacteria levels in the ocean.
C. High bacteria levels in the ocean help most species of fish to survive by providing them with
food.
D. Rivals or not, all fish help one another survive by preventing bacteria from proliferating.
65. The author discusses nutrient recycling in paragraph 2 in order to .
A. show how bacteria act similarly in the ocean and the human body
B. explain the different roles of nutrients and oxygen for species of fish
C. indicate that bacteria do have a positive impact in the oceans
D. note how chemicals from bacteria are able to stimulate plant growth

6|Page Tài liệu khóa học Live - VIP


OTTO CHANNEL
Otto Channel
66. The word thwarting in the passage is closest in meaning to
A. encouraging B. preventing C. slowing D. sustaining
67. According to paragraph 2, bacteria are dangerous to ocean life because.
A. they have the capability to attack both strong and weaker fish
B. they could monopolize the critical breathable gas in the ocean
C. they get rid of vulnerable links, like dying fish, in the food chain
D. they blossom out of control when overfishing becomes dominant.
68. The word debilitating in the passage is closest in meaning to
A. stimulating B. hindering C. elevating D. weakening
69. The author's description of phagocytes mentions all of the EXCEPT
A. They rid the human body of potentially dangerous organisms.
B. They act in a similar manner as the predators of the ocean.
C. They dispose of bacteria to make weakened cells revive.
D. They are cleaning agents in humans to maintain bacteria levels.
70. According to paragraph 4, the elimination of weaker fish by ocean predator .
A. can often have an adverse effect on the population of the certain prey species
B. inadvertently helps stronger species of fish to proliferate more easily
C. reduces oxygen levels, thereby causing bacteria to multiply in their prey
D. allows bacteria to grow and multiply in the stronger individuals of a species
PASSAGE 2
Read the text below and choose the best answer to each question.
UFFINGTON WHITE HORSE: ANCIENT HILLSIDE CHALK APT
When I was a teenager, I once carved my initials and those of my girlfriend into a tree, something
I thought of at the time as being a permanent statement of our eternal devotion to each other. When
we broke up a year later, I felt obliged to return to the tree, put an X through our initials, and add the
words Null and Void'. The next time I went to find the tree, a number of years after that, it was gone.
My guess is that the tree was so ashamed at having been defaced with was contradictory graffiti that
it simply fell over in act of suicidal protest.
The urge to leave one's mark on the landscape – whether in a tree, a newly poured sidewalk, or
the wall of a cave - goes way, way back. One rather unusual form of ancient markings is found in the
picturesque, pastoral setting of rural England. About a 30-minute drive from the city of Oxford is a
large area covered with the rolling green hills and herds of grazing sheep that have found their way
into countless works of literature and film. Beneath the veneer of grass and soil some of these hills are
made of chalk. And over the millennia, the landscape has become dotted with at least fifty large images
made by carving through the top layers of earth to expose the chalk beneath. Of these, about a dozen
are pictures of horses, and of the horse carvings, the oldest and best known is the Uffington White
Horse.
Although less famous than, say, Stonehenge the Uffington White Horse ranks right up there
among ancient and inexplicable English monuments. It is a highly stylized outline of a horse –
recognizable, but not as well-defined as the other, more solid horse images. The carving is about 374

7|Page Tài liệu khóa học Live - VIP


OTTO CHANNEL
Otto Channel
feet (113m) long, with the lines forming it ranging in width from about 5 to 10 feet (two to three
meters). This particular carving doesn't actually go all the way through the crust to the chalk beneath;
instead, a relatively shallow trench was dug and filled in with to make it almost flush with the surface.
The Uffington White Horse has the distinction of being the largest of Britain's horse carvings
(measured from head to tail). It's also one of only four such horses facing to the right, though no one
knows for sure the significance of the horse's direction, if any. And it's the oldest horse carving,
meaning it may have served as a prototype for the others.
Scientists have determined that the carving is about 3000 years old (give or take a few centuries),
and though it is mentioned in literature dating back to the eleventh century, its original purpose - along
with the identity of its creators- 1s uncertain. Conjecture ranges from a tribal emblem, the equivalent
of a modern flag, to a commemorative symbol of King Alfred's triumphs over the Danes. Or perhaps it
was a Celtic symbol of the goddess Epona, whose job it was to protect horses. It may have been a
territory marker, or simply (perish the thought) giant piece of abstract art. Although it has been referred
to as a "horse" for at least 1,000 years, there are some who believe that it was intended to represent
a dragon. If so, then dragons must have been much more horse-shaped in those days. In any case,
the carving has been well tended over the centuries. Every seven years, weeds are removed and the
outline smoothed to maintain its original size and shape.
One of the most interesting things about the Uffington White Horse is that the only place to get
a good view of the whole thing is from the sky above. There are a few spots several miles away that
provide a fair view of most of the outline, but the local topography is such that there is just no vantage
point from which you can get a good view of the whole horse. This has, predictably, led some people
to speculate that it was created as a signal to UFOs, although what exactly it would signify is a bit
unclear ("Horses for sale – next exit?""). Be that as it may, this peculiarity of perspective must have
made it a challenge to carve, and it makes the horse's original purpose all the more mysterious.
71. The word "pastoral" is closest in meaning to
A. peaceful B. panoramic C. delirious D. unpolluted
72. Why does the writer tell us about the time he carved initials into a tree?
A. To prove that he loved his girlfriend.
B. To tell us about the damage we can do to the landscape.
C. To illustrate our need to preserve the present into the future.
D. To show that nothing lasts forever.
73. The writer mentions the area around Oxford to illustrate that it .
A. has featured in too many books and films.
B. lends itself to the creation of hill carvings.
C. is covered in chalk and grass.
D. is covered in carvings of horses.
74. The horse of Uffington differs in that .
A. it's not as well-known as many of Britain's monuments.
B. nobody knows why it was originally built.
C. it wasn't carved into the chalk like the others.
D. it's the only horse that does not face to the left.

8|Page Tài liệu khóa học Live - VIP


OTTO CHANNEL
Otto Channel
75. The word “trench" is closest in meaning to
A. a shallow pond B. a deep track
C. a long deep hole D. a short bending tunnel
76. The White Horse of Uffington .
A. is the largest carved chalk figure in Britain.
B. may well have inspired similar designs.
C. was first mentioned 3000 years ago.
D. is a few centuries older than originally thought.
77. The true function of the white horse is thought to be .
A. a religious symbol. C. a decorative design.
B. a monument to a victory in battle. D. a mystery.
78. The writer believes .
A. the carving looks more like a horse than a dragon.
B. the horse was originally meant to be a dragon.
C. the carving has changed shape over the years.
D. the original design is restored every seven years.
79. The writer believes that
A. the horse may have been built to attract extra-terrestrial.
B. there's no logical reason to assume the carving was intended for aliens
C. the whole horse can be seen well from several places.
D. it's unclear how the original designers used perspective.
80. The word "Conjecture" is closest in meaning to
A. Belief B. Vision C. Guess D. Saying

SECTION B. WRITTEN TEST (60PTS)


PART I. CLOZE TEST (20PTS)
Use Read the text below and think of the word which best fits each only ONE WORD for each space.
OPEN CLOZE 1
The first two decades of this century were dominated by the microbe hunters. These hunters had
tracked (1) one after another of the microbes responsible for the most dreaded scourges of many
centuries: tuberculosis, cholera, diphtheria. But (2) remained some terrible diseases for which no
microbe could be incriminated: scurvy, pellagra, rickets, beriberi. Then it was discovered that these
diseases were (3) by the lack of vitamins, a trace substance in the diet. The disease could be
prevented or cured by consuming foods that contained the vitamins. And so in the (4) of the
1920's and 1950's, nutrition became a science and the vitamin hunters replaced the microbe hunters.
In the 1940's and 1950's, biochemists strived to learn (5) each of the vitamins was
essential for health. They discovered that key enzymes in metabolism depend on one or (6) of the
vitamins as coenzymes for growth and function. Now, these enzymes hunters occupied center stage.
You are aware that the enzyme hunters have been replaced by a new breed of hunters who are tracking
genes – the blueprints for each of the enzymes and are discovering the defective genes that caused
(7) diseases – diabetes, cystic fibrosis. These gene hunters, or genetic engineers, use

9|Page Tài liệu khóa học Live - VIP


OTTO CHANNEL
Otto Channel
recombinant DNA technology to identify and clone genes and (8) them into bacterial cells
and plants to create factories for the massive production of hormones and (9) for medicine
and for better crops for agriculture. Biotechnology has become a multibillion dollar (10) .
OPEN CLOZE 2
Situated in the central mountains of Alaska, a peak named Denali rises 20,320 feet above sea level.
It is the (11) peak in North America and the center of Denali National park. One of the
America's greatest wilderness areas, the park has had (12) access to visitors, but in spite of
this, tourism rose from 6000 visitors in 1950 to over 546,000 visitors in 1990. The increasing (13)
of this park is prompting serious discussions about the future use of Denali as well as how to
preserve wilderness areas in (14) .
One important issue of land use arises when parts of the National Parks are owned by individuals.
In Denali, (15) most of the land in this vast tract of more than a million acres is owned by
the National Park Service, several thousand acres are still (16) owned as missing tracts.
These mining tracts in Denali were once abundant sources of gold, but they also were sources of (17)
metals such as arsenic and lead that polluted rivers and streams.
Environmentalists were successful in getting the government to require mining companies to
submit statements showing the potential impact of a mining project before they are (18)
to begin mining. Because of this requirement, many individuals closed their mines and some
sold their land to the National Park Service. Some land owners, (19) , are wondering if it is
better to sell their land to the government or keep it for possible future use. Tourism in this previously
remote area is bound to rise, as more roads are built to provide easier access to the park. This increase
in the number of visitors creates a demand for hotels and other real estate development. The economic
implications of this are of (20) to the land owners, but are dismaying to those interested in
preserving the wilderness.
PART II. WORD FORMATION (20PTS)
A. Complete each sentence, using the correct form of the word in parentheses.
1. The sports stadium is the pride of the city. (FUTURE)
2. She is one of those people who never get angry or upset. (PERTURB)
3. Nadal has defeated his Ferrer in the quarter final. (PATRIOTISM)
4. Salt water has more than fresh water. (BUOY)
5. His injury put him out of contention for the championship. (CAPACITY)
6. She the things we need to buy sugar, tea and sandwiches. (NUMBER)
7. , we should stay in a hostel instead of a hotel. (BUDGET)
8. The new BMW has a more powerful engine than its . (PRECEDE)
9. She herself, left the farm and moved to London. (ROOT)
10. Many fish protect their eggs from predators. (GENIUS)

10 | P a g e Tài liệu khóa học Live - VIP


OTTO CHANNEL
Otto Channel
B. Complete the passage with appropriate forms from the word given in the box.

south lie exploit intimacy ornament


hill migrate front conserve originate
The whooping crane is a grand, (11) waterfowl native only to North America. Its features
are striking, as a mature whooping crane is all white and stands five feet tall with a wingspan of about
eight feet. Previously wild flocks spent the warmer parts of the year in their native habitat of northern
Canada, and, around August, they made the 2,500-mile (12) journey to the gulf coast of Texas
in the United States. Today, only one flock remains in the wild, and it consists of about one hundred
and fifty to two hundred whooping cranes. Continuing (13) efforts are attempting to increase
its populations, which face, as they traditionally have, a/an (14) battle, by introducing new
habitats and educational methods of migration.
There are two major factors which (15) the decline of the whooping crane in the early
twentieth century. First, they were (16) hunted by people for food as well as beautiful
feathers, which were used for (17) purposes. Second, their natural habitat wetlands were
beginning to dry up due to agricultural (18) and development. Thirdly, which perhaps had the
most dramatic effect, was the fact that the eggs of the whooping crane were prized by collectors. Once
the eggs were pillaged from nests, future generations became placed even more in jeopardy.
Fortunately, the whooping crane, like other endangered species such as the bald eagle, (19)
became a protected species by federal law. The issue of how to increase their numbers back to
comfortable levels, therefore, has moved to the (20) of attention.

PART III. ERROR CORRECTION (10PTS)


The following passage contains 10 errors. Identify and correct them.
1 Since water is the basis of life, composing the greater part of the tissues of all living things,
2 the crucial problem of desert animals is to survive in a world where sources of flowing
3 water are rare. And since man's inexorably necessity is to absorb large quantities of water
4 in frequent intervals, he can scarcely comprehend what many creatures of the desert pass
5 their entire lives without a single drop.
6 Uncompromising as it is, the desert has not eliminated life but only unable to withstand
7 its desiccating effects. No moist-skinned, water-loving animals can exist there. Few large
8 animals are found. The giants of the North American desert are the deer, the coyote, and
9 the bobcat. Since desert country is open, it holds more swift-footed running and leaping
10 creations than the tangled forest. Its population is hugely nocturnal, silent, filled with
11 reticence, and ruled stealth. Yet they are not emaciated.
12 Having adapted to their austere environment, they are as healthful as animals anywhere
13 else in the world. The secret of their adjustment lies in the combination of behavior and
14 physiology. Nobody could survive if, like mad dogs and Englishmen, they go out in the
15 midday sun; many would die in a while of minutes. So most of pass the burning hours
16 asleep in cool, humid burrows underneath the ground, emerging to hunt only by night.
17 The coverage of the sun-baked desert averages around 150 degrees, but 18 inches down
18 the air is only 60 degrees.

11 | P a g e Tài liệu khóa học Live - VIP


OTTO CHANNEL
Otto Channel
Lines Mistake Correction Lines Mistake Correction

PART IV. SENTENCE TRANSFORMATION (10PTS)


Rewrite the following sentences using the words given.
1. Richard only took over the family business because his father decided to retire early.
→ Had ................................................................................................................................
2. We would always take great care when flying at night. (WITS)
→ We always used ..............................................................................................................
3. There haven't been such long queues at the cinema since the release of the last blockbuster.
→ Not .................................................................................................................................
4. I will only be satisfied if the manager apologizes fully.
→ Nothing short ..................................................................................................................
5. She was concentrating so hard on her work that she didn’t notice when I came in. (WRAPPED)
→ She was so .....................................................................................................................
6. The price of the house has been reduced as much as possible because the owner needs some
money. (BONE)
→ The house .......................................................................................................................
7. The permit expires at the end of the month.
→ The permit is not..............................................................................................................
8. The first sign of the disease is blurred vision. (ONSET)
→ .......................................................................................................................................
9. Don't say anything negative about her hair because she's very sensitive and might be offended
by your remarks. (OFFENCE)
→ .......................................................................................................................................
10. The new teacher was so nervous that the class reacted mischievously. (REACTION)
→ The new teacher's ............................................................................................................
THE END

12 | P a g e Tài liệu khóa học Live - VIP


OTTO CHANNEL
Otto Channel
SỞ GIÁO DỤC & ĐÀO TẠO KỲ THI OLYMPIC TRUYỀN THỐNG 30/4
TP. HỒ CHÍ MINH LẦN THỨ XXV – NĂM 2016
ĐỀ CHÍNH THỨC Môn thi: Anh văn - Khối: 10
Thời gian làm bài: 180 phút

➢ Thí sinh làm phần trắc nghiệm (MULTIPLE CHOICE) trên phiếu trả lời trắc nghiệm và phần tự
luận (WRITTEN TEST) trên phiếu trả lời tự luận.
➢ Trên phiếu trả lời trắc nghiệm, thí sinh tô thêm 2 số 00 vào trước số báo danh (bằng bút chì).
Phần mã đề thi trên phiếu trắc nghiệm, thí sinh tô vào ô 002.
 --------------------------------------------------------------------------------------------------------------------

CẤU TRÚC ĐỀ THI


SECTION A. MULTIPLE CHOICE (40 PTS) ........................................................................ 1
PART I. PHONOLOGY (5PTS) ..................................................................................................................... 1
PART II. WORD CHOICE (5PTS)................................................................................................................. 1
PART III. GRAMMAR AND STRUCTURES (5PTS) .................................................................................... 2
PART IV. PHRASAL VERBS AND PREPOSITIONS (5PTS) ..................................................................... 3
PART V. GUIDED CLOZE (10PTS) .............................................................................................................. 3
PART VI. READING COMPREHENSION (10PTS) ..................................................................................... 5

SECTION B. WRITTEN TEST (60PTS) ............................................................................... 8


PART I. CLOZE TEST (20 PTS) .................................................................................................................. 8
PART II. WORD FORMATION (20PTS) ..................................................................................................... 9
PART III. ERROR CORRECTION (10PTS) ................................................................................................ 10
PART IV. SENTENCE TRANSFORMATION (10 PTS) ............................................................................. 10
Otto Channel
SỞ GIÁO DỤC & ĐÀO TẠO KỲ THI OLYMPIC TRUYỀN THỐNG 30/4
TP. HỒ CHÍ MINH LẦN THỨ XXV – NĂM 2016
ĐỀ CHÍNH THỨC Môn thi: Anh văn - Khối: 10
Thời gian làm bài: 180 phút

➢ Thí sinh làm phần trắc nghiệm (MULTIPLE CHOICE) trên phiếu trả lời trắc nghiệm và phần tự
luận (WRITTEN TEST) trên phiếu trả lời tự luận.
➢ Trên phiếu trả lời trắc nghiệm, thí sinh tô thêm 2 số 00 vào trước số báo danh (bằng bút chì).
Phần mã đề thi trên phiếu trắc nghiệm, thí sinh tô vào ô 002.
 --------------------------------------------------------------------------------------------------------------------
SECTION A. MULTIPLE CHOICE (40 PTS)
PART I. PHONOLOGY (5PTS)
Choose the word whose underlined part is pronounced differently from the others.
1. A. epoch B. lichen C. archipelago D. chemise
2. A. hatred B. allegedly C. deluged D. dogged
3. A. complacent B. preface C. menace D. palace
4. A. convalescence B. crescendo C. effervescent D. scenic
5. A. massage B. pilgrimage C. pillage D. dosage
Choose the word which is stressed differently from the other three.
6. A. commentator B. salbutamol C. netiquette D. predecessor
7. A. procrastinate B. jeopardize C. hibernate D. frivolous
8. A. hermaphrodite B. telepathy C. kaleidoscope D. nevertheless
9. A. discrepancy B. convivial C. gimmickry D. naturalistic
10. A. conscientious B. officialese C. consanguinity D. irrevocable
PART II. WORD CHOICE (5PTS)
Choose the best options to complete the following sentences.
11. All his hard work in great success.
A. accounted B. culminated C. merged D. succumbed
12. She's certainly a writer; she has written quite a few books this year.
A. fruitful B. prolific C. fertile D. successful
13. The contemporary dialogue for me struck a slightly note.
A. disembodied B. discordant C. dismissive D. disconcerting
14. We decided to celebrate by going out and painting the town .
A. red B. purple C. gold D. brown.
15. Not being able to find my phone number is a pretty excuse for not contacting me.
A. fragile B. frail C. feeble D. faint
16. The new airport was constructed in the of fierce opposition from environmentalists.
A. face B. teeth C. tooth and nail D. fangs
17. According to the of the contract, tenants must give six months' notice if they intend to live.
A. laws B. rules C. terms D. details

1|Page Tài liệu khóa học Live - VIP


OTTO CHANNEL
Otto Channel
18. She wasn't allowed into the country, because her parents aren't in order.
A. subsequently B. presumably C. admittedly D. paradoxically
19. Both of the jobs I've been offered are fantastic opportunities - I'm in such .
A. a dwelling B. a grudge C. an array D. a quandary
20. The Red Cross is an international aid organization.
A. intriguingly B. intrusively C. intrinsically D. intrepidly
PART III. GRAMMAR AND STRUCTURES (5PTS)
Choose the best options to complete the following sentences.
21. Up , and the people cheered.
A. went the balloon C. had the balloon
B. did the balloon go gone D. the balloon went
22. Marine reptiles are among the few creatures that are known to have a possible life span greater
than .
A. man B. the man C. the one of the man’s D. that of man
23. He seems for the experience.
A. none worse B. none the worse
C. none worse at all D. none the worst
24. The university's programs .
A. only comes second after Harvard
B. are second only to those of Harvard
C. are first except for Harvard's
D. are second place from that of Harvard
25. You promised to have a day off and go camping with us and you were at work until midnight
this Sunday. I would rather you your word.
A. keep B. would have kept C. had kept D. kept
26. Business has been thriving for the past few years. Long it continue to do so.
A. could B. does C. may D. might
27. All the way from Leeds .
A. he came B. came he C. did he come D. comes he
28. The first people to live in Hawaii were the Polynesians, who sailed there in large canoes from
other Pacific Islands about 2,000 years ago.
A. now where is B. what is now C. the place is now D. now this is
29. It was asked that the traffic laws strictly.
A. must be observed C. be observed
B. had to be observed D. were observed
30. Nobody understands what the strange man has said, ?
A. do they B. don't they C. doesn't he D. does he

2|Page Tài liệu khóa học Live - VIP


OTTO CHANNEL
Otto Channel
PART IV. PHRASAL VERBS AND PREPOSITIONS (5PTS)
31. It that the mayor had bribed several councilors to vote.
A. broke out B. pointed at C. came round D. turned out
32. They are two a kind, talkative and lazy.
A. for B. with C. in D. of
33. I wish you'd telling me what to do all the time.
A. lay off B. let out C. take in D. go with
34. When the professor advanced his theory at a medical conference a couple of years ago,
everyone was skeptical about its validity. However, the latest research has it
A. borne / out B. carried / out C. stood / for D. cleared / at
35. The Russians treat him suspiciously, so he the Russian fortress in order to return to
the mountains.
A. gets away with B. breaks out of C. cracks down on D. holds out for
36. Creative writers are largely endowed the gift of writing.
A. with B. of C. on D. at
37. There's been a bomb scare and the police have the area.
A. made out B. put down C. rambled on D. cordoned off
38. hindsight, it would have been better to wait.
A. On B. For C. From D. In
39. He tried to the country's deep-seated problems.
A. paper over B. pencil in C. screen off D. box in
40. He was very upset when the boss passed him and promoted a newcomer to the
assistant’s job.
A. by B. up C. over D. aside
PART V. GUIDED CLOZE (10PTS)
Read the text below and decide which answer best fits each space.
PASSAGE 1
As computers have become (41) tools for the rapid and economic production of pictures,
computer graphics has emerged as one of the most rapidly growing fields in computer science. It is
used routinely in such (42) areas business, industry, art, government, education, research,
training, and medicine.
One of the initial uses of computer graphics, and ultimately its greatest use, has been as an aid to
design, generally (43) to as computer - aided design (CAD). One of its greatest advantages is
(44) designers can see how an object will look after construction and make changes freely
and much more quickly than (45) hand drafting.
For three-dimensional (46) of machine parts, engineers now rely heavily on CAD.
Automobile, spacecraft, aerospace, and ship designers use CAD techniques to design vehicles and (47)
their performance. Building designs are also created with computer graphics systems.

3|Page Tài liệu khóa học Live - VIP


OTTO CHANNEL
Otto Channel
Architects can design a building (48) , create a three-dimensional model, and even go for a
simulated "walk" through the rooms or around the outside of the building.
Business graphics is another rapidly growing area of computer graphics, where it is used to create
graphs, charts, and cost models to summarize financial, statistical, mathematical, scientific, and
economic data. As an educational aid, computer graphics can be used to create weather maps and
cartographic materials. Computer art also has creative and commercial art applications, where it is
used in advertising, publishing, and film productions, (49) for computer animation, which is
achieved by a (50) process.
41. A. handful B. overbearing C. controlling D. powerful
42. A. widening B. diverse C. versatile D. variant
43. A. resorted B. inferred C. implied D. referred
44. A. whether B. as C. that D. then
45. A. for B. with C. of D. over
46. A. rendering B. severing C. hindering D. pondering
47. A. try B. test C. experiment D. challenge
48. A. fallout B. layout C. breakout D. workout
49. A. specifically B. specially C. particularly D. definitely
50. A. consecutive B. connective C. sequential D. successive

PASSAGE 2
Reading is an activity characterized by the translation of symbols, or letters, into words and
sentences that have (51) to the individual. The (52) goal of reading is to be able to
understand written material, to evaluate it, and to use it for one's needs.
In order to read, one must follow a sequence of characters arranged in a particular spatial order.
For example, English flows from left to right, Hebrew from right to left, and Chinese from top to bottom.
The reader must know the pattern and use it (53) . Ordinarily, the reader sees the symbols on a
page, (54) the image from the eye to the brain, but reading also can be accomplished (55)
touch, as in the Braille system, a printing method for the blind that involves raised or (56)
dots.
Reading refers to activities as varied as a first grader's struggling with simple sentences in a
storybook, a cook's following (57) from a cookbook, or a scholar's attempting to understand the
meanings of a poem. Reading (58) people to the accumulated wisdom of human civilization.
Mature readers bring to the text their experiences, abilities, and interests; the text, in turn, allows them
to (59) those experiences and abilities and to find new interests. To reach maturity in reading,
an individual goes through a (60) of stages, from readiness to adult reading ability.

51. A. significance B. importance C. meaning D. inference


52. A. final B. ultimate C. hindmost D. eventual
53. A. consistently B. persistently C. insistently D. intently
54. A. transmitting B. transferring C. transiting D. transporting
55. A. within B. in C. by D. through
56. A. holed B. drilled C. punched D. pierced

4|Page Tài liệu khóa học Live - VIP


OTTO CHANNEL
Otto Channel

57. A. orientation B. tutorials C. guidances D. directions


58. A. presents B. carries C. exposes D. drives
59. A. expand B. extend C. enlarge D. stretch
60. A. flow B. series C. chain D. course
PART VI. READING COMPREHENSION (10PTS)
Read the text below and choose the best answer to each question.
PASSAGE 1
The air above our head is becoming cleaner. A breath of fresh air has been running right round the
planet for the past five years. The planet is apparently purging itself of pollution. Paul Novell of the
University of Colorado, the co-author of a report on this phenomenon says, "It seems as if the planet's
own cleansing service has suddenly got a new lease of life. Suddenly, there are a lot of changes going
on up there.”
Estimates of the death toll from urban smog’s have been steadily rising, so the new cleaner trend
could have significant consequences for life expectancy in cities as well as for the planet itself. The
sudden and unexpected reversal of several decades of worsening pollution extends from the air in city
streets to the remotest mid-Pacific Ocean and Antarctica.
Among the pollutants which have begun to disappear from the atmosphere are carbon monoxide,
from car exhausts and burning rain forests, and methane from the guts of cattle, paddy fields, and gas
fields. Even carbon dioxide, the main gas behind global warming, has fallen slightly.
There are two theories about why pollution is disappearing. First, that there is less pollution to start
with due to laws to cut down urban smog’s and acid rain starting to have a global impact. Second, that
the planet may be becoming more efficient at cleaning up.
The main planetary clean-up agent is a chemical called hydroxyl. It is present throughout the
atmosphere in tiny quantities and removes most pollutants from the air by oxidizing them. The amount
of hydroxyl in the air had fallen by a quarter in the 1980s. Now, it may be reviving for two reasons:
ironically, because the ozone hole has expanded, letting in more ultraviolet radiation into the lower
atmosphere, where it manufactures hydroxyl. Then the stricter controls on vehicle exhausts in America
and Europe may have cut global carbon monoxide emissions, thereby allowing more hydroxyl to clean
up other pollutants.
61. What is the main topic of the passage?
A. The decreasing pollution of the atmosphere. B. The changing pollutants in the atmosphere.
C. Hydroxyl's influence on the atmosphere. D. The oxygenation of the atmosphere.
62. The word 'purging' is closest in meaning to
A. destroying B. refining C. filtering D. ridding
63. According to the passage, life expectancy partly depends on people having.
A. access to details about atmospheric pollution
B. recommendations from university research
C. improvement in atmospheric conditions
D. changes in their lifestyle

5|Page Tài liệu khóa học Live - VIP


OTTO CHANNEL
Otto Channel
64. The word 'toll' could best be replaced by other pollutants.
A. costs B. count C. damage D. loss
65. What does the author suggest is the main cause of pollution reduction?
A. Less impact from burning forests B. A smaller number of cars
C. A curtailment of chemicals D. Fewer cattle and gas fields
66. The word 'it' refers to .
A. urban smog B. a clean-up agent C. acid rain D. the ozone hole
67. It can be inferred from the passage that the new cleaner trend is
A. inexplicable B. confusing C. surprising D. predictable
68. Based on information in the passage, all of the following information referring to hydroxyl is
true EXCEPT.
A. the reduction in the ozone layer is beneficial to hydroxyl
B. oxidization of pollutants is carried out by hydroxyls
C. there is difficulty in destroying carbon dioxide by hydroxyl
D. ultraviolet radiation increases production of hydroxyl
69. The word reviving' is closest in meaning to
A. reappearing B. refreshing C. reproducing D. repeating
70. The passage supports which of the following conclusions?
A. The decrease of methane has enabled ultraviolet radiation to enter the atmosphere.
B. An expansion in hydroxyl has enlarged the ozone hole.
C. The reduction in carbon dioxide has produced a cleaner atmosphere.
D. The beneficial effect of hydroxyl has aided the cleaning process.
PASSAGE 2
We find that bright children are rarely held back by mixed-ability teaching. On the contrary, both
their knowledge and experience are enriched. We feel that there are many disadvantages in streaming
pupils. It does not take into account the fact that children develop at different rates. It can have a bad
effect on both the bright and the not-so-bright child. After all, it can be quite discouraging to be at the
bottom of the top grade!
Besides, it is rather unreal to grade people just according to their intellectual ability. This is only
one aspect of their total personality. We are concerned to develop the abilities of all our pupils to the
full, not just their academic ability. We also value personal qualities and social skills, and we find that
mixed-ability teaching contributes to all these aspects of learning.
In our classrooms, we work in various ways. The pupils often work in groups: this gives them the
opportunity to learn to co-operate, to share, and to develop leadership skills. They also learn how to
cope with personal problems as well as learning how to think, to make decisions, to analyze and
evaluate, and to communicate effectively. The pupils learn from each other as well as from the
teachers.
Sometimes the pupils work in pairs; sometimes they work on individual tasks and assignments, and
they can do this at their own speed. They also have some formal class teaching when this is
appropriate. We encourage our pupils to use the library, and we teach them the skills they need in

6|Page Tài liệu khóa học Live - VIP


OTTO CHANNEL
Otto Channel
order to do this effectively. An advanced pupil can do advanced work; it does not matter what age the
child is. We expect our pupils to do their best not their least, and we give them every encouragement
to attain this goal.
71. In the passage, the author's attitude towards “mixed-ability teaching" is
A. critical B. questioning C. objective D. approving
72. The words "held back" means
A. made to remain in the same classes B. prevented from advancing
C. forced to study in lower classes D. badly affected in studying
73. The author argues that a teacher's chief concern should be the development of the pupils'.
A. personal and social skills B. learning ability and communicative skills
C. intellectual abilities D. total personality
74. Which of the following is NOT mentioned in the passage?
A. Pupils also learn how to participate in teaching activities.
B. Group work gives pupils the opportunity to learn to work together with others.
C. Pupils also learn to develop their reasoning ability.
D. Group work provides the pupils with the opportunity to learn to be capable organizers.
75. The author's purpose of writing this passage is to .
A. recommend pair work and group work classroom activities
B. offer advice on the proper use of the school library
C. argue for teaching bright and not-so-bright pupils in the same class
D. emphasize the importance of appropriate formal classroom teaching
76. According to the passage, which of the following is NOT TRUE?
A. It's not good for a bright child to find out that he performs worst in a mixed-ability class.
B. Development of pupils as individuals is not the aim of group work.
C. Pupils cannot develop in the best way if they are streamed into classes of different intellectual
abilities.
D. There is no fixed method in teaching pupils to develop themselves to the full.
77. According to the passage, which of the following is an advantage of mixed- ability teaching?
A. Pupils as individuals always have the opportunities to work on their own.
B. Pupils can be hindered from an all-round development.
C. A pupil can be at the bottom of a class.
D. Formal class teaching is the important way to give the pupils essential skills such as those to
be used in the library.
78. Which of the following statements can best summarize the main idea of the passage?
A. Children, in general, develop at different rates.
B. The aim of education is to find out how to teach the bright and not-so- bright pupils.
C. Bright children do benefit from mixed-class teaching.
D. Various ways of teaching should be encouraged in class.

7|Page Tài liệu khóa học Live - VIP


OTTO CHANNEL
Otto Channel
79. According to the passage, "streaming pupils".
A. will help the pupils learn best
B. is the act of putting pupils into classes according to their academic abilities
C. aims at enriching both their knowledge and experience
D. is quite discouraging
80. According to the author, mixed-ability teaching is more preferable because.
A. it doesn't have disadvantages as in streaming pupils
B. children can learn to work with each other to solve personal problems
C. it aims at developing the children's total personality
D. formal class teaching is appropriate

SECTION B. WRITTEN TEST (60PTS)


PART I. CLOZE TEST (20 PTS)
Read the text below and think of the word which best fits each space. Use only ONE WORD for each
space.
PASSAGE 1
In the theater of Ambassador Hotel in Los Angeles, on the evening of 27 September 1922, a new
form of film-making (1) its commercial debut: 3-D1. The film, The Power of Love, was then shown
in New York City to exhibitors and press, but was subsequently not (2) up for distribution and is
now believed to be lost. The following three decades were a period of quiet experimentation for 3-D
pioneers, as (3) adapted to new technologies and steadily improved the viewing experience. In
1952, the "golden era" of 3-D is considered to have begun (4) the release of Bwana Devil, and
over the next several years, audiences met with a string of films that used the technology. Over the
following decades, it (5) and waned within film-making circles, peaking in the 1970s and again in
the 1990s when IMAX gained traction, but it is only in the last few years (6) 3-D appears to
have firmly entered mainstream production.
Released worldwide in September 2009, the fantasy film Avatar quickly became the highest-
grossing film (7) made, knocking Titanic from the top slot. Avatar, set in 2154 on a planet
in a distant solar system, went on to become the only film to have earned US $2 billion worldwide, and
now approaching the $3 billion mark. The main (8) for this runaway popularity appears to be its
visual splendors; though most (9) raised the film, it was mostly on account of its ground- is
breaking special effects. Kenneth Turan of the Los Angeles Times praised Avatar's "powerful" visual
accomplishments, (10) suggested the dialogue was “flat" and the characterizations “obvious". A
film analyst at Exhibitor Relations has agreed, noting that Avatar has cemented the use of 3-D as a
production and promotional tool for blockbuster films, rather than a mere niche or novelty experiment.
"This is why all 3-D venues were built," he said. "This is the one. The behemoth... The holy grail of 3-
D has finally arrived."
PASSAGE 2
Since the world became industrialized, the number of animal species that have either become
extinct or have (11) extinction has increased. Bengal tigers, for instance, which (12)
roamed the jungles (13) vast numbers, now number only about 2,300. By the year 2025, it is
estimated that they will become extinct.
8|Page Tài liệu khóa học Live - VIP
OTTO CHANNEL
Otto Channel
(14) is alarming about the case of the Bengal tiger is that extinction will have been caused
(15) entirely by poachers who, according to some sources, are not always interested in
material (16) but in personal gratification. Animals such as the Bengal tiger are valuable parts
of the world's ecosystem. International laws protecting these animals must be enacted to ensure their
survival - and the survival of our planet.
Countries around the world have begun to deal with the problem in various ways. Some countries,
in an effort to circumvent the problem, have allocated large amounts of land to animal reserves. They
then charge admission prices to (17) defray the costs of maintaining the parks, and they often
must also depend on world organizations for (18) support. This money enables them to invest in
equipment and patrols to protect the animals. Another response to the increase in animal extinction is
an international boycott of (19) made from endangered species. This has had some effect, but by
(20) it will not prevent animals from being hunted and killed.
PART II. WORD FORMATION (20PTS)
A. Complete each sentence, using the correct form of the word in parentheses.
1. She became well-known as a consistent of slavery. (OPPOSE)
2. is a list of errors in a printed work as a separate page of corrections, known as an errata
page. (CORRECT)
3. Many scientists argue that dinosaurs developed extensive passages with membranes to
cool their skin surfaces. (NOSE)
4. Premature disclosure of the test sites might lead to of the experiment. (VALID)
5. He became in his opposition in the plan. He gave up viewpoints after his boss criticized.
(RESOLVE).
6. He was returned to prison in 1977 for a police officer. (PERSON)
7. The soldiers have been ordered to from firing the guns. (INSIST)
8. I lingered in San Francisco, myself after the hardships of the cruise, spending money,
regretting it, continually promising departure for the morrow. (DAMAGE)
9. One of the cars involved in the accident was a complete . (WRITE)
10. The work is marred by a number of typing and spelling errors, and would have benefited from
stricter of the before submission. (READ)
B. Complete the passage with appropriate forms from the words given in the box.

alien stay throw quietening foundation


type wither front just tribute

SUBVERSIVE ART
Subversive political and social messages are a (11) of the art community. In most modern
democratic nations, where free speech is considered and (12) right, such subversive art is easily
found. American film is an (13) of this. Many American films are openly critical of American
society and its values. Apocalypse Now, a film about the Vietnam War, is a (14) attack on America's
involvement and conduct in that conflict. In a more modern setting, African-American film director

9|Page Tài liệu khóa học Live - VIP


OTTO CHANNEL
Otto Channel
Spike Lee makes films which depict America as a racist and (15) society. Other arts are filled with
examples of subversive art as well. There are countless paintings and sculptures which openly attack
Christianity and other values which are considered to be core aspects of America's identity. Punk Rock,
which originated in England derided many of England's most sacred of social values. Most famously,
the 1970s punk band The Sex Pistols frequently sang songs about (16) the government, even
going in so far as to describe the Queen of England as subhuman.
Not all subversive art is as (17) its social criticism. Much of subversive art is (18)
subversive. There are a number of reasons for this. The first is, of course, economics. An artist who is
too open or too vehement in his or her criticism of society may find it hard to find an audience willing
to pay for such art. In some societies there may be a fear of (19) , either by the government or by
individual society member offended by the artist's message. Nor are such fears (20) . Author
Salman Rushdie was forced to go into hiding after he wrote a book that Muslims took to be critical of
their religion. Therefore, the subversive messages in art are often hidden.
PART III. ERROR CORRECTION (10PTS)
The following passage contains 10 errors. Identify and correct them.
1 The weight and pressure of icy accumulation causes glacier movement. Glaciers move out
2 from under them, via plastic deformation and basal slippage. First, the internal flow of ice
3 crystals begins to spread outward and downward from the thickened snow pack also known
4 as the zone of accumulation. Next, the ice along the ground surface begins to slip on the
5 same direction. Seasonal thawing at the basis of the glacier helps facilitating this slippage.
6 The middle of a glacier moves faster than the sides and bottom because there was no rock
7 to cause any friction. The above part of a glacier rides on the ice below. As a glacier moves
8 it carves out a U-shape valley to a riverbed, but with much steeper walls and flat bottom.
9 Beside the extraordinary rivers of ice, glacial erosion creates other unique physical features
10 in the landscape such as horns, fjords, hanging valleys and cirques.

Lines Mistake Correction Lines Mistake Correction

PART IV. SENTENCE TRANSFORMATION (10 PTS)


Rewrite the following sentences using the words given.
1. He really upset everyone when he told them his secret. (PIGEONS)
→ He ................................................................................................................................
2. Your attitude will have to change if you want to succeed. (LEAF)
→ You will ........................................................................................................................
3. You will you are talented but horses will always show you are not. (PEG)
→ No matter .....................................................................................................................
4. I avoided mentioning Jack's demotion as I realized that it might upset him. (SLEEPING)
→ Realizing ........................................................................................................................

10 | P a g e Tài liệu khóa học Live - VIP


OTTO CHANNEL
Otto Channel
5. Susan is far superior to me in terms of technical knowledge. (MATCH)
→ When it comes
6. It is important that we have to get the team to comply with the rules. (LINE)
→ It is of ..........................................................................................................................
7. Winning the prize has made him very conceited. (HEAD)
→ Winning ........................................................................................................................
8. Prompt action from the police helped to reduce the number of people killed and injured in the
fighting. (SUFFERED)
→ Were it .................................................................................................. casualties.
9. His efforts to find a solution didn't deserve such a savage criticism.
→ He ..........................................................................................................to find a solution.
10. He did not succeed in reaching the top of the highest mountain in this area.
→ In vain ..........................................................................................................................
THE END

11 | P a g e Tài liệu khóa học Live - VIP


OTTO CHANNEL
Otto Channel
SỞ GIÁO DỤC & ĐÀO TẠO KỲ THI OLYMPIC TRUYỀN THỐNG 30/4
TP. HỒ CHÍ MINH LẦN THỨ XXV – NĂM 2017
ĐỀ CHÍNH THỨC Môn thi: Anh văn - Khối: 10
Thời gian làm bài: 180 phút

➢ Thí sinh làm phần trắc nghiệm (MULTIPLE CHOICE) trên phiếu trả lời trắc nghiệm và phần tự
luận (WRITTEN TEST) trên phiếu trả lời tự luận.
➢ Trên phiếu trả lời trắc nghiệm, thí sinh tô thêm 2 số 00 vào trước số báo danh (bằng bút chì).
Phần mã đề thi trên phiếu trắc nghiệm, thí sinh tô vào ô 002.
 --------------------------------------------------------------------------------------------------------------------

CẤU TRÚC ĐỀ THI


SECTION A. MULTIPLE CHOICE (40 PTS) ........................................................................ 1
PART I. PHONOLOGY (5PTS) ..................................................................................................................... 1
PART II. WORD CHOICE (5PTS)................................................................................................................. 1
PART III. GRAMMAR AND STRUCTURES (5PTS) .................................................................................... 2
PART IV. PHRASAL VERBS AND PREPOSITIONS (5PTS) ..................................................................... 3
PART V. GUIDED CLOZE (10PTS) .............................................................................................................. 3
PART VI. READING COMPREHENSION (10PTS) ..................................................................................... 5

SECTION B. WRITTEN TEST (60 PTS) .............................................................................. 8


PART I. CLOZE TEST (20 PTS) .................................................................................................................. 8
PART II. WORD FORMATION (20 PTS) .................................................................................................... 9
PART III. ERROR CORRECTION (10 PTS) ............................................................................................... 10
PART IV. SENTENCE TRANSFORMATION (10 PTS) ............................................................................. 10
Otto Channel
SỞ GIÁO DỤC & ĐÀO TẠO KỲ THI OLYMPIC TRUYỀN THỐNG 30/4
TP. HỒ CHÍ MINH LẦN THỨ XXV – NĂM 2017
ĐỀ CHÍNH THỨC Môn thi: Anh văn - Khối: 10
Thời gian làm bài: 180 phút

➢ Thí sinh làm phần trắc nghiệm (MULTIPLE CHOICE) trên phiếu trả lời trắc nghiệm và phần tự
luận (WRITTEN TEST) trên phiếu trả lời tự luận.
➢ Trên phiếu trả lời trắc nghiệm, thí sinh tô thêm 2 số 00 vào trước số báo danh (bằng bút chì).
Phần mã đề thi trên phiếu trắc nghiệm, thí sinh tô vào ô 002.
 --------------------------------------------------------------------------------------------------------------------
SECTION A. MULTIPLE CHOICE (40 PTS)
PART I. PHONOLOGY (5PTS)
Choose the word whose underlined part is pronounced differently from the others.

1. A. decoy B. perishable C. benzene D. supreme


2. A. mayor B. quay C. prayer D. layer
3. A. mahout B. foul C. poultry D. drought
4. A. hombre B. hauteur C. heirloom D. haulage
5. A. simile B. anemone C. acne D. acquiesce
Choose the word which is stressed differently from the other three.

6. A. xenophobia B. salmonberry C. palindrome D. moderator


7. A. horoscope B. mahogany C. deplorable D. prerequisite
8. A. interchange B. infamous C. contributory D. undercurrent
9. A. culminate B. negligence C. diplomat D. intriguing
10. A. viticulture B. concertina C. preferential D. misbegotten
PART II. WORD CHOICE (5PTS)
Choose the best options to complete the following sentences.
11. Because of the unfortunate , your order was not dispatched by the date requested.
A. hindrance B. oversight C. negligence D. transgression
12. Failing to submit the proposal on time was for Tom.
A. a nail in the coffin B. a real kick in the pants
C. a shot in the dark D. an open and shut case
13. Be careful not to your finger with that needle.
A. prick B. bite C. scratch D. sting
14. The jury her compliments on her excellent knowledge of the subject.
A. paid B. gave C. made D. said
15. His new yacht is certainly an display of his wealth.
A. ostentatious B. ossified C. intuitive D. elusive
16. I don't need any medicine. I'm as right as
A. clouds B. rays C. rain D. a haze

1|Page Tài liệu khóa học Live - VIP


OTTO CHANNEL
Otto Channel
17. They continued fighting despite all the they met with.
A. amenities B. properties C. liabilities D. adversities
18. I appealed to all people to support me and I was successful.
A. same-minded B. thought-sharing C. familiar-thinking D. like-minded
19. He seemed very quiet, but it would be a mistake to his intelligence.
A. devalue B. deprecate C. underrate D. minimize
20. With all respect, I think your comments are very short-sighted.
A. anticipated B. due C. limited D. firm
PART III. GRAMMAR AND STRUCTURES (5PTS)
Choose the best options to complete the following sentences.
21. Your ideas, , seem unusual to me.
A. as hers B. like hers C. similar as hers D. different than hers
22. It is moved that the campaign funds at once.
A. to raise / be launched B. raise / to be launched
C. raise / launched D. to raise / is launched
23. A new generation of performers, to those who by now had become a household name,
honed their skills before following the same path onto television.
A. no less talented than C. together with talented as funds at once.
B. along with talented as D. having more talented than
24. , he remained optimistic.
A. Though badly wounded he was C. As he was badly wounded
B. Badly wounded as he was D. As badly wounded he was
25. At the deep bottom of Atlantic .
A. lied the Titanic C. lay the Titanic
B. did the Titanic lie D. had the Titanic lain
26. He finally agreed, reluctantly, to help us.
A. albeit B. nonetheless C. somehow or other D. in all likelihood
27. Charlie a speech at the end of the last term, but he didn't.
A. were to give B. is to give C. was to be giver D. was to have given
28. circling the globe faster than Jules Verne's fictional Phileas Fogg.
A. A pioneer journalist, Nellie Bly's exploits included
B. Also included in the exploits of Nellie Bly, a pioneer journalist, was
C. The exploits of Nellie Bly, a pioneer journalist, included
D. The pioneer journalist's exploits of Nellie Bly included
29. He paused, afraid lest he too much.
A. didn't say B. say C. has said D. hadn't been saying
30. We you the money you needed but you didn't tell us.
A. could have lent B. should have lent C. could lend D. were able to

2|Page Tài liệu khóa học Live - VIP


OTTO CHANNEL
Otto Channel
PART IV. PHRASAL VERBS AND PREPOSITIONS (5PTS)
31. Junior hospital doctors are thrown at the deep end in their first jobs.
A. in B. away C. to D. with
32. I was the impression that you liked Indian food.
A. at B. on C. with D. under
33. Governments should international laws against terrorism.
A. bring up B. bring about C. bring in D. bring back
34. of the financial crisis, all they could do was hold on and hope that things would improve.
A. At the bottom B. At the height C. On the top D. In the end
35. I couldn't decide what to write about, when I suddenly upon the idea of doing something
on the writer's block.
A. thought B. chanced C. hit D. arrived
36. Jane got the job virtue of her greater experience.
A. with B. on C. by D. for
37. I see a lot of young people with this new hairstyle. It seems to be .
A. piling up B. got round C. coming off D. catching on
38. Be careful! The young horse hasn't been .
A. broken in B. bringing off C. taken over D. set up
39. Theories about the causes of bird flu have changed the light of recent research.
A. on B. in C. to D. with
40. Jane managed to eke her student loan till the end of the year.
A. off B. out C. in D. through
PART V. GUIDED CLOZE (10PTS)
Read the text below and decide which answer best fits each space.
PASSAGE 1
Working to (41) through with my university years did, in (42) , prepare me
for life after graduation. I've never been lazing (43) and have already crammed as much into each
day as I could. I guess I've always been someone who takes things in their stride and snapped up any
opportunity that has come my way. Looking back. I feel a great (44) of self-respect for
finishing my studies without (45) with any debts but I'll admit it wasn't always plain sailing.
Not until I scraped through my first-year exams did I realize the (46) of time management. I
think personality has played a huge role in my success, (47) I've always taken pleasure in
whatever I do, whether it's for work or play. Sometimes people are surprised at my ability to solve
problem (48) and wonder where my energy comes from. I think it must be (49) . My
parents' work ethic and attitude towards life had a great impact on me and (50) have me
down the path I've taken.
41. A. get B. pull C. see D. make
42. A. reflection B. reflexion C. retrospect D. hindsight

3|Page Tài liệu khóa học Live - VIP


OTTO CHANNEL
Otto Channel
43. A. away B. around C. round D. off
44. A. state B. reason C. awareness D. sense
45. A. incurring B. meeting C. encountering D. heading
46. A. significance B. magnificence C. principle D. vitality
47. A. nonetheless B. nevertheless C. though D. then
48. A. affectionately B. proficiently C. efficaciously D. prominently
49. A. heredity B. inherence C. heritage D. hierarchy
50. A. propelled B. urged C. geared D. diverted
PASSAGE 2
It is important to recognize that online learning has (51) advantages and disadvantages.
Although students' grades appear to be (52) by the mode of instruction, certain courses are
more challenging to students who carry out their studies in the virtual environment than in the
classroom. However, in online classes, participation in learning activities may be less daunting, (53)
for shy students, and the quality and quantity of student - student and teacher - student
interaction may be higher. Increasingly. it is a/an (54) issue for designers of online curriculums
to decide how to match the advantages of different modes of instruction to specific courses, by offering
not only fully classroom-based or online courses, but also courses that take the best elements of both
types to (55) the needs of students and teachers and to (56) the most efficient use of
resources.
Students who study online tend to lack a sense of community, trust and positive interaction with
other course members and teachers - all elements that were in the past believed to increase the
effectiveness of classroom learning amongst least confident learners. However, online students
generally also feel that they learn at a similar rate to their (57) in the classroom, and in
fact at my college their grades are just as good as those who are taught in (58) . But just
imagine how much more effective our online courses could be if they fostered a culture of class
cohesion, spirit, trust and interaction, both among students and between students and faculty. Perhaps
the most effective way achieve this improvement is for online educators to give more (59)
contact and to encourage students to collaborate.
Online learning provides a far more student-centered teaching approach than the traditional
classroom method, and all school directors should aim to adopt it as their main means to (60)
education.
51. A. infinite B. definite C. defined D. intimate
52. A. unharmed B. invulnerable C. impervious D. unaffected
53. A. especially B. namely C. specifically D. explicitly
54. A. crucial B. vital C. obvious D. expendable
55. A. insatiate B. cater C. address D. respond
56. A. put B. make C. earn D. gain
57. A. peers B. counterparts C. partners D. partakers
58. A. private B. person C. individual D. secret
59. A. one-for-one B. one-by-one C. one-in-one D. one-on-one
60. A. convey B. cover C. present D. deliver

4|Page Tài liệu khóa học Live - VIP


OTTO CHANNEL
Otto Channel
PART VI. READING COMPREHENSION (10PTS)
Read the text below and choose the best answer to each question.
PASSAGE 1
A fold culture is a small, isolated, cohesive, conservative, nearly self- sufficient that is
homogeneous in custom and race, with a strong family or clan structure and highly developed rituals.
Order is maintained through sanctions based in the religion or family, and interpersonal relationships
are strong. Tradition is paramount, and change comes infrequently and slowly. There is relatively little
division of labor into specialized duties. Rather, each is expected to perform a great variety of tasks,
though duties may differ person between the sexes. Most goods are handmade, and a subsistence
economy prevails. Individualism is weakly developed in folk cultures, as are social classes. Unaltered
folk cultures no longer exist in industrialized countries such as the United States and Canada. Perhaps
the nearest modern equivalent in Anglo- America is the Amish, a German American farming sect that
largely renounces the products and labor saving devices of the industrial age. In Amish areas, horse-
drawn buggies till serve as a local transportation device, and the faithful are not permitted to own
automobiles. The Amish's central religious concept of Demut, "humility", clearly reflects the weakness
of individualism and social class so typical of folk cultures, and there is a corresponding strength of
Amish group identity. Rarely do the Amish marry outside their sect. The religion, a variety of the
Mennonite faith, provides the principal mechanism for maintaining order.
By contrast, a popular culture is a large heterogeneous group, often highly individualistic and
constantly changing. Relationships tend to be impersonal, and a pronounced division of labor exists,
leading to the establishment of many specialized professions. Secular institutions of control such as
the police and army take the place of religion and family in maintaining order, and a money-based
economy prevails. Because of these contrasts, "popular" may be viewed as clearly different from
"folk". The popular is replacing the folk in industrialized countries and in many developing nations.
Folk-made objects give way to their popular equivalent, usually because the popular item is more
quickly or cheaply produced, is easier or time saving to use, or lends more prestige to the owner.
61. What does the passage mainly discuss?
A. Two decades in modern society
B. The influence of industrial technology
C. The characteristics of "folk" and "popular" societies
D. The specialization of labor in Canada and the United States
62. The word "homogeneous" is closest in meaning to
A. uniform B. general C. primitive D. traditional
63. Which of the following is typical of folk cultures?
A. There is a money-based economy. B. Social change occurs slowly.
C. Contact with other cultures is encouraged. D. Each person develops one specialized skill.
64. What does the author imply about the United States and Canada?
A. They value folk cultures. C. They have popular cultures.
B. They have no social classes. D. They do not value individualism.

5|Page Tài liệu khóa học Live - VIP


OTTO CHANNEL
Otto Channel
65. The phrase "largely renounces" is closest in meaning to
A. generally rejects B. greatly modifies C. loudly declares D. often criticizes
66. What is the main source of order in Amish society?
A. The government B. The economy C. The clan structure D. The religion
67. Which of the following statements about Amish beliefs does the passage support?
A. A variety of religious practices is tolerated.
B. Individualism and competition are important.
C. Pre-modern technology is preferred.
D. People are defined according to their class.
68. Which of the following would probably NOT be found in a folk culture?
A. A carpenter B. A farmer C. A weaver D. A banker
69. The word "prevails" is closest in meaning to .
A. dominates B. provides C. develops D. invests
70. Which of following is NOT given as a reason why folk-made objects are replaced by mass-
produced objects?
A. Cost B. Prestige C. Quality D. Convenience
PASSAGE 2
According to sociologists, there are several different ways in which a person may become
recognized as the leader of a social group in the United States. In the family, traditional cultural
patterns confer leadership on one or both of the parents.
In other cases, such as friendship groups, one or more persons may gradually emerge as leaders,
although there is no formal process of selection. In larger groups, leaders are usually chosen formally
through election or recruitment.
Although leaders are often thought to be people with unusual personal ability, decades of research
have failed to produce consistent evidence that there is any category of "natural leaders". It seems
that there is no set of personal qualities that all leaders have in common; rather, virtually any person
may be recognized as a leader if the person has qualities that meet the needs of that particular group.
Furthermore, although it is commonly supposed that social groups have a single leader, research
suggests that there are typically two different leadership roles that are held by different individuals.
Instrumental leadership is leadership that emphasizes the completion of tasks by a social group. Group
members look to instrumental leaders to "get things" done. Expressive leadership, on the other hand,
is leadership that emphasizes the collective well-being of a social group's member. Expressive leaders
are less concerned with the overall goals of the group than with providing emotional support to group
members and attempting to minimize tension and conflict among them. Group members expect
expressive leaders to maintain stable relationships within the group and provide support to individual
members.
Instrumental leaders are likely to have a rather secondary relationship to other group members.
They give orders and may discipline group members who inhibit attainment of the group's goals.
Expressive leaders cultivate a more personal or primary relationship to others in the group. They offer

6|Page Tài liệu khóa học Live - VIP


OTTO CHANNEL
Otto Channel
sympathy when someone experiences difficulties or is subjected to discipline, are quick to lighten a
serious moment with humor, and try to resolve issues that threaten to divide the group. As the
differences in these two roles suggest, expressive leaders generally receive more personal affection
from group members; instrumental leaders, if they are successful in promoting group goals, may enjoy
a more distant respect.
71. What does the passage mainly discuss?
A. The problems faced by leaders
B. How leadership differs in small and large groups
C. How social groups determine who will lead them
D. The role of leaders in social groups
72. The passage mentions all of the following ways by which people can become leaders EXCEPT
A. recruitment B. formal election process
C. specific leadership training D. traditional cultural patterns
73. In mentioning “natural leaders" in line 7, the author is making the point that.
A. few people qualify as "natural leaders"
B. there is no proof that "natural leaders" exist
C. "natural leaders" are easily accepted by the members of a social group
D. "natural leaders" share a similar set of characteristics
74. Which of the following statements about leadership can be inferred from paragraph 2?
A. A person who is an effective leader of a particular group may not be an effective leader in
another group.
B. Few people succeed in sharing a leadership role with another person.
C. A person can best learn how to be an effective leader by studying research on leadership.
D. Most people desire to be leaders but can produce little evidence of their qualifications.
75. The passage indicates that instrumental leaders generally focus on.
A. ensuring harmonious relationships B. sharing responsibility with group members
C. identifying new leaders D. achieving a goal
76. The word "collective" is closest in meaning to .
A. necessary B. typical C. group D. particular
77. The word "them" refers to .
A. expressive leaders B. goals of the group
C. group members D. tension and conflict
78. A "secondary relationship" between a leader and the members of a group could best be
characterized as.
A. distant B. enthusiastic C. unreliable D. personal
79. The word "resolve" is closest in meaning to
A. avoid repeating B. avoid thinking about
C. talk about D. find a solution for

7|Page Tài liệu khóa học Live - VIP


OTTO CHANNEL
Otto Channel
80. Paragraphs 3 and 4 organize the discussion of leadership primarily in terms of .
A. examples that illustrate a problem B. cause and effect analysis
C. narration of events D. comparison and contrast

SECTION B. WRITTEN TEST (60 PTS)


PART I. CLOZE TEST (20 PTS)
Read the text below and think of the word which best fits each space. Use only ONE WORD for each
space.
PASSAGE 1
SMALL TALK
It's often said the British talk about the weather more than any other nationality in the world. Some
people even go so far as to claim that they talk about little (1) . But while it may seem that the
British alone have an undue (2) with the weather, the fact is, climatic conditions are a common topic
of conversation all around the world. This is hardly surprising. After all, the weather is one of the few
things that we all have in common, and it influences us profoundly. It affects our (3) of mind, our
daily activities, our weekend plans and more.
But that's only (4) of the story. There's really a lot (5) to it than that. In Britain,
conversations about the weather are usually not really about the weather at all. The British use
comments about the weather to (6) ice in social situations, (7) the awkward or
uncomfortable silences during conversations, or, (8) , as a greeting. Remarks like "Nice day,
isn’t it?", "Ooh, isn't it hot?" and "Looks like rain, no?" are not requests for meteorological data. Rather,
they are ritual greetings used to indicate someone wishes to engage you in conversation; or they are
just signs of friendliness. Learners of the English should do (9) to understand the
fundamental function remarks about the weather (10) to understand the fundamental function
remarks in British social interaction.
PASSAGE 2
AIRPORT AVATARS
Smiling, computerized, talking avatars are being introduced at airports around the world in a bid to
make travelers’ journeys more efficient. The life-sized hologram projections which are situated at key
points in airports such as just before security, read out pre-recorded messages designed to help
passengers anything from the location of bathrooms and taxi stands (1) on-flight liquid restrictions
and security regulations. Some of these virtual assistants, which have been unveiled at New York's JFK
and Paris' Orly airport among others, have a sensor which activates their (2) when a person
walks (3) a few feet of them. (4) repeat their pre-programmed speeches on a loop. All
dispense their information in comforting and informative voices intended to calm passenger's nerves.
So (5) are these latest airport employees faring? According to airport staff, feedback from
passengers has been largely positive. More importantly, though, these avatars are proving effective.
Because they are so lifelike, passengers can't help but look at them and take notice of what they are
saying. Thus, they are doing what they were designed to (6) ; ease the workload. on the
airport's flesh-and- (7) staff. Will we one day see avatars at all airports? Perhaps. But with one

8|Page Tài liệu khóa học Live - VIP


OTTO CHANNEL
Otto Channel
(8) . The current avatars are not interactive - they cannot hold conversations with (9) .
Developers are hopeful that (10) additions to this series of avatars will be active.

PART II. WORD FORMATION (20 PTS)


A. Complete each sentence, using the correct form of the word in parentheses.
1. He is for his charitable activities than for his business in the steel industry. (KNOW)
2. Our bodies are naturally by our organs of elimination- the skin, lungs, intestines,
kidneys and liver. (TOXIC)
3. He's such a/an who always pretends to know everything. (INTELLECT)
4. She should take these drugs to treat her state. (DEPRESS)
5. The audience was mesmerized by her clear and voice. (SOUND)
6. A number of local church leaders have acted as with the people in the vicinity. (GO)
7. The ban on firecrackers was introduced to reduce the number of accidents. (EXPRESS)
8. He sold the car to a/an who paid with a worthless stolen cheque. (TRICK)
9. It was for a new band to be offered such a deal. (HEAR)
10. John was arrested for smuggling . (BAN)
B. Complete the passage with the appropriate forms from the words given in the box.

Satisfy graph succeed write intense


precede apparent great force sign

The invention of the telegraph in 1843 (1) a new era in communication technology, the
electronic era – so called because the telegraph used electrical signals to carry information along an
electrical wire. The telegraph fed society’s (2) appetite for immediate access to the
information, and it provided a foundation for (3) technologies: the telephone in 1876, the (4)
- or record player - in 1878, film and movies in the 1890s, radio in 1919, and television in 1925.
As with the earliest (5) books, radio technology was at first restricted to the educated few,
scientists and researchers, but later spread to the (6) society. By the 1930s, the middle
classes were listening to news, drama, comedy, and musical performances from thousands of miles
away.
Radio was the first mass broadcast medium and, together with television a few decades later, it
was responsible for altering both the pattern and volume of information that flowed into people's
homes.
The invention of electronic media changed communication more than any other technological event
since the development of writing. The spread of electronic media occurred with a/an (7)
speed, thus (8) commercial and social interaction. There was a tremendous shift in how people
received the information that contributed to their understanding of the world. Voices or pictures could
create ideas that (9) authentic - more so than the ideas conveyed by the printed world. The
graphic, intensely human nature of electronic media (10) the belief that if it was on the air, it had
to be true.

9|Page Tài liệu khóa học Live - VIP


OTTO CHANNEL
Otto Channel
PART III. ERROR CORRECTION (10 PTS)
The following passage contains 10 errors. Identify and correct them.
BLUE WHALES
1 Blue whales, the world's largest animals, have been sighted again in British waters for the
2 first time in at least twenty years. Indications that a population of blue whales was inhabiting
3 the waters west of Scotland came for the first time from the United States Navy, which
4 surveillance system picked up the songs of a lot of different whales. American zoologists
5 later subsequently certified the blue whale song among them. Now mariner biologist, Carol
6 Booker, has actually seen a blue whale there herself. She has no doubt about what she saw,
7 because they have distinctive fins which are very small for their size. She says, “Worldwide
8 they were mostly extinct and it seemed they had completely vanished from North Atlantic,
9 so you can imagine how I felt actually seeing ones! However, it is certainly too soon to say
10 if it is an indication of a populous recovery." She goes on to say, "What it does show is the
11 importance of this area of the ocean for whales, and how essential it is to control pollution
12 of the seas." Bigger than any dinosaur known to man, blue whales are the largest animals
13 ever to have lived on earth. A blue whale is more than six meters long at birth and, when
14 completely grown, its heart is the same height as a tall man and weighs as much as a horse
15 is.

Lines Mistake Correction Lines Mistake Correction

PART IV. SENTENCE TRANSFORMATION (10 PTS)


Rewrite the following sentences using the words given.
1. He was too frightened to admit that he had broken the window. (OWN)
→ So ...................................................................................................................................
2. I wish he would stop criticizing my work. (FAULT)
→ I'd sooner ........................................................................................................................
3. Zoe always makes spontaneous decisions concerning her travel plans. (ACTS)
→ Zoe always ......................................................................................................................
4. He really disappointed me when breaking the promise to help me out. (TEETH)
→ He really .........................................................................................................................
5. If her father hadn't retired, she wouldn't have taken over his work. (STEPPED)
→ But for ............................................................................................................................
6. His efforts to find a solution didn't deserve such savage criticism.
→ He shouldn't ....................................................................................................................

10 | P a g e Tài liệu khóa học Live - VIP


OTTO CHANNEL
Otto Channel
7. People became aware of the damage to the ozone layer when an enormous hole was discovered
over the South Pole.
→ It was the .......................................................................................................................
8. When I grow up, I'm going to be really important. (CAT)
→ .......................................................................................................................................
9. I thought very hard but couldn't remember the answer. (RACKED)
→ I .....................................................................................................................................
10. Alex grimaced as he swallowed the foul-tasting medicine. (PULLED)
→ Alex ................................................................................................................................

THE END

11 | P a g e Tài liệu khóa học Live - VIP


OTTO CHANNEL
Otto Channel
SỞ GIÁO DỤC & ĐÀO TẠO KỲ THI OLYMPIC TRUYỀN THỐNG 30/4
TP. HỒ CHÍ MINH LẦN THỨ XXV – NĂM 2018
ĐỀ CHÍNH THỨC Môn thi: Anh văn - Khối: 10
Thời gian làm bài: 180 phút

➢ Thí sinh làm phần trắc nghiệm (MULTIPLE CHOICE) trên phiếu trả lời trắc nghiệm và phần tự
luận (WRITTEN TEST) trên phiếu trả lời tự luận.
➢ Trên phiếu trả lời trắc nghiệm, thí sinh tô thêm 2 số 00 vào trước số báo danh (bằng bút chì).
Phần mã đề thi trên phiếu trắc nghiệm, thí sinh tô vào ô 002.
 --------------------------------------------------------------------------------------------------------------------

CẤU TRÚC ĐỀ THI


SECTION A. MULTIPLE CHOICE (40 PTS) .......................................................................... 1
PART I. GRAMMAR AND STRUCTURES (20 PTS) ................................................................................... 1
PART II. GUIDED CLOZE (10PTS) ................................................................................................................ 3
PART III. READING COMPREHENSION (10PTS) ........................................................................................ 5

SECTION B. WRITTEN TEST (60 PTS) .............................................................................. 10


PART I. CLOZE TEST (20 PTS) ................................................................................................................... 10
PART II. WORD FORMATION (20 PTS)..................................................................................................... 11
PART III. ERROR IDENTIFICATION (10 PTS) .......................................................................................... 12
PART IV. SENTENCE TRANSFORMATION (10 PTS) ............................................................................... 12
Otto Channel
SỞ GIÁO DỤC & ĐÀO TẠO KỲ THI OLYMPIC TRUYỀN THỐNG 30/4
TP. HỒ CHÍ MINH LẦN THỨ XXV – NĂM 2018
ĐỀ CHÍNH THỨC Môn thi: Anh văn - Khối: 10
Thời gian làm bài: 180 phút

➢ Thí sinh làm phần trắc nghiệm (MULTIPLE CHOICE) trên phiếu trả lời trắc nghiệm và phần tự
luận (WRITTEN TEST) trên phiếu trả lời tự luận.
➢ Trên phiếu trả lời trắc nghiệm, thí sinh tô thêm 2 số 00 vào trước số báo danh (bằng bút chì).
Phần mã đề thi trên phiếu trắc nghiệm, thí sinh tô vào ô 002.
 --------------------------------------------------------------------------------------------------------------------
SECTION A. MULTIPLE CHOICE (40 PTS)
PART I. GRAMMAR AND STRUCTURES (20 PTS)
Choose the best options (A, B, C, or D) that best complete the following sentences.
1. The local authority expressed regret as US drone strike has killed innocent hostages.
A. incongruously B. vehemently C. inadvertently D. graciously
2. These days the castle is swamped with of tourists.
A. mobs B. throngs C. shoals D. cliques
3. He left the meeting early on the unlikely that he had a sick friend to visit.
A. excuse B. pretext C. motive D. claim
4. He decided to withdraw from the powerboat race as he could see a(n) of danger.
A. prediction B. foreboding C. omen D. dearth
5. I slipped briefly back into sleep and emerged when breakfast was being served outside in a(n)
garden-courtyard.
A. whimsical B. extravagant C. extortionate D. enchanting
6. He had a momentary of concentration and before he knew it the car had spun out of
control.
A. lapse B. loss C. slip D. mistake
7. In the hands of a careless driver, a car becomes a weapon.
A. fatal B. mortal C. lethal D. venal
8. He glanced at Juliet accusingly and she looked abashed.
A. completely B. absolutely C. utterly D. suitably
9. We were sorted out into groups according to the types of honors and quite a long wait
A. ensued B. eventuated C. supervened D. transpired
10. The Prime Minister will decide whether to release the prisoner or not; that's his .
A. derogatory B. abdication C. prerogative D. humanity
11. His new play is not only interesting but also unique. It is really off the beaten
A. road B. path C. route D. track
12. That Peter was born and brought up in a rich family is as clear as the
A. nose on his face B. tip of his tongue
C. back of his hand D. hair on his head

1|Page Tài liệu khóa học Live - VIP


OTTO CHANNEL
Otto Channel
13. I will try to finish the job to the best of my .
A. knowledge B. ability C. means D. command
14. Tim said the meal was , so we didn't have to worry about the price.
A. on the house B. on his expense C. for him D. for his money
15. Let Hercules himself do what he may, for a cat will mew and a dog will have his
A. day B. time C. month D. year
16. Breaking his leg a blow to his chances of becoming a professional footballer.
A. brought B. caused C. dealt D. struck
17. In the acting career, the moment one first cut his will be the most memorable with
embarrassment and pride bubbling up inside.
A. nails B. teeth C. fingers D. hair
18. His French is roughly with my Japanese, so communication was rather difficult.
A. in harmony B. on a par C. on equal term D. on good terms
19. Max has been my ears all night about his new job.
A. bending B. deafening C. rolling D. biting
20. The fighting has stopped, so to , the war is over.
A. all pins and needles C. all prim and proper
B. all chop and change D. all intents and purposes
21. We would sooner Mr. Tram us the urgent information the other night.
A. would have sent B. had sent C. sent D. had been sent
22. Would you be my letter while I am away?
A. too good as to forward B. so good as to forward
C. as good as to forward D. so good as forwarding
23. He us on the last day of the congress, so his presence at the opening ceremony was
something of a surprise.
A. could have joined B. was about to join C. had to join D. was to join
24. You could have done inviting Sam to the party.
A. better or worse than B. a lot worse than
C. much better as D. nothing as worse as
25. He works until nine o'clock every evening, and that's quite the work he does over the
weekend.
A. except for B. apart from C. without D. but for
26. I would rather go skiing picnicking this weekend.
A. than going B. than to going C. than to go D. than go
27. It is mandatory that smoking in public .
A. is prohibited B. must be prohibited C. prohibiting D. be prohibited

2|Page Tài liệu khóa học Live - VIP


OTTO CHANNEL
Otto Channel
28. In geometry, an ellipse may be defined as the locus of all points distances from two fixed
points is constant.
A. the sum of whose B. of which the sum C. whose sum of D. which the sum of
29. Tony gripped his brother's arm lest he by the mob.
A. would be trampled B. were trampled
C. be trampled D. could have been trampled
30. The mini dress was , but now it is making a comeback.
A. a fad once thought to be finished C. thought a fad to be finished once
B. once thought a fad to be finishing D. once thought to be a finishing fad
31. If you never put oil into your car engine, one day it will .
A. flake out B. shut down C. seize up D. run off with
32. If you pay the restaurant bill with your credit card, it will with you later.
A. settle down B. settle up C. pay back D. pay off
33. In those days, doctors ladled antibiotics to patients.
A. with B. on C. in D. out
34. Let's find a place where we can the storm.
A. wait out B. wear off C. wind down D. shrug off
35. The schoolboy winced the sight of the cane in the headmaster's hand.
A. at B. for C. by D. of
36. I was completely bowled their warm reception.
A. with B. up C. off D. over
37. I am not liberty to tell you anything about his private life.
A. in B. at C. by D. on
38. The figure is more 200, I think.
A. of B. at C. like D. with
39. Their performances are really compare. How amazing!
A. out of B. over C. within D. beyond
40. The party was full swing when I arrived. Everyone was singing and dancing.
A. in B. on C. about D. with

PART II. GUIDED CLOZE (10PTS)


Choose the most appropriate words to fill in the blank
PASSAGE A:

The national park movement began in the United States in 1870 when a team of explorers
suggested that part of the Yellowstone River region be (41) in order to protect its geothermal
(42) wildlife, forests, and (43) scenery for the benefit of future generations.
Congress (44) by creating Yellowstone National Park, the world's first, in 1872.

3|Page Tài liệu khóa học Live - VIP


OTTO CHANNEL
Otto Channel
The idea proved (45) , and the number of national parks in this country grew rapidly, new
parks being set up by presidential (46) and sometimes as a result of gifts by states of the
union or by individuals. Administration of this increasingly complex system was in the hands of the
U.S. Army for thirty years from 1886, but then Congress created the National Park Service as part of
the Department of the Interior to (47) it.
Today, in addition to what might be thought of as typical national parks, the Service also manages
places of historic interest, hiking trails, seashores, rivers, (48) of scientific interest and
memorials. In all, more than 300 entities are involved, covering over 32 million hectares. Each unit is
directed by a superintendent who is responsible for all aspects of the operation. Staff (49)
administrative personnel and, according to the nature of the unit, park rangers, naturalists, historians,
and (50) workers.

41. A. set up B. set aside C. set by D. set in


42. A. tokens B. characters C. features D. traits
43. A. breathless B. optimal C. exceptional D. prominent
44. A. countered B. retrieved C. designated D. responded
45. A. prevailing B. banal C. widespread D. popular
46. A. analogy B. proclamation C. constitution D. dissemination
47. A. oversee B. overlook C. overcharge D. overact
48. A. observatories B. reserves C. reservoirs D. estuaries
49. A. cover B. control C. undergo D. undertake
50. A. maintenance B. sustainable C. protectorate D. conservation

PASSAGE B:
Ocean water plays a(n) (51) role in supporting life. The great ocean basins hold about 300
million cubic miles of water. From this vast amount, about 80.000 cubic miles of water are sucked into
the atmosphere each year by evaporation and returned by precipitation and drainage to the ocean.
More than 24.000 cubic miles of rain descend annually upon the continents. This vast amount is
required to (52) the lakes and streams, springs and water tables on which all flora and
fauna are dependent. Thus, the hydrosphere permits organic existence.
The hydrosphere has strange characteristics because water has (53) unlike those of any other
liquid. One (54) is that water upon freezing (55) by about 9 percent, whereas most
liquids contract on cooling. (56) this reason, ice floats on water bodies instead of sinking to the
bottom. If the ice sank, the hydrosphere would soon be frozen solidly, except for a thin layer of surface
melt water during the summer season. Thus, all aquatic life would be destroyed and the interchange
of warm and cold currents which moderates climate, would be (57) absent.
Another outstanding characteristic of water is that water has a heat capacity which is the highest
of all liquids and solids except ammonia. This characteristic often enables the oceans to absorb and
store vast quantities of heat, (58) often preventing climatic extremes. In addition, water
dissolves more substances than any other liquid. It is this characteristic which helps make oceans a
great from the storehouse for minerals which have been washed (59) from the
continents. In several areas of the world these minerals are being commercially exploited. Solar

4|Page Tài liệu khóa học Live - VIP


OTTO CHANNEL
Otto Channel
evaporation of salt is widely (60) , potash is extracted from the Dead Sea, and magnesium is
produced from sea water along the American Gulf Coast.
51. A. principle B. principal C. dispensable D. expendable
52. A. replenish B. reinforce C. replete D. restore
53. A. property B. characteristics C. assets D. nature
54. A. alienation B. distinction C. contrast D. inconsistency
55. A. extends B. inflates C. increases D. expands
56. A. For B. By C. In D. indefinitely
57. A. remarkably B. vaguely C. unnoticeably D. With
58. A. so B. hence C. and then D. consequently
59. A. off B. out c. up D. down
60. A. exercised B. exerted C. practiced D. proceeded
PART III. READING COMPREHENSION (10PTS)
Read the following passage carefully and answer the questions below.
PASSAGE A:
STEP BACK IN TIME
Historical biographer Antonia Fraser reveals the pleasures of studying a bygone era.
Gibbon was inspired to write The Decline and Fall of the Roman Empire sitting on the steps of the
Capitol at Rome one evening, listening to the sound of monks chanting vespers. My own inspiration to
become a historical biographer came in rather less elevated circumstances, as a teenager one rainy
Oxford afternoon: I began to read Lytton Strachey's Eminent Victorians and was in particular fascinated
by his essay on the worldly Cardinal Manning. This was going to be the life for me! Once back at school
I plunged into further research in the convent library. A very different picture emerged. Gradually as I
pursued the topic, I became aware of Strachey's daring sallies into "artistic truth" (as opposed to
historical truth). Nevertheless. I never forgot my original sense being transported into a world more
vivid than my own.
An ability to convey this sensation is, I believe, at the heart of the matter. If you, the biographer,
don't thrill to your subject, you can hardly in all fairness expect the reader to do so. In a sense (not of
course the commercial sense) the choice of subject is irrelevant so long as it meets that requirement.
You could say that I was extremely lucky to choose Mary Queen of Scots for my first foray since there
proved to be a world-wide public for the troubles of the ill-fated Queen. But you could argue equally
that I made my own luck, since I had always been obsessed by Mary's story from childhood. Nor was
success fore-ordained. It was, after all, the leading publisher Mark Bonham-Carter of (then) Collins
who said to me when I confessed my project, "They say that all books on Mary Queen of Scots sell
and no books on South America do", before adding with a laugh, "Perhaps yours will be the exception."
Nevertheless, I did have luck. In the 60s, so-called narrative biography was said to be passé. Mary
Queen of Scots was an early beneficiary from the fact that the public continued to have an appetite
for it, so long as the research was felt to be solid.
The actual research for a biography - now that's a whole other matter. The paramount need for it
historical truth not Stracheyesque truth must be established - means that biographers discover for

5|Page Tài liệu khóa học Live - VIP


OTTO CHANNEL
Otto Channel
themselves the reality of Dr. Johnson's wise dictum: "A man will turn over half a library to make a
book."
And what about those fabled things boasted of on blurbs: hitherto unpublished documents?
Obviously it is every researcher's dream to discover such papers, and their discovery once again may
make a project commercial which would not otherwise be so. At the same time, I would issue a caveat
about hitherto unpublished documents. HUDS are not in themselves more valuable than the printed
sources - it's a historical coincidence that one set has become known early on, the other not. One
needs to evaluate them even more closely. Here I speak from personal experience. A series of chances
led me to the discovery of some hitherto unpublished letters of Oliver Cromwell just as I was finishing
my manuscript. I blazoned my finds across the text: only to realize at the proof stage, that they might
be unpublished but they were not very important in the grand scheme of things... an expensive
mistake.
Where the perils and pleasures of writing historical biography are concerned, there are two perils
which seem to me to raise points of principle. The first is the peril of anachronistic judgements.
For example, in the 16th century more or less everybody took astrology seriously and more or less
everybody enjoyed a jolly afternoon out to see the bears baited. It's no good dismissing the former as
meaningless and cringing from the latter as disgusting.
I would further cite the peril of hindsight. We may know that Henry VIII will marry six times, but
he didn't, and he would have been amazed if it had been predicted at the time of his first marriage to
Catherine of Aragon.
And the pleasures? Manifold! Principal among them however is the opportunity to lead a life less
ordinary. As a biographer, 1 can rule over kingdoms, lead the cavalry into battle, patronize the great
artists of the past and all without leaving my chair.
61. What did the writer learn while researching a historical figure as a teenager?
A. There was a surprising amount of information available.
B. It was not possible to take everything she read as fact.
C. It was difficult to interpret the true meaning of what she read.
D. It was necessary to consult a wide range of sources.
62. What does that requirement refer to?
A. the reader's response to a writer's subject
B. the correct choice of subject
C. the commercial appeal of the book
D. the writer's ability to communicate their enthusiasm
63. What did Mark Bonham-Carter believe about the writer's choice of subject?
A. Her long-standing interest in it may ensure her book's success.
B. It did not guarantee her book's success.
C. There are already too many books written on it.
D. It was a wise choice for her first biography.
64. The main point that the writer is making in the fourth paragraph is that.
A. a biography is more likely to be successful if it contains new information.
B. researchers must be careful to check all facts thoroughly.

6|Page Tài liệu khóa học Live - VIP


OTTO CHANNEL
Otto Channel
C. research material can include inaccurate information.
D. extensive reading is crucially important.
65. What warning does the writer give to biographers about unpublished documents?
A. They are difficult to obtain as their discovery is down to chance.
B. Their overall significance to the book must be carefully considered.
C. Their use could result in diminished commercial success for a book.
D. It should not be assumed that they are authentic.
66. An example of an anachronistic judgement that the writer gives is .
A. not being able to imagine oneself living in the sixteenth century.
B. being uninformed about sixteenth century customs and practices.
C. viewing the sixteenth century from a twenty-first century perspective.
D. focusing only on the negative side of life in the sixteenth century,
67. In the article as a whole, the writer implies that her main motivation for becoming a historical
biographer was the chance to.
A. carry out extensive research. C. discover unpublished documents.
B. become immersed in history. D. establish historical truth.
68. The word elevated is closest in meaning to.
A. lofty B. normal C. raised D. high
69. The word passé is closest in meaning to.
A. obsolete B. out of fashion C. antique D. archaic
70. The word there in the fifth paragraph refers to.
A. blurbs B. researchers C. unpublished documents D. historical truths
PASSAGE B
The Amazonian wilderness harbors the greatest number of species on this planet and is an
irreplaceable resource for present and future generations. Amazonia is crucial for maintaining global
climate and genetic resources, and its forest and rivers provide vital sources of food, building
materials, pharmaceuticals, and water needed by wildlife and humanity.
The Los Amigos watershed in the state of Madre de Dios, southeastern Peru, is representative of
the pristine lowland moist forest once found throughout most of upper Amazonian South America.
Threats to tropical forests occur in the form of fishing, hunting, gold mining, timber extraction,
impending road construction, and slash and burn agriculture. The Los Amigos watershed, consisting of
1.6 million hectares (3.95 million acres), still offers the increasingly scarce opportunity to study
rainforest as it was before the disruptive encroachment of modern human civilization. Because of its
relatively pristine condition and the immediate need to justify it as a conservation zone and as a corridor
between Manu National Park and the Tambopata-Candamo Reserved Zone, this area deserves
intensive, long-term projects aimed at botanical training, ecotourism, biological inventory, and
information synthesis.
On July 24. 2001, the government of Peru and the Amazon Conservation Association, represented
by Enrique Ortiz, signed a contractual agreement creating the first long-term permanently renewable
conservation concession. To our knowledge this is the first such agreement to be implemented in the

7|Page Tài liệu khóa học Live - VIP


OTTO CHANNEL
Otto Channel
world. The conservation concession protects 340,000 acres of old growth Amazonian forest in the Los
Amigos watershed which is located in southeastern Peru. This watershed protects the eastern flank of
Manu National Park and is part of the lowland forest corridor that links it to Bahuaja-Sonene National
Park. The Los Amigos conservation concession will serve as a mechanism for the development of a
regional center of excellence in natural forest management and biodiversity science.
Several major projects are being implemented at the Los Amigos Conservation Area. Louise
Emmons is initiating studies of mammal diversity and ecology in the Los Amigos area. Other projects
involve studies of the diversity of arthropods, amphibians, reptiles, and birds. Robin Foster has
conducted botanical studies at Los Amigos, resulting in the labeling of hundreds of plant species along
two kilometers of trail in upland and lowland forest. Los Amigos has also been a major field site for
Robin's rapid identification laminated photographic field guides to tropical plants. Michael Goulding is
leading a fisheries and aquatic ecology program, which aims to document the diversity of fish, their
ecologies, and their habitats in the Los Amigos area and the Madre de Dios watershed in general.
With support from the Amazon Conservation Association, and in collaboration with US and Peruvian
colleagues, the Botany of the Los Amigo project has been initiated. At Los Amigos, we are attempting
to develop a system of preservation, sustainability, and scientific research; a marriage between various
disciplines, from human ecology to economic botany, product marketing to forest management. The
complexity of the ecosystem will best be understood through a multidisciplinary approach, and
improved understanding of the complexity will lead to better management. In essence, we must be
informed to make wise management decisions about Amazonian forests. These forests hold the
greatest number of species on our planet and are an irreplaceable resource for present and future
generations. The future of these forests will depend on sustainable management and development of
alternative practices and products that do not require irreversible destruction.
The botanical project will provide a foundation of information that is essential to other programs
at Los Amigos. By combining botanical studies with fisheries and mammology. we will better
understand plant/animal interactions. By providing names, the botanical program will facilitate
accurate communication about plants and the animals that use them. Included in this scenario are
humans, as we will dedicate time to people-plant interactions in order to learn what plants are used
by people in the Los Amigos area, and what plants could potentially be used by people.
To be informed, we must develop knowledge. To develop knowledge, we must collect, organize,
and disseminate information. In this sense, botanical information has conservation value. Before
we can use plant-based products from the forest, we must know what species are useful. We must
know what their names are in order to be able to communicate accurately about them. We must be
able to identify them, to know where they occur in the forest, how of them exist, how they are
pollinated and when they produce fruit (or other useful products). Aside from understanding the
species as they occur locally at Los Amigos, we must have information about their overall distribution
in tropical America in order to better understand and manage the distribution, variation, and viability
of their genetic diversity and germplasm. This involves a more complete understanding of the species
through studies in the field and herbarium.
71. The phrase genetic resources refers to .
A. plant seeds C. diverse species of plants and animals
B. different races of people D. cells that can be used in genetic cures for diseases

8|Page Tài liệu khóa học Live - VIP


OTTO CHANNEL
Otto Channel
72. In paragraph 2. the author emphasizes that the current environmental condition of Amazonian
South America is .
A. mostly unscathed C. irredeemable everywhere but in the Los Amigos watershed
B. restorable through his project D. varying from destroyed to virtually pristine
73. The word encroachment in paragraph 2 is closest in meaning to .
A. intrusion B. augmentation C. infringement D. seepage
74. The author implies in paragraph three that the agreement between Peru and the Amazon
Conservation Association is history primarily because it .
A. was the first long-term agreement regarding land in the Amazon Rainforest
B. represented the first time a South American government had agreed to renew a conservation
agreement
C. is essentially a permanent conservation agreement
D. represents the first time such an agreement has been in the form of a renewable contract
75. The author's main purpose in the passage is to
A. demonstrate that conservation efforts have been historically successful and so should be
continued
B. garner support for opposition to destructive activities in the Los Amigos watershed
C. position the Los Amigos watershed agreement as a success towards the achievement of the
vital goal of conservation the Amazonian rainforests
D. argue that the study pristine rainforests is essential for documenting and studying the myriad
new species that the forests contain
76. The author's tone in the passage can be best described as .
A. advocacy for his project over the other competing projects
B. general praise for conservation projects in Amazonian South America
C. passionate support for his and related projects
D. zealous advocacy for his point of view
77. The work of Louise Emmons, Robin Foster, and Michael Goulding (in paragraph 4) are employed
in the passage as .
A. colleagues of the author's in his botanical project
B. examples of the kinds of activities the author and his colleagues are trying to halt
C. scientists who are representative of new trends of study in Amazonian botany
D. scientists involved in projects related and amenable to the author's
78. The author's botanical project involved all of the following EXCEPT
A. studying plants in laboratory
B. studying how plants are used by humans and animals
C. facilitating pharmaceutical use of plants
D. labeling plants in the Los Amigos area
79. When the author says that the botanical project will provide names he means that the project
will .
A. help recognize new species
B. aid in the standardization of names for new species

9|Page Tài liệu khóa học Live - VIP


OTTO CHANNEL
Otto Channel
C. participate in naming the region's different zones
D. clarify the conclusion surrounding the names of different organizations working in Amazonia
80. When the author says that botanical information has conservation value he
A. a robust understanding of conservationism is aided by botanical information
B. conservationists should strive to preserve botanical information
C. specification is of importance for conservation means that
D. political discussions about conservation should use botanical nomenclature

SECTION B. WRITTEN TEST (60 PTS)


PART I. CLOZE TEST (20 PTS)
Fill in each numbered blank with ONE suitable word.
PASSAGE A
A new threat to our health seems to have arisen in our midst, confusion and stress brought (1)
by technology. All you need to do to prove this to (2) is to telephone a large company; a
recorded voice will (3) you with a bewildering list of choices, and when you have finished
answering its questions, you will probably be subjected to several minutes of piped music before you
eventually make contact with a human being. But the stress you undergo as a result is negligible
compared to the (4) the telegraph made on people 150 years ago. Until (5) , messages
could only travel as fast as a messenger could carry them. But now they could be sent great (6)
in seconds. Before long, (7) cables were laid across the oceans, and thirty years later, the
network reached 20,000 towns around the world.
Information arrived so quickly, often contradicting what had previously been transmitted. (8)
businessmen had to work much harder to (9) abreast of developments. If we find
difficulty with the Internet, which is technological evolution, (10) revolution, our ancestors
had harder task in getting used to the invention in the first place.
PASSAGE B
Negative impacts from tourism occur when the level of visitor use is greater than the
environment’s (11) to cope with this use within the acceptable limits of change. Uncontrolled
conventional tourism poses potential (12) to many natural areas around the world. It can put
enormous pressure on an area and lead to impacts such as soil erosion, increased pollution, (13)
into the sea, natural habitat loss, increased pressure on endangered species and heightened
(14) to forest fires. It often puts a strain on water resources and it can force local populations to
compete for the use of critical resources.
Water, and especially fresh water, is one of the most critical natural resources. The tourism
industry generally (15) water resources for hotels, swimming pools, golf courses and personal use
of water by tourists. This can result (16) water shortages and (17) of water supplies,
as well as generating a greater volume of waste water. Tourism can create great pressure on local
resources like energy, food and other raw materials that may already be in short supply. Greater
extraction and transport of these resources exacerbates the physical impacts associated with their
exploitation. Because of the (18) character of the industry, many destinations have ten times

10 | P a g e Tài liệu khóa học Live - VIP


OTTO CHANNEL
Otto Channel
(19) inhabitants in the high season than in the low season. A high demand is placed upon
these resources to (20) the high expectations tourists often have (proper heating. hot water, etc.)
PART II. WORD FORMATION (20 PTS)
A. Supply the correct forms of the words given.
1. To the , most computer systems seem complex and difficult to understand. (INITIATIVE)
2. The pieces of evidence fell into place with the precision of a well-made jigsaw puzzle.
(VOICE)
3. Parents have deep about allowing business values to be used in schools. (GIVE)
4. For many people, social networking offers them a feeling of from the real world. (ESCAPE)
5. Not sick. Mai guessed, but probably now that she drank a lot at the party last night.
(HANG)
6. The boats surrounded the whales, drove them into nets, where they became and
were rendered helpless by harpoon thrusts. (MESH)
7. "P" is a consonant. (LIP)
8. Perhaps , recent computer modeling studies predict fewer tropical cyclones if the
ocean heats up further as a result of global warming. (INTUITION)
9. New immigrants have been successfully into the community. (SIMILAR)
10. We have to learn good examples, to look at our behavior and to stop being . (RIGHT)

B. Fill in the blank with an appropriate form of one of the words given to make a meaningful
passage.
bridge live allegation sequential name
healthy corporate accompany license gainful

William Sydney Porter (1862-1910), who wrote under the (11) of O. Henry, was born in
North Carolina. His only formal education was to attend his Aunt Lina's school until the age of fifteen,
where he developed his (12) love of books.
By 1881 he was a (13) pharmacist. However, within a year, on the recommendation of a
medical colleague of his Father's, Porter moved to La Salle County in Texas for two years herding
sheep. During the time, Webster's (14) Dictionary was his constant (15) , and Porter
gained a knowledge of ranch life that he later (16) into many of his short stories. He then moved
to Austin for three years, and during this time the first recorded use of his pen name appeared, (17)
derived from his habit of calling “Oh, Henry" to a family cat. In 1887, Porter married Athol
Estes. He worked as a draftsman, then as a bank teller for the First National Bank.
In 1894 Porter founded his own humor weekly, the "Rolling Stone", a venture that failed within a
year, and later wrote a column for the Houston Daily Post. In the meantime, the First National Bank
was examined, and the (18) indictment of 1886 stated that Porter had embezzled funds. Porter
then fled to New Orleans, and later to Honduras, leaving his wife and child in Austin. He returned in
1897 because of his wife's continued (19) , however, she died six months later. Then, in 1898
Porter was found guilty and sentenced to five years' imprisonment in Ohio. At the age of 35, he entered

11 | P a g e Tài liệu khóa học Live - VIP


OTTO CHANNEL
Otto Channel
prison as a defeated man; he had lost his job, his home, his wife, and finally his invented name he
now used to hide his identity. He wrote at least twelve stories in jail, and after (20) his freedom,
went to New York City, where he published more than 300 stories and gained fame as America's
favorite short story writer. Porter married again in 1907, but after months of poor health, he died in
New York City at the age of 48 in 1910. O. Henry's stories have been translated all over the world.
PART III. ERROR IDENTIFICATION (10 PTS)
Identify 10 mistakes in this passage and suggest corrections.
1 Preserving organisms in museums is one way of retaining them for posterity, but almost of
2 people agree that it would be nice to keep a few of them live in the wild, too. At the moment,
3 which species survive, which decline to threatened or even status and which succumb for
4 extinction is something of a lottery. WORLDMAP is an easy-to-use software that identified
5 geographical patterns in diversity, rarity and conservation priorities. It can perform a range
6 specialist biological analysis infinitely countless numbers of species, with a view to provide
7 biodiversity data for research purposes. The program divides the surface area of the world
8 into cells, usually arranging in a rectangular grid. WORLDMAP can also predict the likelihood
9 of a hitherto unobserved species found in an area on the basis of theirs known distribution.
10 Given the patchiness of most records, which is a useful trick. Furthermore, it can select
11 complementary areas for preservation. Those are not necessarily cells with the highest
12 individual biodiversity, but for those which, together, maximize what is preserved by picking
13 places with the least overlapping species.

Lines Mistake Correction Lines Mistake Correction

PART IV. SENTENCE TRANSFORMATION (10 PTS)


Rewrite the following sentences using the words given.
1. Linda was very nervous, which made her look like a bashful girl. (CAME)
→ Such ..............................................................................................................................
2. As soon as the funds ran out, they had to abandon the scheme. (PETERED)
→ The instant .....................................................................................................................
3. Why did you reveal my plan to Kathy? (BREATHED)
→ I’d .................................................................................................................................
4. David was responsible for the family business as soon as his father retired. (CHARGE)
→ Scarcely .........................................................................................................................
5. Your encouragement helped to make things less grievous after such a heavy loss. (CUSHION)
→ It was ............................................................................................................................

12 | P a g e Tài liệu khóa học Live - VIP


OTTO CHANNEL
Otto Channel
6. Tina was crazy about stamps, so she spends lots of money on them every month. (SPLASHED)
→ Had it .............................................................................................................................
7. He tried hard but couldn't compensate for what he had done. (AMENDS)
→ Try ................................................................................................................................
8. Experts think that all dogs evolved from wolves. (DESCENDED)
→ All dogs ............................................................................................................... experts.
9. We didn't learn he still managed to live with very little money as a waiter until later. (EKED)
→ Not until .........................................................................................................................
10. Nobody is certain if the project will be permitted to continue. (GO-AHEAD)
→ It's still touch ...................................................................................................... or not

THE END

13 | P a g e Tài liệu khóa học Live - VIP


OTTO CHANNEL
Otto Channel
SỞ GIÁO DỤC & ĐÀO TẠO KỲ THI OLYMPIC TRUYỀN THỐNG 30/4
TP. HỒ CHÍ MINH LẦN THỨ XXV – NĂM 2019
ĐỀ CHÍNH THỨC Môn thi: Anh văn - Khối: 10
Thời gian làm bài: 180 phút

❖ Thí sinh làm phần trắc nghiệm (MULTIPLE CHOICE) trên phiếu trả lời trắc nghiệm và phần tự
luận (WRITTEN TEST) trên phiếu trả lời tự luận.
❖ Trên phiếu trả lời trắc nghiệm, thí sinh tô thêm 2 số 00 vào trước số báo danh (bằng bút chì).
Phần mã đề thi trên phiếu trắc nghiệm, thí sinh tô vào ô 002.
 --------------------------------------------------------------------------------------------------------------------

CẤU TRÚC ĐỀ THI


SECTION A. MULTIPLE CHOICE (40 PTS) ............................................................................ 1
PART I. GRAMMAR AND STRUCTURES (5PTS) .................................................................................................. 1
PART II. PHRASAL VERBS AND PREPOSITIONS (5 PTS)................................................................................ 2
PART III. VOCABULARY (10 PTS) ................................................................................................................................ 2
PART IV. GUIDED CLOZE (10 PTS) ............................................................................................................................ 4
PART V. READING COMPREHENSION (10 PTS) ................................................................................................. 5

SECTION B. WRITTEN TEST (60PTS) ................................................................................. 10


PART I. CLOZE TEST (20 PTS) ................................................................................................................................ 10
PART II. WORD FORMATION (20PTS) .................................................................................................................. 10
PART III. ERROR CORRECTION (10PTS) ............................................................................................................... 11
PART IV. SENTENCE TRANSFORMATION (20 PTS) .......................................................................................... 12
Otto Channel
SỞ GIÁO DỤC & ĐÀO TẠO KỲ THI OLYMPIC TRUYỀN THỐNG 30/4
TP. HỒ CHÍ MINH LẦN THỨ XXV – NĂM 2019
ĐỀ CHÍNH THỨC Môn thi: Anh văn - Khối: 10
Thời gian làm bài: 180 phút

❖ Thí sinh làm phần trắc nghiệm (MULTIPLE CHOICE) trên phiếu trả lời trắc nghiệm và phần tự
luận (WRITTEN TEST) trên phiếu trả lời tự luận.
❖ Trên phiếu trả lời trắc nghiệm, thí sinh tô thêm 2 số 00 vào trước số báo danh (bằng bút chì).
Phần mã đề thi trên phiếu trắc nghiệm, thí sinh tô vào ô 002.
 --------------------------------------------------------------------------------------------------------------------
SECTION A. MULTIPLE CHOICE (40 PTS)
PART I. GRAMMAR AND STRUCTURES (5PTS)
Choose the best options to complete the following sentences.
1. ______ happy in the new school, he missed his old friends.
A. Usually B. Although C. Being D. Even
2. ______ we leave at 3:00, we should get there by 5:30.
A. Assuming B. Having assumed C. To assume D. Assumed
3. I am sorry to keep you waiting. I hope you ______ long.
A. are not waiting B. don’t wait C. haven’t waited D. haven’t been waiting
4. It's about time you ______ the balcony. It's covered in leaves and dust.
A. cleaned B. had cleaned C. to be cleaned D. to have cleaned
5. Don't be silly! That ______ possibly be David Beckham!
A. mustn't B. shouldn't C. won't D. can't
6. “I locked myself out of my apartment. I didn't know what to do.”
“You ______ your roommate.”
A. could have called B. may have called
C. would have called D. must have called
7. “Did the principal of the school answer you yet?”
“No, but ______ I hear from him by 5pm, I’ll let you know.”
A. might B. could C. would D. should
8. ______ help me make this decision. I’m just so unsure of which direction to take for my future.
A. I’d sooner you will B. I wish you will
C. If only you could D. I’d rather you
9. ______ we have enough money, where would you like to travel this summer?
A. So that B. Provided that
C. Despite the fact that D. Unless

1|Page Tài liệu khóa học Live - VIP


OTTO CHANNEL
Otto Channel
10. Twenty people were arrested during the demonstration, of ______ four were charged with
obstruction.
A. who B. whom C. which D. them
PART II. PHRASAL VERBS AND PREPOSITIONS (5 PTS)
Choose the best options to complete the following sentences.
11. She was very appreciative ______ all the support she got from her friends.
A. of B. for C. on D. with
12. Her latest novel is coming ______ in paperback soon.
A. across B. off C. out D. about
13. As you’ve arrived late, you’ll have to ______ the time you have lost.
A. make up to B. do up to C. do up for D. make up for
14. Mary is jealous ______ her sister because she is much more popular.
A. on B. of C. for D. with
15. Your skirt needs taking ______; it's too large.
A. up B. on C. over D. in
16. Can you make ______ the meaning of this passage?
A. out B. for C. up D. into
17. He’s such a hard man to ______ as he’s always flitting from one site to another.
A. pin in B. lock in C. narrow down D. nail down
18. Don't be put ______ by his manner. He always acts that way.
A. on B. away C. off D. down
19. His proposal met ______ total opposition from the committee.
A. about B. by C. for D. with
20. You ought to stand ______ your little brother when the others tease him.
A. over with B. by for C. about with D. up for
PART III. VOCABULARY (10 PTS)
Choose the best options to complete the following sentences.
21. It is possible to ______ out of the pension scheme if you do not wish to participate.
A. back B. charge C. opt D. break
22. She always gets what she wants because she knows how to ______ the rules.
A. circumvent B. desert C. slack D. elicit
23. Anthropologists have spent years studying the social system of this ______.
A. breed B. caste C. tribe D. sect
24. I tried to catch the mouse, but it was too ______.
A. intentional B. obsolete C. uncommon D. elusive

2|Page Tài liệu khóa học Live - VIP


OTTO CHANNEL
Otto Channel
25. I have no appetite and I am lethargic. I've been feeling under ______ for ages.
A. pair B. stress C. par D. threat
26. Peter does everything himself because he doesn’t like to ______ control.
A. extinguish B. relinquish C. vanish D. elicit
27. I don't know how I can ______ up the courage to tell him the awful news.
A. pick B. pluck C. store D. set
28. I need to study more for the test. I don’t have a very good ______ of the material.
A. abstract B. grasp C. hint D. gist
29. The weekend is over, so tomorrow morning it's back to the ______.
A. grind B. labour C. drudgery D. toil
30. We could hear the monkey ______ long before we reached their cage.
A. chatting B. chattering C. prattling D. babbling
31. Bob is so short-tempered; he should try to ______ his anger.
A. monitor B curb C temper D stunt
32. The police ______ the woods looking for the lost child.
A. scoured B. integrated C. traced D. encountered
33. Don't mention work to Ray, as it's a sore ______ with him at the moment.
A. finger B. point C. place D. nail
34. The couple ______ under the umbrella to keep dry.
A. enclosed B. muffled C. huddled D. augmented
35. A long, green snake ______ through the grass and disappeared.
A. strutted B. slunk C. slithered D. scampered
36. This schedule isn’t final. It’s only ______.
A. tentative B. sporadic C. contemporary D. subsequent
37. My new pullover______ to half its previous size when I washed it.
A. shrank B. reduced C. diminished D. dwindled
38. The new accounting system ______ all my work useless.
A. transformed B. rendered C. transposed D. converted
39. Martin just loves to ______ his teeth into a really challenging crossword.
A. grind B. get C. put D. sink
40. The doctor said that sweets should be eaten in ______.
A. compulsion B. restriction C. moderation D. qualification

3|Page Tài liệu khóa học Live - VIP


OTTO CHANNEL
Otto Channel
PART IV. GUIDED CLOZE (10 PTS)
Read the texts below and decide which answer best fits each space.
PASSAGE 1
The case of food storage in tin cans illustrates how an invention can be successful even when it
(41) ______ a new problem for the one it solves. In 1810, Peter Durand (42) ______ tremendous
progress in food preservation with his invention of the tin can. The cans were ideal for transporting
food and for (43) ______ spoilage for extended periods. But consumers of canned food had to (44)
______ their own ingenuity to open the solidly constructed cans, which sometimes outweighed the
foodstuffs inside. Durand’s solution had created a new problem, (45) ______ opening the cans. In the
early days, the (46) ______ most favored was the brute force of a hammer and chisel. In 1858, Ezra
Warner designed a crude and somewhat (47)______ can opener. Finally (48) ______ 1930, the safe
design that underlies today’s common kitchen utensils had been invented. From the (49) ______ of
their introduction, however, the problem of opening the cans (50)______ not detract from their wide
acceptance as a solution to the problem of food preservation.
41. A. substitutes B. becomes C. causes D. is
42. A. provided B. invented C. made D. discovered
43. A. storing B. preserving C. keeping D. preventing
44. A. create B. know C. use D. make
45. A. easily B. especially C. hardly D. namely
46. A. consumer’s B. method C. can D. food
47. A. called B. dangerous C. safer D. sophisticated
48. A. in B. beyond C. prior D. by
49. A. latest B. way C. time D. first
50. A. did B. does C. could D. will
PASSAGE 2
Two eyes help us to see in three dimensions; and two ears allow us to hear in stereo. If recent
research findings are (51) ______, two nostrils function in (52) ______ the same way. In the olfactory
system, each nostril conveys to the brain a slightly different olfactory (53) ______ upon smelling an
odor. What is perceived in combination is (54) ______ used by the brain to get a precise (55) ______
of what the odor is.
The slight difference (56) ______ the olfactory images seems to occur (57) ______ the air flows at
a different rate through each nostril, one of (58) ______ has a low flow-rate and the other a high one.
Odors that dissolve slowly have their maximum effect in the (59) ______ that has a slow movement
of air. Conversely, those that dissolve quickly have their (60) ______ effect when the air stream is
moving rapidly. Thus, air drawn into the nose will give different responses in each nostril. Not yet well
understood is how the brain processes the disparate olfactory images as a single recognizable aroma.
Future studies will be needed to completely explain this phenomenon.

4|Page Tài liệu khóa học Live - VIP


OTTO CHANNEL
Otto Channel
51. A. incorrect B. reported C. accurate D. similar
52. A. so B. as C. much D. such
53. A. process B. image C. where D. once
54. A. which B. earlier C. strongly D. then
55. A. meaning B. measurement C. location D. sense
56. A. in B. about C. to D. when
57. A. before B. then C. however D. because
58. A. those B. which C. each D. such
59. A. nostril B. chemical C. flow D. other
60. A. personal B. strongest C. negative D. slowest
PART V. READING COMPREHENSION (10 PTS)
Read the texts below and choose the best answer to each question.
PASSAGE 1
Recently, researchers conducted a survey to investigate what motivates young adults to exercise.
The study might shed light on why so few college students exercise and why many stop their exercise
workouts after they graduate. Researchers analyzed responses from 937 randomly selected college
students at a leading U.S. university. What the research team found was that 39% of the male students
and 26% of the female students exercised at least three days a week for 20 minutes at a time. These
exercise patterns were fairly similar to those in surveys at other universities. The survey also revealed
some critical factors which motivate college students to exercise.
Men who exercised regularly reported that their friends provided considerable support for them to
do so. Those who exercised only occasionally had moderate support from their friends, while those
who did not exercise at all had little or no support from friends. For women, however, the crucial
motivating factor seemed to come from family members rather than from friends. Women who
exercised regularly had a great deal of positive support to do so from their families. However, women
who exercised only occasionally received moderate levels of encouragement from their families, while
those who did not work out at all received little or no support from their families. Since college students
often live far from home, women students are likely to have a weaker support system than do their
male counterparts.
A medical professor, upon seeing the results of the study, remarked that generally there is not so
much emphasis on body performance among girls and young women. “It’s a macho thing to be able
to run faster or lift weights,” he said. “However,” he continued, “for the younger generation, this
gender gap appears to be getting smaller.”
61. What is the main purpose of the study presented in this passage?
A. to investigate college students’ attitudes towards good health and exercise
B. to find out why young women don’t like to exercise
C. to learn what makes college students exercise
D. to find the best type of exercise workout for college students

5|Page Tài liệu khóa học Live - VIP


OTTO CHANNEL
Otto Channel
62. What research method was used in this study?
A. College students reported on their friends’ exercise workouts.
B. College students reported on their families’ exercise patterns.
C. College professors answered questions about their students.
D. College students answered questions about themselves.
63. What similarity between men and women did the study find?
A. The more friends they have, the more they exercise.
B. The more support they have to exercise, the more they exercise.
C. The closer they live to their families, the more they exercise.
D. The more health-conscious they are, the more they exercise.
64. In comparison to the general college population, students at this university exercise ___.
A. less often than average B. about the same amount as average
C. more often than average D. for longer at a time than average
65. Why does the passage mention that some college students live far from home?
A. to emphasize that they do not see their families every day
B. to criticize their life styles
C. to show how independent they are
D. to explain why they depend on their friends so much
PASSAGE 2
It has long been known that underweight, premature babies develop into children who perform
worse at school than children who had normal birth weight and were full-term. A recent study
examining the effects of birth weight on intelligence suggests that even among full-term babies the
heavier ones have an advantage. The study has been following 3,900 British men and women since
their birth in 1946. Birth weight was correlated with scores on tests of reading and arithmetic skills,
non-verbal reasoning, memory, speed, and concentration. These tests were conducted when the
participants were 8, 11, 15, 26, and 43 years old. Generally speaking, the heavier children performed
better on the tests. The relationship was strongest at age 8 and then weakened over time. By age 43,
the relationship was negligible. The results were not affected by birth order, gender, father’s social
class, or mother’s education and age.
These findings must be interpreted with caution. First, the results were based on averages. Second,
birth weight is only one of numerous factors influencing cognitive function. Parental interest in
education - such as volunteering at school or helping with homework - may offset the effect of birth
weight. Furthermore, poor environmental conditions, such as living in an overcrowded home, breathing
polluted air, or being caught in a bitter divorce can diminish the early advantages enjoyed by heavier
babies.
Although no one knows exactly what makes a newborn baby heavy, it is known that healthy, well-
nourished mothers tend to have heavier babies, while those who eat poorly, smoke, and are heavy

6|Page Tài liệu khóa học Live - VIP


OTTO CHANNEL
Otto Channel
drinkers tend to produce smaller ones. There are probably several other variables that affect birth
weight, but if and how those are connected to intelligence is not known.
66. In this study, the researchers examined the relationship between _______.
A. maternal nutrition and birth weight
B. home environment and intelligence
C. age and intelligence
D. intelligence and birth weight
67. What did the researchers find out about the relationship they were studying?
A. It increased up to age 26, then decreased.
B. It remained steady until age 26, then decreased.
C. It decreased as the subjects grew older.
D. It remained steady throughout the study.
68. Which children have an advantage at school?
A. Those who were full-term, lighter weight babies.
B. Those who were premature but normal weight babies.
C. Those who were full-term, heavier babies.
D. Those who were premature babies who gained weight quickly.
69. According to the passage, what is one possible reason to question the conclusions of the study?
A. The study did not consider some environmental factors.
B. The study was done too long ago.
C. The study was done on premature babies, not full-term ones.
D. The study tested different children at different ages.
70. What is the main idea of the passage?
A. Premature babies are disadvantaged throughout their lives.
B. Parents’ educational level affects intelligence.
C. No one knows the exact relationship between test scores and intelligence.
D. Bigger babies may be more intelligent than smaller ones.
PASSAGE 3
Diving deep into the ocean is difficult for humans, although whales do it easily. The deepest a
human has dived into the ocean without scuba gear is to 450 feet below the surface, which required
him to hold his breath for about two minutes. Humans are unable to go much deeper because at a
certain point the external pressure becomes so great it crushes the ribcage and lungs. Furthermore,
lack of oxygen also prevents humans from staying under water for long periods. It is amazing, then,
that whales can dive to depths of up to 10,000 feet and remain there for as long as two hours. Several
adaptations enable the whale to do this, among them the construction of its ribcage. Unlike humans,
whales have fewer “true” ribs, ribs joined directly to the breastbone, and more “floating” ribs, which
are attached only to the preceding rib by cartilage. This flexible design allows the whale’s ribcage to

7|Page Tài liệu khóa học Live - VIP


OTTO CHANNEL
Otto Channel
fold under pressure instead of breaking, and for its lungs to collapse without rupturing capillaries. Also,
during a dive, the whale’s heart slows to as low as 3 to 5 beats a minute. Arteries constrict, reducing
blood flow to many of the whale’s less vital organs, thus conserving oxygen and maintaining blood
pressure in the brain and heart. Humans emerging too suddenly from dives can suffer from “the bends,”
when small bubbles of nitrogen gas form in body fluids and obstruct blood flow, leading to death.
Because whales collapse their lungs, air is pushed from the lungs into the windpipe, which is lined with
thick membranes, making it difficult for nitrogen bubbles to pass into the bloodstream or other tissues.
Together, these and other adaptations enable whales to dive to such depths.
71. What are “floating” ribs?
A. Ribs that are not attached to other ribs.
B. Ribs that are not attached to the breastbone.
C. Ribs that are made of cartilage.
D. Ribs that are not able to fold under pressure.
72. What is the main purpose of this passage?
A. to explain why whales can do something that humans cannot
B. to explain how humans and whales adapted
C. to explain what happens at great depths
D. to explain how the human respiratory system works
73. When a whale dives, what contributes to the conservation of oxygen?
A. Blood pressure increases. B. Blood flow decreases.
C. The lungs collapse. D. The ribcage folds.
74. Compared to whales, humans have ______.
A. weaker capillaries
B. thicker membranes in the windpipe
C. fewer floating ribs
D. slower heartbeats
75. How does the whale’s ribcage differ from that of a human?
A. The whale’s ribcage contains fewer vital organs.
B. The whale’s ribcage can bend without breaking.
C. Whales have fewer ribs than humans.
D. The whale’s ribs are not attached to the breastbone.
PASSAGE 4
Scientists have not been able to determine the exact age of the Earth directly from its rocks.
Geologists have yet to find any of Earth’s original rocks that have not been recycled and destroyed
through the process of plate tectonics. However, scientists have been able to determine the probable
age of our solar system and to calculate an age for the Earth by looking elsewhere - to outer space.

8|Page Tài liệu khóa học Live - VIP


OTTO CHANNEL
Otto Channel
In doing so, they had to assume that the Earth and the rest of the solid bodies in our solar system
formed at the same time and are the same age.
Asteroids in outer space have not been subjected to the crushing forces of plate tectonics.
Meteorites, which are fragments of asteroids that fall to Earth, contain clues about planetary formation.
Being primordial rocks still in their original state, they can be dated fairly accurately by measuring the
radioactive elements remaining in them since the formation of the solar system. One group of scientists
from Germany and another from France analyzed meteorites and reached the same conclusion: The
final phase of Earth’s formation, the separation of its metallic core from its silicate-based mantle,
occurred about 30 million years earlier than previously thought.
The ages of more than seventy meteorites have been measured using radiometric dating
techniques. The results show that meteorites, and therefore presumably Earth, formed about 4.5 billion
years ago. These findings also push back the origin of Earth’s moon because most scientists believe
that the Moon formed from material ejected when a Mars-sized planet collided with Earth. The fact
that these conclusions about the age of the Earth were reached by two independent groups of scientists
increases the weight of the findings.
76. What strengthens the claim made in this passage about Earth’s age?
A. Earth and meteorites are the same age.
B. Two separate research studies had the same results.
C. Geologists have found Earth’s original rocks.
D. The moon is older than previously thought.
77. What conclusion did the French and German scientists reach?
A. Meteorites have radioactive elements.
B. Plate tectonics does not occur on asteroids.
C. The Earth is older than was believed in the past.
D. The Moon is older than the Earth.
78. According to the passage, what is involved in “radiometric dating techniques”?
A. using radioactivity to restore an object to its original state
B. measuring the radioactive elements in an object
C. calculating how many times Earth’s rocks were recycled
D. subjecting an object to crushing forces
79. How did scientists date the Earth?
A. directly, by analyzing plate tectonics B. directly, by measuring Earth’s metallic core
C. indirectly, by dating the Earth’s Moon D. indirectly, by dating meteorites
80. What indicated the last stage in the Earth’s formation?
A. a reduction in the crushing force of plate tectonics
B. the separation of Earth’s core from its mantle
C. a shower of seventy meteorites falling to Earth
D. the collision of a Mars-sized planet with Earth
9|Page Tài liệu khóa học Live - VIP
OTTO CHANNEL
Otto Channel
SECTION B. WRITTEN TEST (60PTS)
PART I. CLOZE TEST (20 PTS)
Read the texts below and complete each space with ONE suitable word.
CLOZE TEST 1
Fallingwater is recognized as one of the most unique and innovative (1) of American
architecture of the 20th century. Designed in 1935 by architect Frank Lloyd Wright, this beautiful house
in western Pennsylvania is actually built over a waterfall. Wright placed the house above the waterfall
by anchoring it to the (2) next to the falls with concrete “trays,” (3) mimic the natural
shape of rock ledges. As a(n) (4) , the house appears to be suspended above the waterfall,
which cascades underneath it.
Wright took much care in (5) Fallingwater as harmonious with nature as possible by linking
the house to its natural surroundings. He placed large windows on opposite sides of the main rooms
so that the breeze and (6) of the water could flow through the house (7) hindrance.
Wright also proposed covering the building in gold leaf in (8) to imitate the color of dying
plants, and thereby connect the house to the (9) of seasons and the passage of time.
Furthermore, he added a (10) of outdoor terraces where people could enjoy the cool air of the
wooded valley.
Today, Fallingwater is open to the public with its original setting, furnishings, and artwork intact.
CLOZE TEST 2
Global warming may be threatening one of the world’s most important crops: rice. Increased
nighttime temperatures are associated with significant declines in crop (11) , according to
a study conducted by researchers at the International Rice Institute. The researchers analyzed twelve
years of rice production along with twenty-five years of temperature data. This study, a direct
measurement of yields produced under normal field conditions, using (12) that good
farmers normally employ, has confirmed previous simulations and suggests that (13) increases
due to global warming will make it increasingly difficult to feed the Earth’s growing population.
Average (14) temperatures, which increased 0.35 degrees Celsius over the (15) of the
study, have little effect on rice production. However, a strong correlation exists between warmer
nighttime temperatures, which have risen an average of 1.1 degrees, (16) decreasing
rice yields. Although the underlying (17) of this relationship is unknown, researchers speculate
that plants work harder to maintain themselves in (18) to the warmer nights and consequently
divert energy from growth. Thus, an average (19) of just one degree Celsius can (20) in a
ten percent reduction in yields. Similar findings have been reported for corn and soybean yields in the
United States.
PART II. WORD FORMATION (20PTS)
A. Complete each sentence, using the correct form of the word in parentheses.
1. They were in search of archaeological remains. (DATE)

10 | P a g e Tài liệu khóa học Live - VIP


OTTO CHANNEL
Otto Channel
2. He always gets aggressive when he’s drunk. (RESTRAIN)
3. The drop in share prices in March was a of the financial crash that followed in June. (RUN)
4. She sang the first three verses with a piano and the last verse . (COMPANY)
5. I really didn't mean to offend her - I just said it . (THINK)
6. They were accused of in their treatment of the hostages. (HUMAN)
7. Alcohol is a factor in 10% of all road accidents. (CONTRIBUTE)
8. He had to face up to his own as a father. (ADEQUATE)
9. It was a step in the right direction. (QUESTION)
10. Hospital beds were scarce and medicines were practically . (EXIST)
B. Complete the passage with the appropriate forms from the words given in the box.
complicate stand dine welcome depend
occur defend terror delicate expose

POISONOUS SEALIFE
One of the most lethal poisons on Earth, ten thousand times more deadly than cyanide, is
tetrodotoxin, more concisely known as TTX. Its potency is well known in East Asia, where it regularly
kills (11) who have braved the capricious (12) known as puffer fish. This toxin has a (13)
method of operation: twenty-five minutes after (14) , it begins to paralyse its victims,
leaving the victim fully aware of what is happening. Death usually results, within hours, from
suffocation or heart failure. There is no known antidote. If lucky patients can (15) the
symptoms for twenty-four hours, they usually recover without further (16) . It is no ordinary
poison. What is strange about its (17) is that it is found in such a wide range of creatures, from
algae to angelfish spanning entire kingdoms of life. It is rather unlikely that such an unusual toxin
evolved (18) in so many unrelated animals. Marine biologists have discovered that the poison
is produced by bacteria living in the gut of its host. The best explanation is that a symbiotic relationship
exists between host and the not (19) guest, where microbes exchange poison for nutrients,
providing a valuable (20) weapon for its host.
PART III. ERROR CORRECTION (10PTS)
The following passage contains 10 errors. Identify and correct them.
BEWARE OF VITAMINS!
1 Vitamins are good for our health, aren't they? Perhaps not. New research suggests that rather
2 than ward off disease, high doses of certain vitamins may make more harm than good and
3 could even put you in an early grave. Some recent studies suggest that far from improving
4 health, these vitamins, when taken at very high doses, may actually increase the risks of
5 cancer and a range of debilitating diseases, a discovery that has sent the medicinal world into
6 a spin. Scientists are unsure as to when vitamins, so essential to health, can be toxic in high
7 doses. The most likely explanation is that the body is only equipped to deal with the levels

11 | P a g e Tài liệu khóa học Live - VIP


OTTO CHANNEL
Otto Channel
8 found naturally in the environment. If the intake is too far for the normal range, then the
9 body's internal chemistry must be shunted out of alignment. What this means is that the
10 commercially sold vitamins and those provided by nature is not always compatible. The
11 commercial forms may interfere with the body's internal chemistry by “crowding out” the most
12 natural and beneficial forms of the nutrients. The vitamins obtained in food are also allied with
13 a host of other substances which may moderate or augment its activity in the body. The latest
14 advice is to eat a balance diet to ensure you get all the nutrients you need, and if you must
15 take supplements make sure you take the lowest recommended dose and follow the

Lines Mistake Correction Lines Mistake Correction

PART IV. SENTENCE TRANSFORMATION (20 PTS)


Rewrite the following sentences using the words given.
1. People know more about my novels than the plays I write. (BETTER)
→ I'm ................................................................................................................................
2. The staff hated his new policies intensely and so went on strike. (HATRED)
→ So intense ....................................................................................................................
3. Your attitude to life would be greatly improved by regular exercise. (WONDERS)
→ Regular exercise would ..................................................................................................
4. There is someone in the office twenty-four hours a day. (STAFFED)
→ The office ......................................................................................................................
5. Do you think her grandmother was offended by what I said? (EXCEPTION)
→ Do you think .................................................................................................................. ?
6. His behavior at the conference gave him the bad reputation he now has. (CONDUCTED)
→ The way .......................................................................................................................
7. Unless we can obtain more information, we can't process your claim. (FORTHCOMING)
→ Unless further ...............................................................................................................
8. Our teacher thinks it would be better to get on as quickly as possible. (MUCH)
→ Our teacher would prefer us ..........................................................................................
9. I had to wait for the manager for almost an hour before he would see me. (BEST)
→ The manager kept .........................................................................................................
10. They remain close friends despite having had many arguments. (FALL)
→ Frequently as ................................................................................................................
THE END

12 | P a g e Tài liệu khóa học Live - VIP


OTTO CHANNEL

You might also like